<<

Prelims_2.pdf Chapter-01_Pediatrics in Last Minutes.pdf Chapter-02_Pre Neet Pediatric Questions.pdf Chapter-03_Pre Neet Pediatric Answers.pdf Chapter-04_Previous Years Questions of DNB.pdf Pre NEET

Taruna Mehra MBBS MD PEDIATRICS (MAMC)

®

JAYPEE BROTHERS MEDICAL PUBLISHERS (P) LTD New Delhi • Panama City • London • Dhaka • Kathmandu ®

Jaypee Brothers Medical Publishers (P) Ltd

Headquarters Jaypee Brothers Medical Publishers (P) Ltd 4838/24, Ansari Road, Daryaganj New Delhi 110 002, India Phone: +91-11-43574357 Fax: +91-11-43574314 Email: [email protected]

Overseas Offices J.P. Medical Ltd Jaypee-Highlights Medical Publishers Inc. 83, Victoria Street, London City of Knowledge, Bld. 237, Clayton SW1H 0HW (UK) Panama City, Panama Phone: +44-2031708910 Phone: +507-301-0496 Fax: +02-03-0086180 Fax: +507-301-0499 Email: [email protected] Email: [email protected]

Jaypee Brothers Medical Publishers (P) Ltd Jaypee Brothers Medical Publishers (P) Ltd 17/1-B Babar Road, Block-B, Shaymali Shorakhute, Kathmandu Mohammadpur, Dhaka-1207 Nepal Bangladesh Phone: +00977-9841528578 Mobile: +08801912003485 Email: [email protected] Email: [email protected] Website: www.jaypeebrothers.com Website: www.jaypeedigital.com © 2013, Jaypee Brothers Medical Publishers

All rights reserved. No part of this book may be reproduced in any form or by any means without the prior permission of the publisher.

Inquiries for bulk sales may be solicited at: [email protected]

This book has been published in good faith that the contents provided by the author contained herein are original, and is intended for educational purposes only. While every effort is made to ensure accuracy of information, the publisher and the author(s) specifically disclaim any damage, liability, or loss incurred, directly or indirectly, from the use or application of any of the contents of this work. If not specifically stated, all figures and tables are courtesy of the author(s). Where appropriate, the readers should consult with a specialist or contact the manufacturer of the drug or device.

Pre NEET Pediatrics

First Edition: 2013 ISBN : 978-93-5090-314-8 Printed at Dedication I dedicate my work to my teachers,my parents and most importantly my patients. Preface

Why should I change anything in your life till the time you decide to change yourself… Quran

With the challenging task ahead in month of November and December with all the major exams within a period of 20 days, the committee has decided to release Pre NEET Pediatrics inclusive of DNB questions and important last minute revision points so that you are confident in attempting maximum questions in all the exams.This book has been written keeping in mind that maximum time taken to revise pediatrics is less than one day.

All the best may God help you cross the bridge

Taruna Mehra “Do not count the days make the days count”

From the Publisher’s Desk We request all the readers to provide us their valuable suggestions/errors (if any) at: [email protected] so as to help us in further improvement of this book in the subsequent edition. Contents

1. Pediatrics in Last Minutes ...... 1 – 65

2. Pre NEET Pediatric Questions ...... 66 – 77

3. Pre NEET Pediatric Answers...... 78 – 150

4. Previous Year’s Questions of DNB...... 151 – 174

PEDIATRICS in Last Minutes

Developmental Milestones

Age Gross motor Fine motor Language Social

1 mo Momentarily lifts Has tight grasp, Responds to Regards face head when prone follows objects sound of bell intently to midline 2 mo Holds head in No longer Smiles after Recognize midline. Lifts clenches fist being stoked or parent, social chest when prone tightly, follows talked to smile objects past midline Follows moving Coos (produces Reaches for 3 mo Head holding objects in a vowel sounds familiar people achieved circular fashion, in a musical or objects, converges and fashion), laughs anticipates focuses aloud feeding

4-5 mo Rolls over, sits Grasps objects/ Orients to Enjoys looking with support rattle crudely voice/bell around (localizes environment laterlly); "ahgoo", razzes

6 mo Takes foot to Transfers Babbles Stranger mouth, lifts head objects from anxiety, smiles and upper chest one hand to back at mirror with support another image of self

Uses pincer Responds to Responds to 9 mo Sits unassisted (8 grasp, probes name, says social play, mo), Crawls, with forefinger, mama/dada plays pat-a- cruises, pulls to holds feeding (non-specific) cake, starts to stand bottle explore environment (9-10 mo)

Contd... 2 Pre-NEET Pediatrics

Contd...

Age Gross motor Fine motor Language Social 10 mo Pulls from supine Understands to to sitting; from some "mama" sitting to standing, stands holding furniture 12 mo Gives hand held Says 'mama, Imitates actions objects to dada' (specific) mother when asked, turns 2-3 pages at a time 15 mo Walks well Uses 3-5 words Temper without support, meaningfully tantrums, Walks backwards separation and sideways anxiety 18 mo Starts to run, Makes a tower About 10 words Copies parents climb stairs with of 3-4 cubes, spoken in tasks, toilet help scribbles including name training started spontaneously, may draw a vertical line 2 yrs Runs well, climbs Makes a tower Points to at Follows 2-step stairs alone; of 6-7 cubes, least one commands walks on tiptoes turns one page named body (30 months) of a book at a part, simple time, may draw 2 word horizontal line sentences (2 words at 2 years)

3 yrs Pedals tricycle, Make a tower of Uses plurals, 3 Dresses and jumps with both 9-10 cubes, word undresses feet off ground, draws (copies) a sentences (3 partially can alternate feet circle word at buttons/ when climbing 3 years) unbuttons stairs 4 yrs Hops, alternates Draws (copies) Knows colours Buttons feet going a Cross clothing fully, downstairs (At 4 plays with other yr hop off the children floor)

5 yrs Jumps on one Draws (copies) Dresses foot, heel to toe a Square without walk supervision Pediatrics in Last Minutes 3 Calcification Age at Eruption Age at Shedding Begins At Complete At Maxillary Mandibulaur Maxillary Mandibular Mand, 3-4 mo Mand, 8-10 yr Primary Teeth incisorsCentral incisorsLateral (canines)Cuspids FirstmolarsmolarsSecond 5th fetal mo Secondary teeth 5th fetal mo 6th fetalmoCentral incisorsmo 18–24 incisorsLateral mo 18–24 30–36 mo 6th fetal momo 6–8 5th fetal moCuspids (canines)mo 8–11 16–20 momo 36 mo 24–30 mo 5–7 Firstpremolars (biscuspids) 3-4momo 16–20 mo 7–10 Secondmolars (biscuspids) 18-21mo Max, 10-12 momo 10–16 yr 7–8 yr 11–l2 First molarsyr 24-30mo 8–9 10-11yr 4-5momo 20–30 moyr 10–16 9–11 Secondmolarsyr 6–7 9-10yr 12-13yryr 7-8 mo 20–30 Third molarsyr 10–12 12-14yr 8-9yryr 10–12 yr 10–12 12-15yryr 11–13 10-11yr 7-8yr Birth 10-12yryr 7-8 30-36mo 11-l2yr 10-12yr 6-7yr Max, 7-9 yr 11-13yr 14-16yr 9-11yr 9-10yr 18-25 yr 12-13yr 17-22 yr 6-7yr 12-13yr 17-22 yr 6-7yr 4 Pre-NEET Pediatrics

• Delayed eruption is usually considered when there are no teeth by approximately 13 months of age. • Causes of delayed eruption – Idiopathic (most common), Trisomy 21, Hypopituitarism, Trisomy 21 (Down syndrome) – Hypothyroidism, – Familial, Hypoparathyroidism, Cleidocranial dysplasia Remember • Central incisors is first to develop in primary dentition. • 1st molar is first to develop in secondary dentition. • Second molar is last to develop in primary dentition. • Third molar is last to develop in secondary dentition.

Weight 10% of body weight lost in first few days of life; regained by 2 weeks. Birth weight doubles by 4 months, triples by 12 months, quadruples by 24 months. Height Height is increased by 50% at 1 year of age, doubles at 4 years and triples at 13 years. Head circumference Measured during 1-3 years of life 5 cm growth during age 0-3 months; 4 cm in 3-6 months, 2cm in 6-9 months and 1cm in 9-12 months. (newborn = 35 cm; 3 months = 40 cm, 9 months 45 cm, 3 years =50 cm, 9 years = 55cm.

SURVEILLANCE OF GROWHAND DEVELOPMENT

Surveillance of growth and development is an important component of the routine anticipatory care of children. The main purpose of growth surveillance is to identify those children who are not growing normally. Surveillance for physical growth can be done in following ways. 1. Weight for age • Measurement of weight and rate of gain in weight are the best single parameters for assessing physical growth. The weight should be carefully repeated at intervals: – Birth - 1 year  Monthly – Second year  Every two months – 2-5 years  Every 3 months Pediatrics in Last Minutes 5

• These measurements when compared with the reference standards of weight of children of same age, the trend of growth becomes obvious. • Weight for age can be used to classify malnutrition and determine its prevalence. • 80 % of the median weight for age of the reference is cut off point below which children should be considered malnourished. 2. Height for age • Height is a stable measurement of growth as opposed to body weight. • Whereas weight reflects only the present health status of the child, height indicates the events in the past also. Low height for age. • This is also known as nutritional stunting or dwarfing. • It reflects past or chronic malnutrition. • The cut off point commonly taken for the diagnosis of stunting is 90 percent of the united states NCHS height for age. 3. Weight for height • Weight in relation to height is now considered more important than weight alone. It helps to determine whether a child is within range of normal weight for his height. • Low weight for height • This is also known as nutritional wasting or emaciation (Acute malnutrition). • It is associated with increased risk of mortality and morbidity. • A child who is less than 70% of the expected weight for height is classed as severely wasted. 4. Head and chest circumference • Chest circumference At birth–Less than 2 cm from head circumference 6-9 months–Two measurements become equal >6–9 months–Overtakes head circumference • In severely malnourished children this overtaking may be delayed by 3 to 4 years. 6 Pre-NEET Pediatrics

• Wasting (deficit in weight for height)  Acute malnutrition • Stunting (deficit in height for age)  Chronic malnutrition • Wasting and stunting  Acute on chronic malnutrition • Underweight (low weight for age)  Combined indicator to reflect both acute and chronic malnutrition.

WHO Classification of Under Nutrition

Moderate undernutrition Severe undernutrition • Weight for SD score -2 to -3 SD score <-3 height (wasting) (70-79% of expected) (<70% of expected) • Height for age SD score -2 to -3 SD score <-3 (stunting) (85-89% of expected) (<85% of expected)

Age-Independent Anthropometric Indices

Normal Severely malnourished Dugdale’s Weight(kg)/ (height in cm)1.6 X 100 0.88-0.97 <0.79 Rao’s Weight(kg)/ (height in cm)2 X 100 0.15-0.16 <0.14 Kanawati Mid arm circumference (cm)/ 0.32-0.33 d”0.25 cm Head circumference (cm) MAC Midarm circumference >13.5 <12.5 cm (1-5 years)

Gomez classification is a classification system to assess Protein Energy Malnutrition of PEM • It is based on weight retardation • The child on the basis of his/her weight is compared with a ‘normal’ child of the same age • The ‘normal’ reference child is the 50th centile of the Boston standards

Weight of child Weight for age (%): 100 Weight of normal child of same age

90-110%: Normal Nutritional status 75 – 89%: Mild Malnutrition (1st degree) 60 – 74%: Moderate Malnutrition (2nd degree) Under 60%: Severe Malnutrition (3rd degree) Pediatrics in Last Minutes 7

• The Gomez system has distinct advantages and disadvantages

Advantages Disadvantages • Classification is easy to compute as • Some normal children may be weight is a widely recorded parameter classified as 1st degree malnutrition because a cut-off point of 90 percent of reference is high. • Classification has prognostic values • Does not distinguish between for hospitalized patients (This is sudden acute episode of because the cut off values were set malnutrition and along standing during a study of risk of death chronic malnutrition as observation based on weight for age at admission only consider weight for age to a hospital unit) measurements.

Facts to remember regarding other classification systems for PEM: • Indian academy of Pediatrics (IAP) classification system is based on defecit in weight for age • Wellcome Trust classification system is based on deficit weight for age and presence or absence of edema. • Waterlow classification is based on Height for age (Stunting / Chronic PEM) measurments and Weight for Height measurments (Wasting /Acute) Drop in Height for Age Ratio – Chronic PEM or Stunting Drop in Weight for Height ratio – acute PEM or Wasting • WHO classification is also based on Height for Age and Weight for Height measurement. Recommended daily intake of energy

Group Body weight Kg. Energy allowance per day K.Cals MJ

Infancy 0-6 months 118 7-12 months 108 } K.Cal/kg/day

Children 1- 3 years 12.03 1240 5.1 4- 6 years 18.87 1690 7.0 7 - 9 years 26.37 1950 8.1 8 Pre-NEET Pediatrics

Table Comparison of Human Milk and Cow’s Milk

Parameter Human Cow Bacterial contamination None Likely Anti-infective substances Antibodies, Leukocytes, Not active Lactoferrin Bifidus factor, Others

Protein Total 1% 4% (too much) Casein 0.5% 3% (too much) Amino acids Cystine Enough for growing brain Not enough Taurine Enough for brain, retina and bile acid conjugation Fat Total 4% (average) 4% Saturation of fatty acids Enough unsaturated Too much saturated Linoleic acid (essential) Enough for growing brain Not enough Cholesterol Enough Not enough Lipase to digest fat Present None Lactose 7% (enough) 4.5%

Salts (mEq/1) Sodium 6.5 (correct amount) 25 (too much) Chloride 12 (correct amount) 29 (too much) Potassium 14 (correct amount) 35 (too much) Minerals (mEq/Il) Calcium 350 (correct amount) 1,400 (too much) Phosphate 150 (correct amount) 900 (too much) Iron Small amount, but well- Small amount, absorbed (enough) poorly absorbed (not enough) Vitamin Enough Extra needed Pediatrics in Last Minutes 9

Approach to Short Stature

Bone-Age

Bone age is delayed compared to chronological age: • In almost all cases of short stature • In case of constitutional delay, undernutrition and systemic illness, bone age is less than chronological age and equals height age. • In case of growth hormone deficiency and hypothyroidism, bone age may be even less than height age if the endocrine condition is diagnosed late. Advanced bone age is common when a child has had prolonged elevation of sex steroid levels, as in: 10 Pre-NEET Pediatrics

• Precocious puberty or congenital adrenal hyperplasia (A delayed bone age in congenital adrenal hyperplasia indicates glucocorticoid over-treatment. • Bone age may be significantly advanced in genetic overgrowth syndromes, such as Soto’s syndrome, Beckwith wideman syndrome, and Marshall Smith syndrome. Delayed Bone age (Retarded skeletal maturation)

Chronic ill Health • Congenital heart disease - cyanotic • Renal failure • Inflammatory bowel disease • Malnutrition • • Maternal deprivation • Any other chronic illnesses

Endocrine Disorders • Hypothyroidism • Steroid therapy and Cushing’s disease • Hypogonadism (including Turner’s syndrome) • Hypopituitarism - panhypopituitarism, growth hormone deficiency, Laron dwarfism

Chromosome disorders • Trisomy 21, Trisomy 18

Other Congenital Disorders • Bone dysplasias • Malformation syndromes

Advanced Bone Age (Generalized Accelerated Skeletal Maturation) Endocrine disorders • Adrenal and gonadal tumours Pediatrics in Last Minutes 11

• Hyperthyroidism • Idiopathic sexual precocity • Intracranial masses in the region of the hypothalamus (haniartoma, astrocytoma, optic chiasm glioma, , ) Congenital Disorders • McCune Albright syndrome: polyostotic fibrous dysplasia with precocious puberty • Cerebral gigantism (Soto’s syndrome) • Lipodystrophy • Pseudohypoparathyroidism • Acrodysostosis • Weaver smith syndrome • Marshall syndrome D/D of Rickets

Serum Ca Serum Serum Serum Serum 3 P04 Alkaline Parathormone HCO3 phosphatase

Normal 9-10.5 mg/dl 3-4.5mg/dl 30-120 IU 10-55 units 21-30 meqlL Hypophosphatemic N NN Rickets Vit D dependent N/ (7.5-8) N/   Rickets Hyperparathyroidism /N  N Nutritional rickets  N Renal tubular  N   acidosis (distal)

Enuresis • Enuresis is defined as the voluntary or involuntary repeated discharge of urine into clothes or bed after a developmental age when bladder control should be established (mostly mental age of 5 years). • Diagnosis of enuresis requires voiding of urine twice a week for 3 consecutive months or clinically significant distress in child’s life as a result of wetting. • Enuresis is more common in males than females • Most common cause of diurnal enuresis is micturition deferral (waiting until the last minute to void). 12 Pre-NEET Pediatrics

Treatment

• First line treatment for enuresis is behavioral therapy. It consists of rewarding the child for being dry at night, child should void before retiring and the use of conditioning devices (e.g. bed alarm that rings when the child wets a special sheet). • Consistent dry bed training with positive reinforcement has a success rate of 85% and bed and pad alarm systems have a success rate of approximately 75% with relapse rate that are lower than those with pharmacotherapy. • Pharmacotherapy is second line treatment and should be reserved for those patients who have failed behavioural therapy. Imipramine and desmopressin are two important drugs useful for enuresis. • Fast action of desmopressin (orally or intranasaly) suggests a role for special occassions when rapid control of enuresis is required. However, recurrence rate is very high. • Imipramine is associated with cardiac conduction disturbances and is deadly in overdose. Pediatrics in Last Minutes 13

PRENATAL AND PERINATAL FACTORS AFFECTING FETUS

Drugs Taken During And Their Adverse Effects - Teratogenic

Drugs Adverse effects Fetal hydantoin syndrome (microcephaly, cleft palate, hypoplastic changes, IUGR) • Carbamazepine Spina bifida, ? NTD • Phenobarbitone Relatively safe • Sodium vaiproate Neural tube defect (1-2%), hypospadias, microstomia, developmental delay.

Hormonal agents • Corticosterojds Growth retardation, cleft palate and lip • Diethyl stilbestrol (used as ‘morning-after’ pill) Vaginal adenosis in female offspring in adolescence Clear cell vaginal adenocarcinoma in teenagers In male offspring (risk of testicular cancer in later life) • Anti-thyroid drugs Neonatal hypothyroidism and goiter • Clomiphene NTD, multiple gestation, Down’s syndrome • Synthetic Masculinization in female fetus, hypospadias progestins Antibiotics • ‘Gray baby syndrome’ (peripheral vascular collapses) • Sulphonamides , Methemoglobenemia • Tetracyclines Dental discoloration (yellow) and deformity, Inhibition of bony growth, cataracts, • Aminoglycosides Fetal ototoxicity due to eighth N. damage • Anti-malarials Intra-uterine death • Quinine, Retinopathy, congenital deafness, comeal opacities chloroquine Psychiatrics drugs and substances of abuse • Lithium Ebstien’s anomaly • LSD (lysergic “fractured chromosomes” anomaly in fetus, stunted growth acid diethylamide) • Alcohol Foetal alcohol syndrome: prenatal-onset growth deficiency, developmental delay, facial dysmorphism (short palpebral fissures, ptosis, strabismus, ear abnormality, long philtrum with a thin upper lip), multiple joint anomalies and cardiac defects (ASD>VSD); mental subnormality Contd... 14 Pre-NEET Pediatrics

Contd... • Heroin Irritability, hyperactivity, • Cocaine Abruptio placentae, preterm labour, cerebral infarction • Beta-blockers Foetal bradycardia

Anticoagulants • Vitamin K Hyperbilirubinemia (hemolysis) and kernicterus (large dose) • Warfarin Conradi’s syndrome: skeletal and facial anomalies, chondrodysplasia punctata, haemorrhage • Aspirin Haemorrhagic disease of newborn

Other Drugs

• Cytotoxic drugs Multiple foetal malformations and abortion • Isotretinoin CNS defects, facial palsy, deafness, cardiac defects • Oxygen in high Retrolentalfibroplasia and blindness (>35%) concentrations • Thalidomide Phocomelia (seal limbs), cardiac malformations • Thiazide diuretics Neonatal thrombocytopenia • ACE inhibitors Renal tubular dysgenesis, lung hypoplasia, anuria, oligohydramnios • Misoprostol Mobius syndrome, arthrogryposis (prostaglandin) • Vitamin D William syndrome (infantile hypercalcemia, supravalvular aortic stenosis, elfin facies)

Drugs taken during perinatal/neonatal period and their effects on the neonate • Oxytocin (used for induction of labour) Hyperbilirubinemia in babies • Prolonged cortisone Adrenal crisis in • NSAIDS Premature closure of ductus arteriosus • Dexamethasone Periventricular leuocmalacia to baby • Chloramphenicol Grey baby syndrome • Erythromycin Pyloric stenosis • Vitamin K , hepatotoxicity Pediatrics in Last Minutes 15

Maternal Conditions And Fetal Outcome

Disease Outcome/Comment Bronchial asthma IUGR; Feal goiter and hypothyroidism (due to drugs- beta agonists) Chronic cardiac disease IUGR, abortion, asphyxia, prematurity Chronic Crenal disease IUGR, prematurity Hypertension Placental vasculopathy, IUGR Thyroid disorders Maternal hypothyroidism may cause congenital hypothyroidism (TSH does NOT cross placental, barrir but LATS does) SLE Congenital complete heart block, IUGR Smoking IUGR/LBW; Sudden death syndrome (SIDS); Increased ororfacial clefts in the fetus. Developmental lag for first few years of life: adverse effects on language skills and visual and spatial abilities.

NEONATOLOGY

Five Cleans of Intranatal Care • Clean hands • Clean delivery surface • Clean blade • Clean cord • Clean tie (for cord) AAP- AHA Guidelines for Neonatal Resuscitation • Initial steps DO NOT include giving supplemental oxygen. if persist despite giving free flow oxygen, give positive pressure ventilation • Routine intrapartum oropharyngeal and nasopharyngeal suctioning of babies born through meconium stained liquor no longer advisable

For Term Babies • Use of 100% oxygen is recommended when baby is cyanotic or when positive pressure ventilation is required during neonatal resuscitation • In situations where supplementary oxygen is not readily available, positive pressure ventilation should be started with room air. 16 Pre-NEET Pediatrics

For Preterm Babies • Begin PPV with oxygen concentration between room air and 100% oxygen • Increase oxygen concentration up or down to achieve saturation between 90 and 95% • If heart rate does not response by increasing rapidly to 100 per minute, correct any ventilation problem and use 100% oxygen

Regarding PPV LMA should NOT be used — In the setting of meconium stained amniotic fluid — When chest compression is required — In VLBW babies — For delivery of medications Naloxone NOT to be given by ET route; epinephrine preferably by intravenous route only

Capnography (exhaled CO2) recommended for confirming ET tube placement. • Normal APGAR sore = 8-10 at minute after birth; <7 indicates asphyxia. • NO NEED for resuscitation if these 5 criteria are fulfilled by newborn: — Full term, crying, clear of meconium, pink, good muscle tone. • 2 absolute indications for bag and mask ventilation are — Apnea at birth; HR < 100/minute. • Absolute contraindications for bag and mask ventilation are: — Diaphragmatic hernia, meconium aspiration. • Indications to start chest compression are: — HR < 60; ventilate with oxygen for a full 30 seconds initially and proceed with chest compressions if the HR remains <60/ mm. • Indications for endotracheal intubation are: — Prolonged PPV; Ineffective bag and mask ventilation; Diaphragmatic hernia, meconium aspiration. Pediatrics in Last Minutes 17

• Drugs which can be given endotracheally are: — Naloxone, Adrenaline, Lignocaine, Atropine (NALA).

DO NOT Resuscitate if

— When gestation birth weight or congenital anomalies are a/w almost certain death (class Ha) — Anencephaly — Chromosomal anomalies incompatible with life (e.g., trisomy 13) APGAR Score

012

Appearance (Colour) Blue or pale Body pink, Pink extremities blue Pulse (Heart rate) 0 <100 > 100 Grimace to catheter in nose No Response Grimace Cries, Coughs (Reflex stimulation) or Sneezes Activity (Muscle Tone) Flaccid Some Actively flexion moving extremities Respiratory effort (not rate) 0 Slow Good crying irregular

Timing of Selected Primitive Reflexes

Reflex Onset Fully developed at Persist till Palmar grasp 28 wk 32 wk 2-3 months (grasps objects in palm) Rooting (nipple seeking) 32 wk 36 wk Less prominent after 1 month Moro (extenson of 28-32 wk 37 wk 5-6 months limbs when startled) Tonic neck 35 wk 1 month 6-7 months Parachute 7-8 months 10-11 months Persists throughout life 18 Pre-NEET Pediatrics

Birth Trauma Cephalhematoma • Present at birth; • Seen at 2-4 days old; • May extend over suture lines; • Limited by suture lines; • Diffuse, ecchymotic, oedematous • Normal overlying skin; overlying skin; • Disappears spontaneously within • Reabsorbed over 2 weeks to 3 24hrs after birth; months; can calcify; • No treatment needed • May require phototherapy for jaundice 1. # Clavicle MC bone fractured due to ; appears at 1-20+ days 2. Sternomastoid tumor appears at 7-20+ days. Intrauterine posture is also a cause. Located at junction of upper and middle third of muscle. Disappears by 6 months of age. 3. Brain MC internal organ to be injured during birth; next MC is liver

Umbilical Cord • Usually cord is cut 2.5 inches or 6 cm from the umbilical base. • Contains 2 arteries and 1 vein. • — Is a/w genitourinary anomalies (MC), CVS anomalies, esophageal atresia, trache-esophageal fistula, imperforate anus — Incidence is 1%, also a/w DM, prematurity, asphyxia. • Cord clamping — Early cord clamping done in – Prematurity, Rh incompatibility, birth asphyxia, IUGR, baby of diabetic mother. — Delayed cord Clamping done in – Prematurity and cord around neck • Cord blood is useful for — Estimation of TSH, T3 and T4; in screening of congenital hypothyroidism (TSH levels > 50micrU/ml in cord blood are diagnostic of neonatal hypothyroidism). — Screening for inborn errors of metabolism/tandem mass spectroscopy (TMS); e.g., in PKU, , G6PD deficiency. — Sampling of and infant born to Rh positive mother: or study of blood group, Rh type, serum , Coomb’s test. Pediatrics in Last Minutes 19

Neonatal sepsis • Early onset occurs within 72 hours of life and is caused by organisms prevalent in the maternal genital tract or in the labour room. Group B Streptococci are the MC organisms (kelbsiella, E.coli and S.aureus are MC in India). • Late onset neonatal sepsis is a nosocomial infection acquired from the nursery or lying in ward and occurs  72 hours after birth. Gram negative bacilli are MC organisms. • Sepsis screen includes (usually  2 markers are considered significant) a. ANC<1000 b. Leucopenia < 5000/mm3 c. Band cell to neutrophil ratio or I:T ratio (immature to total polymorphs) > 0.2 (band cells > 20%) d. CRP> 8microgm/ml is positive e. Micro ESR> 15 mm/first hour f. Presence of > 5 PMN/hpf in gastric aspirate (limited utility in blood stained or meconium stained liquor) • Other markers - procalcitonin and • Blood culture is confirmatory • Listeria infection may produce preterm delivery, intrauterine death • Early conjugated hyperbilirubinemia with bilirubin > 0.5 mg/dl is noted in neonatal sepsis.

Hypoxic Ischemic (HIE)

• Initial response to hypoxia is increased cerebral blood flow due to redistribution of cardiac output by the ‘diving reflex’ and rise in BP. aggravates HIE. • Status marmoratus is seen in kernicterus and due to basal ganglia defect. • In preterm babies deeper vessels are deficient and hence they develop periventricular ischemia and leukomalacia which leads to spastic diplegia. Disability is more in LL • Sarnat and Sarnat staging is use for HIE - useful only when baby > 36 weeks. 20 Pre-NEET Pediatrics

• Preterm babies — Can tolerate hypoxia for longer periods without sequelae — Suffer more periventricular ischemia as cortical vessels are more superficial — Periventricular insults are more common than term babies • Term babies — Suffer more cortical ischemia and infarcts — It may lead to multifocal necrosis, porencephalic cyst, hydrencephaly ROP screening criteria • All infants born at d” 32 weeks. • All infants with birth weight d” 1500 grams • Very sick babies who require ventilatory support, multiple blood transfusions. Prematurity + oxygen toxicity predisposes to • Retinopathy of prematurity (ROP) • Bronchopulmonary dysplasia (BPD) Antenatal steroid therapy (ANS) • It is a/w reduced incidence of RDS/HMD, NEC Postnatal corticosteroids • Useful in prevention of HMD, BPD, but recent data are inconclusive and not recommended anymore. • They have some therapeutic utility in management of Excess administration of vitamin E is a/w • Intracranial hemorrhage • NEC Deficiency of Vitamin E is a/w • ROP • BPD • Hemolytic anemia Pediatrics in Last Minutes 21

Causes of Impaired Bilirubin Conjugation/ Unconjugated Hyperbilirubinemia

• Physiological jaundice of newborn - Decreased UGT activity • Breast milk jaundice - Inhibition of UGT activity bilirubin • Crigler Najjar syndrome - Genetic deficiency of bitirubin UGT activity Type I - Autosomal recessive /Absent UGT activity Type II - Autosomal dominant! Decreased UGT activity • Gilbert syndrome • Diffuse hepatocellular disease - Decreased UGT activity due to mixed etiologies Two other syndromes are often asked. These are associated with Conjugated hyperbilirubinemia. • Dubin Johnson syndrome Q Impaired biliary excretion of bilirubin (Autosomal Recessive) glucuronides due to Canalicular membrane carrier defect • Rotor’s syndromeQ Decreased hepatic uptake and (Autosomal Recessive) storage, or possibly decreased biliary excretion?

Cause of Late Onset Jaundice After 72 Hours of Age and Within First 2 weeks

• Breast milk jaundice • Biliary atresia • Hypothyroidism • Gilbert’s syndrome • Infection: UTI, Herpes, Hepatitis • Parenteral alimentation in VLBW • Metabolic diseases like galactosemia, alpha 1 antitrypsin deficiency, cystic Fibrosis, hereditary fructose intolerance tyrosenemia 22 Pre-NEET Pediatrics

Guidelines for Phototherapy and in a term Neonate Age Total serum bilirubin (mg/dl) Phototherapy Exchange transfusion 24-48hrs  15  20 48-72hrs  18  25  72hrs  20  25 Guidelines for Management of Jaundice in Preterm babies

Total serum bilirubin (TSB) (mg/dL)

Gestation and birth Healthy baby Sick baby weight Preterm babies Phototherapy Exchange Phototherapy Exchange transfusion transfusion

<1000 5-7 11-13 4-6 10-12 1001-1500g 7-10 13-15 6-8 11-13 1501-2000g 10-12 15-18 8-10 13-15 2001-2500g 12-15 18-20 10-12 15-18 Term babies >2500g 15-18 20-25 12-15 18-20 Causes of Immune

• Rh incompatability Non-immune • Anemia -, G-6PD deficiency • Cardiac dysarrhythmias  Supraventricular tachycardia, AF, congenital heart block • Structual cardiac defects  Tricuspid insufficiency, endocardial cushion defect, cardiomyopathy, hypoplastic left heart, premature closure of foramen ovale • Vascular  Chorioangioma of placenta, Twin-Twin transfusion, umblical artery aneurysm, thrombosis of renal or umbilical vein or IVC • Lymphatic  Lymphangiectasia, cystic hygroma, Noonan syndrome • CNS  Encephalocele, intracranial hemorrhage • Thoracic  Mediastinal teratoma, diaphragmatic hernia

Contd... Pediatrics in Last Minutes 23

Contd...

• Teratomas  Choriocarcinoma, sacrococcygeal teratoma • Tumor & storage diseases  Neuroblastoma, hepatoblastoma, Gaucher disease, Niemann-Pick disease, Mucopolysaccharidosis • Chromosomal  Trisomy 13, 15, 16, 18, 21 • Bone diseases  Osteogenesis imperfecta, skeletal dysplasias • Congenital infections  CMV, rubella, Toxoplasmosis, , Parvovirus, Leptospirosis, disease • Others  Congenital nephrosis, Myotonic dystrophy, Infant of diabetic mother, Maternal therapy with indomethacin, Hepatic fibrosis Erythroblastosis Fetalis • Haemolytic disease of the newborn (erythroblastosis fetalis) is caused by blood group incompatibility between the mother and fetus. • Maternal IgG crosses the placenta and destroys fetal erythrocytes. • An Rh - mother carrying an Rh + fetus is at highest risk • Risk increases when fetal blood crosses into the maternal circulation as in abortion, ectopic pregnancy, amniocentesis and motor vehicle accidents. • Affected neonates present with: anemia, hvperbilirubinemia hepatosplenomegaly, pulmonary edema and ascites. • The direct Coombs’ test is positive. • Treat with exchange transfusions (see below). • Prevent disease with anti-Rh IgG injections for high-risk mothers at 28 weeks of delivery, and at any other time of exposure to fetal blood.

Indications for exchange blood transfusion in infants with Rh- hemolytic disease of the newborn

• Cord Hb of 10g/dl or less • Cord bilirubin of 5 mg/dl or more • Unconjugated serum bilirubin of 10 mg/dl within 24 hours or 15 mg/dl within 48 hours or rate of rise of >0.5 mg/dl per hour 24 Pre-NEET Pediatrics

INFANT OF DIABETIC MOTHER

Congenital Defects • Cardiac (VSD, ASD, TGA Coarctation of aorta) • Neural tube defect • Holoprosencephaly • Sacral agenesis (most specific) • Hydronephrosis • Renal agenesis • Duodenal atresia • Anorectal malformations

General

• Macrosomia • Normal head size • Increased subcutaneous fat • Birth trauma • Hairy pinna

Other

• Renal vein thrombosis • Respiratory distress syndrome • • Small (lazy) left colon syn.

Cardiovascular • Cardiomegaly • Transient hypertonic cardiomyopathy • Persistent fetal circulation Metabolic • Hypoglycemia • Hypocalcemia • Hypomagnesemia • Hyperbilirubinemia Pediatrics in Last Minutes 25

D/D of Neonatal Vomiting

First 1 to 3 days End of first week Very common Common — Feeding problem — Hiatus hernia — Gastric irritation, swallowed blood. — Infection — Functional in premature, Less common stressed. — Infection — Necrotizing enterocolitis — Neurological: asphyxia/birth injury, — Obstructive: pyloric stenosis, intraventricular haemorrhage. volvulus, anal stenosis, small left colon, Hirschsprung’s disease. Uncommon Uncommon Obstruction: Duodenal atresia, Renal failure, CAH, galactosemia, stricture or web, annular pancreas, organic acidosis, lactic acidosis. Ladd’s bands, mid gut volvulus Meconium ileus, Meconium plug, Hirchspring’s disease, Anal atresia. Neonatal Hypoglycemia The whole blood glucose level of < 40 mg/dL indicates hypoglycemia 1. Transient hypoglycemia • Prematurity; small for date infants; infant of diabetic mother; smaller of twins. 2. Persistent hypoglycemia • Hyper-insulinemia, Nesidioblastosis, Adenoma of beta cells, Beckwith syndrome, leucine sensitivity. • Deficiency of hormones such as glucagons, growth hormone, epinephrine, ACTH. • Deficiency of substrate as in ketotic hypoglycemia and Maple syrup urine disease. • Disorders of carbohydrate metabolism such as glycogen storage disease, galactosemia, and fructose intolerance. 3. Other etiologies • Idiopathic; Sepsis; Drugs (maternal tolbutamide); liver disease (), carcinoma etc. 26 Pre-NEET Pediatrics

Age of Onset of

First day Between 1-3 days 4th to 7th day

Hypoxic-ischaemic Intracranial haemorrhage Tetany encephalopathy Birth injury (cerebral contusion) Hypoglycemia ‘First day’ hypocalcemia Narcotic withdrawal TORCH infections (normal phosphate) Pyridoxine dependency Inborn errors of metabolism Developmental Accidental injection of Kernicterus local anesthetic Idiopathic Head Malformations

1. Anencephaly: Due to failure of closure of the rostral neuropore 2. Holoprosencephaly: Incomplete separation of the cerebral hemispheres. Seen in Patau’s syndrome. 3. Porencephaly Cysts or cavities in the brain may result from developmental defect or acquired lesions including infarction of tissue 4. Lissencephaly: Bat like brain with no cerebral convolutions and a poorly formed sylvian fissure due to faulty neuroblast migration (agyria). Hypoplasia of optic nerves and microphthalmia are common. 5. Schizencephaly: Unilateral or bilateral cleft in the cerebral hemispheres, microgyria. 6. Scaphocephaly: MC type of cranoisynostosis. 7. Encephalocele: Is a malformed diverticulum of CNS tissue extending through a defect in the cranium. 8. Shapiro’s syndrome: Agenesis of corpus callosum Torch Infections Infection Description Treatment Prevention

Toxoplasmosis 3Cs– Convulsions, Pyrimethamine, Avoid exposure to intracranial Calcification, sulfadiazine, cats and cat feces Chorioretinal scar, spiramycin during pregnancy; hydrocephalus Ring avoid raw enhancing lesions on undercooked meat head CT Rubella Deafness (MC) None Immunize mother ‘Bluberry muffin’ rash, prior to pregnancy

Contd... Pediatrics in Last Minutes 27

Contd... Infection Description Treatment Prevention

Cataracts and ‘salt and pepper’ chorioretinitis PDA and multiple Pulmonary stenoses encephalitis (periventricular calcification) maximum fetal transmission if iinfected between 6-8 weeks of pregnancy Cytomegalovirus Petechial rash, Ganciclovir Avoid exposure periventricular calcifications, Acyclovir Perform a microcephaly, chorioretinitis Caesarean section Herpes Skin, eye and mouth vesicles, if mother has active can progress to severe lesions at time of systemic/CNS infectioin delivery Syphilis Maculopaular skin rash, Penicillin Treat seropositive lymphadenopathy, ‘snuffles’ mothers with peri-osteitis Interstitial penicillin keratitis If infected during first trimester (maximum risk of abortion)

Management of Neonate Born to HBSAg+ve Mother

• Risk of fetal transmission after maternal exposure to HBV – Depends upon the time of exposure and status of maternal serological markers. – Perinatal transmission occurs MC during passage through the birth canal from infected blood and fluids at the time of delivery. – HBV is usually transmitted at the time of birth (rarely in utero) and commonly results in carrier state (increased risk of cirrhosis and HCC later in life) or it may cause severe hepatitis in infancy. • All neonates born to HBSAg +ve mother should be given HBIG (hepatitis B Ig) 0.5 ml IM + active immunization with HB preferably within 12 hours of delivery.

Perinatal Tuberculosis

• True congenital TB is rare. Congenital TB in a neonate is acquired 28 Pre-NEET Pediatrics

by transplacental transmission through a lesion in the placenta or ingestion of infected liquor. • Transplacental transfer MC affects the liver; hepatomegaly is usual manifestation. • Postnatal TB is more common, usually from an open infectious case, MC the mother. • Management • If the child has clinical features of perinatal TB – Usual treatment Give regimen of 2HRZ +7 HR • If a high risk mother delivers a baby who is asymptomatic – Usual treatment - continue breast feeding; gve BCG to child + ATT to mother and screen the baby (if mother is Mantoux positive, CXR and screening +ve or open case of TB) – If screening of baby normal - give prophylactic ATT to baby (6 HR) – If screening of baby abnormal - treat as perinatal TB

Advanced maternal Age Advanced Paternal Age (>35 years) is a/w (>50 years) is a/w

– Down syndrome - Marfans syndrome – Mental subnromality - Achondroplasia – Premature labour - Craniosynostosis with syndactyly (Aperts syndrome) – Edward syndrome - Downs syndrome (with fusion of chromosome 21,22) – Dizygotic twins - Osteogenesis imperfecta - Congenital deafness

HIGH YIELDING FACTS

Congenital Diaphragmatic Hernia • Bochdalek’s Hernia (posterolateral hernia): This is really the persistence of the pleuroperitoneal canal. MC diaphragmatic hernia in children, more common on left side. Classic triad of respiratory distress, apparent dextrocardia and a scaphoid abdomen. Pulmonary hypoplasia is the MC cause of morbidity and mortality. Pediatrics in Last Minutes 29

• Morgagni Hernia (retrosternal anterior hernia): Defect is between the sternal and costal attachments of the diaphragm. Usually occurs on right side, in adults, particularly females. MC involved viscus is the transverse colon.

Ponderal Index (PI) In Neonate • Ponderal index has been used as an indicator of fetal growth status, especially to assess assymetrical IUGR. • The Ponderal index is calculated by multiplying the weight in grams by 100 and then dividing it by cube of length in cm.

Ponderal Index = Weight (gm)/Length (cm)3 x100 • A ponderal index below the 10th percentile may be used to identify IUGR infants correctly. • A low neonatal ponderal index is defined as less than 1 SD below a mean 2.0. • PI is usually less than 2 in assymmetric growth retarded baby and 2 or more in a baby who has either normal growth or has symmetrical growth retardation. • Fetal ponderal index can also be calculated by USG examination and compared with neonatal P1. Fetal P1 had been found to be predictor of IUGR with the sensitivity and specificity of 76.9 and 82%, respectively. If the fetal P1 is less than 1 SD, the fetal and neonatal well being is compromised.

Apt Test • The Apt test is most commonly used in cases of vaginal bleeding late during pregnancy (antepartum hemorrhage) to determine if the bleeding is from the mother or the fetus. • Exposing the blood to NaOH (alkali) will denature the adult but not the fetal hemoglobin. The fetal hemoglobin will appear pinkish color under the microscope while the adult hemoglobin will appear as a yellow - brownish color. - Positive test à Blood is of fetal origin. - Negative test à Blood is of maternal origin. • The Apt test can be used after birth (post partum hemorrhage) if the newborn has bloody vomiting, bloody stool, or active bleeding from nasogastric tube. A positive Apt test would mean that the blood is either due to GI or pulmonary bleeding from neonate. A negative Apt test would indicate that the blood is of maternal 30 Pre-NEET Pediatrics

origin, suggesting that the neonate swallowed or aspirated maternal blood, either during delivery or during breast feeding (from breast fissures). Kleihauer Betke test (KB test) • The KB test is the standard test for detecting fetal - maternal hemorrhage. It is used to measure the amount of fetal hemoglobin transferred from a fetus to a mother’s blood stream. • It is usually performed on Rh(-ve) mothers to determine the required dose of Rho (D) immunoglobulin to inhibit formation of Rh antibodies in the mother and prevent Rh hemolytic disease in future Rh - positive children. • A standard blood smear is prepared from the mother’s blood and exposed to an acid bath. This removes adult hemoglobin, but not fetal hemoglobin, from the red blood cells. Subsequent staining makes fetal cells (containing HbF) appear rose pink, while adult red cells are only seen as ghosts. • KB test quantifies the fetal - maternal hemorrhage.

Apt test Kleihauer Betke test

Source of sample Maternal or neonatal Maternal How it works Adding 1% NaOH Adding acid destroys adult destroys adult HbA but HbA but not fetal HbF not fetal HbF Assessment type Qualitative Quantitative Results Positive means blood Reported in estimated ‘ml’s of is of fetal origin fetal blood

Hematopoesis in Fetus • The anatomic sites of hematopoesis undergo developmental changes during embryonic and fetal life. • RBC formation can be observed within the developing blood vessels of the yolk sac at 2 weeeks of gestation. • By 8 weeks of gestation the site of RBC formation begins to shift to the sinusoids of the liver, where granulocyte precursors and megakaryocytes are also seen. Hematopoesis in the liver is maximal till 20-24 weeks and declines thereafter. • Hematopoesis in the bone marrow is evident by the 4th month of gestation and this becomes the predominant site of hematopoesis during the rest of gestation and later. Pediatrics in Last Minutes 31

Hematopoiesis After Birth • Upto puberty: It occurs throughout the seletaon. • After puberty: Only the vertebrae, ribs, sternum, , pelvis and proimal epiphyseal regions of humerus and femur retain red marrow Circulatory Adjustments at Birth

These are brought about because of a shift from placental dependance for gas exchange in the fetus to pulmonary gas exchange in neonate. 1. Pulmonary circulation: Immediatly after birth lungs expand due to first few breaths of neonate. – This causes a fall in pulmonary vascular resistance (oxygen causes pulmonary vasodilatation). – This results in increased flow into pulmonary trunk and arteries. – The pulmonary artery pressure falls due to lowering of pulmonary vascular resistance. – The pressure relations between aorta and pulmonary trunk are reversed so that the blood flow through the ductus arteriosus is reversed  Instead of blood flowing from the pulmonary artery to aorta, the direction of flow through ductus, is from aorta to pulmonary trunk. – Increasing oxygen saturation causes the muscle of ductus to constrict  In full term neonates the ductus arteriosus closes within 10 to 25 days. 2. System circulation and circulation through heart – Loss of placental circulation and clamping of the cord after birth results in increase in systemic vascular resistance. – This tends to increase the aortic blood pressure and the left ventricular systolic pressure. The loss of placental circulation results in sudden reduction of flow through ductus venosus which closes off  Flow through ductus venosus disappears by the 7th day of posnatal life. – The loss of placental flow results in a decrease in the volume of blood returning to right atrium  Right atrial pressure decreases. – The left atrial pressure becomes higher than right atrial pressure and the septum primum which acts as a valve of fossa ovalis, approximates with the septum secundum to close off formen ovale. 32 Pre-NEET Pediatrics

 Functional closure of foramen ovale occurs very quickly. – Over a period of months, the septum primum and septum secundum become firmly adherent resulting in anatomical closure of the foramen ovale. – After closure of ductus arteriosus, there is establishment of postnatal circulation: – The blood reaching the right atrium through IVC and SVC is emptied into the right ventricle from where it is pumped into pulmonary trunk. – After coursing through lungs for gas exchange, it reaches the left atrium and ventricle. – The left venthcle pumps it out for distribution in the body for oxygenation of the tissues. – The venous return again comes back to right atrium through IVC and SVC. – All of the blood leaving the right ventricle, after coursing through lungs, reaches the left ventricle  The two ventricles are connected in series and therefore, the output of right and left ventricles are same (in contrast to fetal circulation, where right ventricular output is more).

Congenital Heart Disease

Left to right shunts (Acyanotic) Right to left shunts (cyanotic)

• VSD (MC congenital heart • Transposition of great arteries (MC disease) cyanotic lesioin) • ASD • PDA • VSD> ASD > PDA • Tetralogy of Fallot • Truncus arteriosus • Tricuspid atresia • TAPVR S-I-M 5Ts Pediatrics in Last Minutes 33

ACYNOTIC CONGENITAL HEART DISEASE - LEFT TO RIGHT SHUNTS

Disease Anatomy/ Symptoms Signs

Ventricular - 90% in membranous - Widely split, variable S2. septal defect part of septum; 70-80% - Pansystolic murmur (MC Cong have spontaneous at left sternal border. HD) closure - Small VSD - Loud - In infants FTT, CCF. murmur (Maladie de - MC cong. Heart Roger) disease complicated - CXR: biventricular by IE hypertrophy, pulmonary plethora Patent ductus - In fetus, shunt is right to left, - Continuous machinery arteriosus (normal), through the ductus murmur in left arteriosis which connects infraclavicular area pulmonary artery and aorta. (Gibson's murmur) In first few days of neonatal - Mid diastolic flow period, ductus arteriosus murmur at mitral valve should close, if not lung - Differential cyanosis resistance decrease and occurs in PDA with shunt becomes left to right reversal of shunt with subsequent RVH and failure (abnormal). - Indomethacin is used to - Risk factors: Maternal rubella close PDA. S-I-M in 1st trimester, high altitudes. "Come IN and close the door" - MC in Prematures (recurrent - PGE (alprosatdil) is used apnea, RDS, CCF); in infants to keep PDA open (FIT in additioin); in adults which is necessary (dyspnea, SBE). to sustain life in conditions like TGV. S-I- M "Keep OP(G)En with PGE". ASD - - Ostium secundum (MC, - Left parasternal heave situated in fossa ovalis) (RVH) - Ostium primum (AV septal - Widely fixed split S2 defect, cleft anterior leaflet - Ejection systolic murmur of mitral valve) over pulmonary area - Child: asymptomatic, usually - Mid diastolic tricuspid - Adult breathless from flow murmur pulmonary HTN 34 Pre-NEET Pediatrics

ACYANOTIC CONGENITAL HEART DISEASE - OBSTRUCTIVE

Disease Anatomy/Symptoms Signs

Aortic stenosis (AS) – Infant: valvular (MC) – Thrill over - CCF if severe suprasternalnotch/ stenosis carotids, ejection systolic murmur – Supravalvular: maximal in 2nd asymptomatic, right I/C space a/w Williams syn. with neck radiation Coarctation of aorta – Infantile aortic – Pulse: radiofemoral (COA) stenosis proximal to delay insertion of ductus – Elevated upper limb (preductal) BP – Adult: postductal, – Palpable arterial MC coarctation pulsations in is just below interscapular region origin of left (Suzzman's sign) subclavian A. – A/w HTN in upper – Aortic ejection systolic extremities, weak murmur pulses in lower – CXR: 3 sign, Dock sign extremities (rib notching due to – A/w Turner syn. collateral circulation)

Cyanotic Congenital Heart Disease

Disease Anatomy/Symptoms Signs

Transposition of - Cyanosis from birth - Cyanosis persists in 100% great arteries or shortly after, oxygen which may even (TGA) proportional to worsen cyanosis by shunt through causing closure of ductus. foramen ovale, ductus arteriosus or VSD. - Single Heart sound, murmur often absent - XCR: Egg on side/string appearance Tetralogy of - Infant: progressively - Central cyanosis, clubbing, Fallot deeper cyanosis, right ventricular heave. (Pulmonary weeks or few Single S2 stenosis, RVH, months old. Ejection systolic murmur Pediatrics in Last Minutes 35

Contd...

Disease Anatomy/Symptoms Signs Overriding of Cyanotic spells at third left interspace. aorta, VSD) from infundibular spasm. - Childhood: squatting - CXR: cour en sabot (boot after exertion to shaped heart due to RVH). overcome Oligemia dyspnea, SBE, cerebral abscesses, polycythemia. CCF is very RARE. - Squatting cause compression of femoral arteries increase pressure threby decrease the right to left shunt and directing more blood from the RV to the lungs

Eisenmenger's - Uncorrected VSD, - Loud P2 syndrome ASD, PDA  increase pulmonary vascular resistance due to arteriolar thickening  progressive pulmonary hypertension. - As pulmonary resistance increase the shunt reverses from L  R to R  L shunt which causes late cyanosis (clubbing and polycythemia) 36 Pre-NEET Pediatrics

COMMON MIXING CONGENITAL HEART DISEASE: ACYANOTIC AND CYANOTIC

Disease Anatomy/Symptoms Signs

Truncus - Neonate: breathless, CCF - Minimal cyanosis, arteriosus Bounding pulse, Single S2, short systolic murmur Total anom - Newborn: obstructed - Cyanosis NOT improved alous venous return in 100% oxygen. Poor pulmonary simulates pulse. Loud S2. Murmur venous RDS. Onset 2 often absent, tachypnea, connection months to 2 right ventricular heave. (TAPC) years. Breathless, - CXR: pulmonary edema FTT with Snowman heart (figure of 8 heart of 'cottage leaf') Key: • CCF = congestive cardiac failure; • CHD = Conegnital heart disease; • FTI' = failure to thrive; • TE = Infective endocarditis; • LVH = left ventricular hypertrophy; • PS = Pulmonary stenosis; • RDS = respiratory distress syndrome; • RVH = right ventricular hypertrophy; • SBE = subacute bacterial endocarditis.

ADDITIONAL POINTS ABOUT CONGENITAL HEART DISEASE • Holt-Oram syndrome—ASD with first-degree heart block; hypoplastic thumb or absent radii. • Critical aortic stenosis—severe aortic stenosis that presents in early infancy is termed critical aortic stenosis and is associated with left ventricular failure and signs of low cardiac output. • Endocardial fibroelastosis—In infants with critical aortic stenosis, the left ventricular shortening fraction is usually decreased and the endocardium may be bright on ECHO, indicating the development of endocardial fibrous scarring known as endocardial fibroelastosis. Pediatrics in Last Minutes 37

• Shone complex - Coarctation of aorta + mitral valve abnormalities (supravalvular mitral ring or parachute mitral valve) + subaortic stenosis — this group of left sided obstructive lesions occurring together is called Shone complex. • Blalock-Taussig shunt is the MC aortopulmonary shunt procedure performed for TOF. • Norwood procedure - is performed for hypoplas tic left heart syndrome. • Heath Edwards classification is a grading for the histopathologic changes occurring in small pulmonary arterioles and muscular arteries in Eisenmenger's syndrome.

Congenital Cardiac Disease Associations Disorder Defect • Rubella PDA • Turner's syndrome COA • Infant of diabetic mother TGV • Wiliam's syn. Supravalvular AS • Marfan's syn. Aortic insufficiency • Down's syn. Ostium primum ASD, VSD, AV septal defect (endocardial cushion defect) • 22q11 syn. Truncus arteriosis, TOF

Primary Pulmonary Hypertension • Characterized by pulmonary vascular obstructive disease and right- sided heart failure. • RVH Dilated pulmonary artery  Pulmonary insufficiency  tricuspid insufficiency (late stages). • Predominant symptoms include exercise intolerance and easy fatigability. Ejection click after S1 is present and S2 is closely split. • Chest X-ray — prominent pulmonary artery and right ventricle. ECG — spiked P waves. • Treatment— – blockers (nifedipine); – Intravenous (epoprostinol, treprostinil) or nebulised (iloprost) prostacyclin (PGI2.); – Oral endothelin antagonist (Bosentan), – Oral PDE-5 inhibitor (sildenafil, tadalafil). – Definitive therapy for non-responders lung transplantation. 38 Pre-NEET Pediatrics

NADAs Criteria The Assessment of a child for the presence or absence of heart disease can be done with the help of some guidelines suggested by NADA. These guidelines are called NADA 's criteria.

Major Minor Systolic murmur, Systolic murmur < grade III grade III or more, Abnormal 2 heart sound especially with thrill Abnormal ECG Diastolic murmur Abnormal X Ray Cyanosis Abnormal BP Congestive cardiac failure Presence of one major or two minor criteria are essential for indicating the presence of heart disease.

Criteria for Diagnosis of Rheumatic - Presence of 1 major and 2 minor criteria in the presence of essential criteria is necessary for diagnosis of rheumatic fever.

Major Criteria Minor Criteria 1. Carditis A. Clinical 2. Arthritis i. Fever 3. Subcutaneous nodules ii. Arthralgia 4. iii. Previous rheumatic fever 5. Erythema marginatum or rheumatic heart disease

B. Laboratory Essential criteria i. Acute phase reactants; Evidence of recent leukocytosis, elevated streptococcal sedimentation rate and C infection as indicated reactive protein by ii. Prolonged PR interval in the electrocardiogram a. Increased antistreptolysin 'O' titer b. Positive throat culture c. Recent scarlet fever Pediatrics in Last Minutes 39

Surgical procedure for selected CHD lesions (palliative procedures)

Procedure Anatomy involved Result Indicated for

Blalock- Subclavian artery Increased Tetralogy of Fallot Taussig to ipsilateral pulmonary Pulmonary valve atresia shunt pulmonary artery blood flow Tetralogy of Fallot Waterston Aorta to right Increased pulmonary shunt pulmonary artery pulmonary valve atresia blood flow Tricuspid atresia Rashkind Balloon atria Increased Transposition of procedure septostomy atrial great arteries mixing Tricuspid atresia Blalock- Operative Increased Transposition of Hanlon atrial atrial great arteries procedure septostomy mixing Balloon Valves and Dilation Pulmonary valve angioplasty vessels of valves/ stenosis, Aortic vessels valve stenosis Pulmonary Pulmonary Decreased Endocardial artery banding artery pulmonary cushing blood flow defect, Single ventricle Corrective procedures Fontan Right atrium Atrium Tricuspid atresia single procedure to pulmonary functions ventricle, pulmonary artery as right atresia anastomosis ventricle Mustard Intera-atrial RV remains Transposition of great procedure baffle systemic vessels arteries ventricle Norwood A complex two Hypoplastic left heart procedure stage procedure

Characteristics of murmurs in selected heart lesions Lesion Loudest Quality Radiation Aortic Right 3rd ICS Harsh, SEM Carotids stenosis Aortic MLSB Blowing, Early Apex regurgitation diastolic Pulmonary Left 2nd ICS stenosis Harsh, SEM Lungfields Pulmonary Left 2nd ICS Low pitched, Little to none Contd... 40 Pre-NEET Pediatrics

Contd...

Procedure Anatomy involved Result Indicated for

regurgitation early-mid diast Mitral stenosis Apex Low pitched, None raid diast. Rumble Mitral Apex Blowing, Left axilla regurgitation holosystolic Lung fields ASD Left 2nd ICS Harsh, SEM VSD LLSB Harsh, None Holosystolic PDA Left 2nd - 3rd ICS Continuous Lung fields

Single Gene Defects - Account for 3% of CHD • Autosomal dominant: – Marfan's - aortic and mitral valve incompetence, dilatation of the ascending aorta – Holt - Oram - VSD, ASD – Noonan's - PSASD cardiomyopathy • Autosomal recessive: – Pompe's (Type II A glycogen storage disease) - cardiomyopathy – Ellis - VanCreveld - AVSD, common atrium • X-linked: – Duchenne Muscular Dystrophy - cardiomyopathy • Polygenic inheritance: – Well described for PDA with a recurrence risk of 2.5 inn siblings (recurrence risk increased to 10% if > than one family member affected).

Respiratory Infections

Croup (laryngo Acute epiglotitis tracheobronchitis) (emergency) Etiology MC parainfluenza virus MC by H. inflenzae type b. Presenting 1; also by RSV age 3 months to 3 years 3-7 years Clinically Prodrome with URI Rapid onset (4-12 hours); high symptoms for fever, dysphagia, drooling, 1-7 days; low muffled voice, stridor patients grade fever, ins may be in "sniffing" position

Contd... Pediatrics in Last Minutes 41

Contd...

Croup (laryngo Acute epiglotitis tracheobronchitis) (emergency)

piratory stridor that with neck hyperextended and worsens with agitation; chin protruding hoarseness of voice, barking cough, X-ray "Steeple sign" on lateral "Thumb sign" on lateral neck neck film film Treatment Mist therapy, oxygen, Call anesthesia, transfer to aerosolized racemic operation theatre, epinephrine, endotracheal ntubation and IV dexamethasone antibiotics

Causes Of Increased Sweat Sodium of 70 MmoI/L or more

• Cystic fibrosis • HIV infection • Metabolic: Glucose-6-phosphate deficiency, Adrenal insufficiency, Nephrogenic Diabetes Insipidus, Mucopolysaccharidoses, Fucidosis • Insufficient Sweat: Hypothyroidism, Ectodermal dysplasia, Riley Day syndrome, Malnutrition • False negative sweat test: Edema, Hypoproteinemia

Pediatric Gastrointestinal Conditions • Intussusception – Telescoping of a bowel segment into itself may - edema, arterial occlusion, gut necrosis and death. – Affects children 4 mo -2 years; – MC cause of bowel obstruction in first 2 years of life; usually ileocecal; – Presents as paroxysmal abdominal pain, 'currant jelly' stools (blood and mucus), bilious vomiting; palpable sausage shaped mass; sign de Dance; – Barium enema is both diagnostic and theraputic; – a/w HSP and cystic fibrosis 42 Pre-NEET Pediatrics

• Volvulus – Incomplete fixation to the posterior abdominal wall, causing a malrotated gut to twist on itself. – Affects children 0-2 years; – Sudden onset of pain, distension, peritonitis, "bird's beak" on X- ray, – Treat with surgery immediately since gut may necrose due to superior mesenteric artery occlusion. • Necrotising enterocolitis – Intestinal necrosis occurring primarily in watershed distributions, – Affects children 0-2 months; – Prematurity and congenital heart disease are risk factors; – Fever, rectal bleeding, air in bowel wail (pneumatosis intestinalis) and/or hepatobiliary/ portal air; – Treat with NPO, IV fluids and antibiotics. • Meckel's diverticulum – A remnant of omphalomesenteric duct that persists as an outpouching of the distal ileum; can contain ectopic gastric mucosa – Affects children 0-2 years; – Rule of 2s for Meckel's diverticulum =2% of population affected (MC GI tract abnormality; remnant of omphalomesenteric duct), 2 inches long, within 2 feet of ileocolic junction, presents in the first 2 years of life. – Meckel's diverticulum can cause intussusception, obstruction or volvulus; – Use Meckel scan (technetium radionucide scan) to detect gastric mucosa; – Treat with surgery • Meconium ileus – In cystic fibrosis, meconium plug obstructs intestine preventing stool passage. – Affects children 0-2 weeks; may cause late feculent vomiting, rectal prolapse Pediatrics in Last Minutes 43

Congenital Gastrointestinal Malformations in Children • Pyloric stenosis – Hypertrophy of pylorus - gastric outlet obstruction; – affects first-born male infants at 2 weeks of life (range 2 wk- 4 mo); – nonbilious projectile vomiting palpable olive shaped mass in epigastrium; hypochloremic, hypokalemic metabolic alkalosis secondary to emesis; – U/S is gold standard; – treat by first correcting dehydration and metabolic abnormalities, surgery is pyloromyotomy (Ramstedt's). • Duodenal – Affects infants 0-1 week; atresia – early bilious vomiting with proximal stomach distension ("Double bubble"); – a/w Down's syndrome; – treat with duodenoduodenostomy • Tracheoesopha – Blind esophageal pouch; geal fistula – fistula between distal esophagus and trachea MC type; – presents in the first few hours of life with copious secretions, choking, cyanosis, respiratory distress; ; gastric distension from air occurs • Hirschsprung's – Absence of ganglion cells/enteric nerve disease plexuses (Auerbach's and Meissner's) in colon on rectal bisopsy; – Narrowing of anganglionic segment with dilation of proximal normal colon; can be a short (75%) or long segment; – Presents at infancy or within first 2 years of life; – Failure to pass meconium, abdominal distension, chronic constipation; – Staged procedure with initial diverting colostomy and later resection when infant> 6 months old 44 Pre-NEET Pediatrics

Common pediatric renal diseases Nephrotic Syndrome • MC cause in children is minimal change disease (normal histology on light microscopy). • Clinically: – Massive edema – Massive selective proteinuria (>2 g/day) – Hypoalbuminemia (<2.5 g/dl) – Hypercholesterolemia (>200 mg%) • Gene mutation a/w congenital NS (Finnish) type = Nephrin (NPHS1). S-I-M PHinnish = NePHrin. • Gene mutation a/w steroid resistant NS = Podocin (NPHS2) • Treatment Initial longer course of corticosteroids; relapses are common. Pediatrics in Last Minutes 45

Also Know

Infrequent relapses: If a patient gets 3 or less relapses in a year Frequent relapses: If a patient has 4 or more relapses in a year Steroid dependent: When relapse occurs within 2 weeks of discontinuation of drugs

Acute Post Streptococcal Glomerulonephritis

• Following h/o recent streptococcal pyoderma or pharyngitis. • Edema, Oliguria, hematuria, hypertension, decreased C3 levels • Fluid restriction, furesemide, antihypertensives • > 95% have good prognosis. • Common Pediatric Hematologic Disorders

HUS HSP TTP ITP

MC age Children Children Young Children or adults adults Previous infection Diarrhea URI None Viral (esp. In (EHEC, children) Shigella) RBC count  N  N Platelet count  N  Peripheral smear Hemolysis N Hemolysis N Kidney effects ARF, hematuria Hematuria ARF, None proteinuria Treatment Supportive* Supportive Plasmap- Steroids***, heresis, splenectomy if NSAIDs; drugs fail No platelets** Key differential Age, Rash, CNS Antiplatelet points diarrhea abdominal changes, antibodies pain, age arthritis, melema • Key HUS = Haemolytic uremic syndrome; HSP = Henoch-Scholnlein purpura; Ti? = Thrombotic thrombocytopenic purpura; ITP = Idiopathic thrombocytopenic purpura; ARF = Acute renal failure; 46 Pre-NEET Pediatrics

EHEC = Enterohemorrhagic E.coli. * In HUS and HSP, patients may need dialysis and transfusions. ** Do not give platelet transfusion to patients with TTP, clot may form. *** Give steroids only if the patient is bleeding.

Grading of Vesico-Ureteral Reflux (VUR) Grade 1 Reflux up normal caliber ureter without pelvicalyceal filling Grade 2 Reflux up normal caliber ureter with pelvicalyceal filling Grade 3 Reflux up dilated ureter into the dilated pelvicalyceal system Grade 4 Reflux up markedly dilated ureter and collecting system • Micturation Cystourethrogram (MCUG) best investigation for VUR.

Febrile Convulsions

Simple benign febrile Atypical complex convulsions febrile convulsions • Fits occur within • Anything different from features 24 hours of onset offever of simple febrile convulsions are • Last < 10 minutes and atypical fits. Presence of family are usually single per h/o , neurodevelopmental febrile episode retardation and atypical episodes • Convulsions are increase risk of febrile episodes and generalized subsequent epilepsy. • No post-i ctal neurological deficit • Family history may be present.

Treatment Prophylaxis • (, • Intermitent , clabazam, AVOID aspirin - most desirable. due to risk of Reye's syndrome) • Hydrotherapy, tepid sponging • Continuous prophyl axis - in form of oxygen. antiepileptic drug therapy is advocated • IV diazepam in event of failure of intermittent or phenobarbitone for therapy, especially in recurrent control of , atypical seizures or family h/o epilepsy ( or phenobarbitone). Pediatrics in Last Minutes 47

Common Pediatric Epilepsy Syndromes

Syndrome Symptoms Diagnosis Treatment

Absence Multiple brief Generalized Ethosuximide seizures staring episodes 3-Hz spike and wave pattern on EEG Infantile Affects infants Hypsarrythm- ACTH spasms <1 year, ia on (West 'jackknife EEG. syndrome) spasms', A/w tuberous developmental sclerosis regression Lennox First Atypical spike No effective Gastaut between 1-7 and wave treatment syndrome years of age; pattern in multiple frontal region progressive on EEG difficult to treat type seizures with GTCS and drop attacks Juvenile Affects healthy Positivefamily Easily treated mycolonic adolescents; history with a variety of epilepsy myoclonic antiepileptic jerks in medications early morning hours upon wakening Benign Partial seizures Classic interictal Seizures usually partial during spikes disappear by epilepsy wakefulness from the adolescence (oral, vocal) centrotemporal, (rolandic) region Landau Normal children Bilateral temporal Antiepileptic Klefner and lose spike and sharp medications syndrome language waves on EEG. between 3- 6 years; often confused with autism 48 Pre-NEET Pediatrics

ANTIMICROBIAL THERAPY OF CNS BACTERIAL INFECTIONS BASED ON PATHOGENA

Organism Antibiotic

Neisseria meningitides Penicillin - sensitive Penicillin G or ampicillin Penicillin - resistant Ceftriaxone or cefotaxime Streptococus pneumoniae Penicillin - sensitive Penicillin G (Ceftriaxone or cefotaxime) Penicillin - intermediate Vancomycin Gram-negative bacilli (except Ceftriaxone or cefotaxime Pseudomonas spp.) Pseudomonas aeruginosa Ceftazidime or Cefepime or Meropenam Staphylococci spp. Methicillin-sensitive Nafcillin Methicillin-resistant Vancomycin Listeria monocytogenes Ampicillin + gentamicin Ceftriaxone or cefotaxime Streptococcus agalactiae Penicillin G or ampicillin • Cerebral palsy: it is defined as a non progressive neuromotor disorder of cerebral origin

CLINICAL MANIFESTATIONS OF CP A. Spastic CP – MC type, 65% – May be diplegia, hemiparesis or quadriplegia – Stretch tendon reflexes always brisk – Variable degrees of mental, visual and behavioral problems – Seizures are common B. Hypotonic or Atonic CP – Patients are atonic or hypotonic – Tendon reflexes are normal, or brisk & Babinski response is (+) – Severely mentally retarded C. Extrapyramidal CP – such as , choreiform movements, , tremors and rigidity are seen. – Arms, leg, neck and trunk may be involved. – Mental retardation and hearing deficits may be present. – Cerebellar involvement occurs in less then 5% of cases. Pediatrics in Last Minutes 49

Mental Retardation • MR - Abnormalities in intellectual/adaptive function. • Age on onset before 18 years/age of maturity • Most common cause = Chronic anomalies (Down's syndrome) • Most common cause in males = Down's syndrome (mild - to - moderate MR) • Most common cause of severe MR in males = Fragile X syndrome – Normal = 90 - 120 – Borderline = 70 - 90 – Mild = 51 - 70/ 50 -70 – Moderate = 36 - 50/ 35 - 50 (trainable) – Severe = 21 - 35 (able to guard cannot manage self) – Profound = <20 (unable to guard self) – Severe and profound MR = custodial

Intelligence Assessment Scales • Weschler scale - Adult • WISC - R - Weschler intelligent scale for children revised • WPPSI ' Pre school • Mallin's ' Modification of Weschler 50 Pre-NEET Pediatrics

The "Original" Six Exanthematous Illnesses 1. First disease Measles 2. Second disease Scarlet fever 3. Third disease Rubella (German measles) 4. Fourth disease Duke's disease (scarlantinella) 5. Fifth disease Erythema infectiosum 6. Sixth disease Exanthem subitum ( infantum)

Measles (Rubeola, First Disease) Cause • RNA paramyxovirus; IP = 8-12days

Spread • By airborne droplets from nose and throat secretions, usually 4 days before and 5 days after the appearance of the rash. • Secondary attack rate is 80%.

Age • Children 6 months -3 years of age in developing countries and older children (>5 years) in developed countries.

Immunity • One attack of measles generally confers life long immunity. • Maternal antibodies protect in infants up to 6 month of age.

Nutrition • Mortality 400 times higher in the malnourished child (also vit A. deficiency) and also these children excrete measles virus for longer periods indicating prolonged risk to themselves and, of spread to others.

Prodromal Symptoms • Last 2-3 days, cough, coryza, conjunctivitis, fever, conjunctivae look glassy, and then the semilunar fold swells (Meyer's sign). Pediatrics in Last Minutes 51

Koplik's Spots • Are 1-2 mm bluish-white spots on a red base on the buccal mucosa opposite the first/second lower molars, pathogenomic of measles, generally seen during the first 2 days of symptoms and are often fading as the rash appears

Comby's Sign • Thin, whitish patches on the gums and buccal mucosa formed of degenerated squamous epithelium.

Rash • On the fourth/fifth day, the macular rash appears consisting of discrete lesions that begins behind the ears and become confluent as rash spreads from hairline downward, sparing palms and soles; • Rash and fever disappear in 3-4 days signalling the end of the disease. Prolonged fever suggests a complication of measles. The entire illness lasts about 10 days.

Complications • Respiratory: , pneumonia, fatal giant cell pneumonitis in the immunosuppressed, flare up of primary TB. • Neurological: febrile fits, meningitis, encephalitis, subacute sclerosing panencephalitis (late and rare): • Digestive: resistant diarrhoea, achlorhydria, hepatitis, appendicitis (due to lymphoid tissue blocking the lumen of the appendix); • Miscellaneous: myocarditis, glomerulonephritis, thrombocytopenic purpura, tuberculin anergy, keratoconunctivitis sicca, intrauterine infection may cause fetal malformations.

Diagnosis • Serology for measles IgM antibody. Warthin-Finkleday cells are multinucleate giant cells with inclusion bodies in the nucleus and cytoplasm, found in respiratory and lymphoid tissues, pathognomonic of measles. 52 Pre-NEET Pediatrics

Prevention • : a live attenuated, tissue-culture vaccine presented as freezdried product. • WHO recommends immunization at 9 months of age. 0.5ml SC injection, reconstituted vaccine should be kept on ice and used within 1 hour. 5-10 days after immunization, a mild 'measles' illness (fever and rash) may occur - self limiting. • Immunity develops 11-12 days after vaccination. Susceptible contacts over 9-12 months age, may be protected against measles with measles vaccine, provides this is given within 3 days of exposure.

Rubella (German Measles, Third Disease) Cause • Togavirus, RNA virus; IP= 14-21 days (average 18 days). The patient is infective 5 days before and 5 days after the day the rash starts.

Presentation • Postauricular, cervical and suboccipital lymphadenopathy (may appear as early as 7 days before the appearance of the rash). • Macular rash spreads from hairline downwards, clearing as it spreads. The rash disappears aitogether bythe third day. -

Forschemier's Spots • (palatal petechiae), also seen in scarlet fever and .

Test • The most widely used serological diagnostic test is the haemagglutination inhibition test.

Treatment • Usually none needed, self limiting Pediatrics in Last Minutes 53

Complications • Arthritis especially among women, encephalitis, thrombocytopenic purpura, malformations in utero. • Infection during the first 4 weeks of pregnancy: Cataract in 70%; during weeks 4-8: patent ductus arteriosus; during weeks 8-12: deafness.

Prevention • Live attenuated RA 27/3 vaccine, produced in human diploid fibroblast; seroconversion occurs in more than 95%; pregnancy is considered a contraindication to rubella immunization.

Erythema Infectiosum (Fifth Disease) • Cause: Human parvovirus B19, primarily affects children 3-12 years old.. • Manifests as a bright-red 'slapped cheek' appearance (a raised, fiery flush on the cheeks), followed by a diffuse lacy reticular rash (on the limbs) that waxes and wanes over 3 weeks. • Adults with fifth disease often have arthritis, and fetal hydrops can develop in association with this condition in pregnant women. • Parvovirus B19 is also the cause of aplastic crises in sickle cell disease.

Exanthem Subitum (Roseola Infantum, Sixth Disease) • Cause: Human herpes virus 6 (HHV-6B). • MC in children under 3 years of age. As in erythema infectiosum, rash usually appears after fever has subsided. It is a rose-pink maculopapular rash (spa rinface); resolves within 2 days.

Chicken Pox (Varicella) Etiology – Varicella-zoster virus also called human (alpha) herpes virus 3. Recovery from primary infection is commonly followed by the establishment of latent infection in the sensory ganglia often without clinical manifestations. 54 Pre-NEET Pediatrics

– Source of infection is usually a case of chicken pox. Spread by respiratory droplets and from the vesicular fluid during the first 3 days of the illness. The scabs however are not infective. – Period of infectivity is from 1-2 days before the appearance of the rash and 4-5 days thereafter. – Secondary attack rate is about 90%. One attack usually gives immunity for life.

Presentation – Incubation period =14 to 16 days. – Prodromal stage consists of fever, malaise and shivering. – Rash comes on the day the fever starts. It is centri petal in distributiort,first appears on the trunk where it is abundant and then on the extremities. Mucosal surfaces are also involved, but palms and soles are unaffected. – The rash advances quickly through the stages of macule, papule, vesicle, pustule and scab (pleomorphic rash)

Complications – Secondary bacterial infection of the rashes; varicella pneumonia; haemorrhages into the lesion; encephalitis, acute cerebellar ataxia; purpura fulminans; arthritis; glomeru- loniphritis; Reye's syndrome.

Treatment – Symptomatic and supportive, since this is a self-limiting condition. Aspirin should NOT be used as it may increase the risk of developing Reye's syndrome.

Prevention – Varicella zoster immunoglobulin, 1.25 to 5 ml given within 72 hours of exposure will modify or prevent the disease. – Live attenuated chicken pox vaccine is now available. Pediatrics in Last Minutes 55

Periods of Infectivity In Childhood Infectious Disease

Disease Infectious periods Chickenpox 5 days before rash to 6 days after last crop Diphtheria 2-3 weeks (shorter with antibiotic therapy) Mumps 3 days before salivary swelling to 7 days after Rubella 7 days before onset of rash to 4 days after Measles From onset of prodromal symptoms to 4 days after onset of rash Scarlet fever 10-21 days after onset of rash (shortened to 1 day by penicillin) Whooping cough 7 days after exposure to 3 wks after onset of symptoms (shortened to abt 7 days by antibiotics)

Precocious Puberty • Precocious puberty is defined as the onset of secondary sexual characters before 8 yr of age in girls and 9 yr in boys. • True (central) precocious puberty (gonadotropin dependant) is due to hypothalamic-pituitary-gonadal activation. Caused by – Idiopathic – Organic brain lesions: brain tumours, severe head trauma, hydrocephalus, hypothalamic hamartoma, CNS infections. – Hypothyroidism, prolonged and untreated. • Precocious pseudopuberty (peripheral, gonadotropin independent): here secondary sex characters appear but there is no activation of hypothalamic-pituitary-gonadal axis. Caused by – Females: Congenital adrenal hyperplasia, Hypothyroidism, Exogenous estrogen, McCune Albright syndrome, Aromatase excess syndrome. – Males: Congenital adrenal hyperplasia, Hypothyroidism, Exogenous androgens, Activating LH receptor mutations, HCG-secreting tumour.

Common Pediatric Abdominal Malignancies Wilms Tumour • MC renal malignancy of early childhood (2-4 years). • Risk factors: – WAGR - Wilms tumour, Ariiridia, Genital anomalies, mental Retardation. – Denys-Drash syndrome (Gonadal dysgenesis and nephropathy leading to renal failure, dominant negative mutation in the WT-1 gene; 56 Pre-NEET Pediatrics

– Beckwith-Wideman syndrome (Enlargement of body organs, hemihypertrophy, renal medullary cysts, adrenal cytomegaly). • Huge palpable flank mass, hemihypertrophy, hematuria, hypertension. • Diagnosis by a/Ultrasound and confirm by excisional biopsy. • Treatment transabdominal nephrectomy followed by chemotherapy (vincristine, dactinomycin); prognosis is usually good.

Neuroblastoma • MC tumour of adrenal medulla but can occur anywhere along sympathetic chain; affects children <5 years. • Risk factors: N-myc oncogene; Neurofibromatosis, tuberous sclerosis, pheochromocytoma, Hirschsprung's disease. • Clinically: abdominal mass, hepatomegaly, leg edema, periorbital bruising. • HVA in urine • Treatment localized tumours cured with excision; chemotherapy and radiotherapy can be used as adjunct. Good prognosis if diagnosed <1 year of age. Shimada classification is used.

D/D of Wilms' Tumour, Abdominal Neuroblastoma

Wilms tumor Neuroblastoma

Age in years 2 - 5 <2 yrs, 30% under 1 year Health Well Usually ill, Lethargic Clinical Swollen abdomen Pale, weight loss, diffuse bone pain Mass Lobulated Irregular edge, craggy hard Crosses midline Rare Common Bilateral Rare Occasional Renal pelvis Grossly distorted Pushed down by mass above Ultrasound Calcification found Metastases Lungs Bone (orbits classically) Pediatrics in Last Minutes 57

Approach to a Child with Anemia

FAB classification divides AML into eight types M0 to M7. This scheme takes into account: i. The degree of maturation (Mo to M3) ii. The lineage of leukemic blast (M4 to M7).

Class Blast cells

Mo minimally differentiated AML • Myeloperoxidase negative • Auer rods negative • Express myeloid lineage antigen

M1 AML without differentiation •  3 % blasts myeloperoxidase positive • Auer rods positive

M2 AML with maturation • Full range of myeloid maturation • Myeloperoxidase positive • Auer rods positive

M3 Acute promyelocytic • Maximum Auer rods.

Contd... 58 Pre-NEET Pediatrics

Contd...

Class Blast cells

leukemia • Myeloperoxidase positive

M4 Acute myelomonocytic • Both myelocytic and Monocytic leulcemia differentiation – Myeloperoxidase (+) ye } Myeloblastic – Auer rods (+) ye – Monoblastic – Nonspecific esterase (+) ve - Monoblastic

M5 Acute monocytic leukemia • Non specific esterase (+) ve • Myeloperoxidase and Auer rods (-) ve

M6 Acute erythroleukemia • Dysplastic erythroid precursors • Myeloblasts seen in advanced age

M7 Acute Megakaryocytic • Blasts of megakaryocytic lineage • GP IIb/ IIIa or VwF (+) ve Juvenile Rheumatoid Arthritis can be divided in 3 major clinical types Pediatrics in Last Minutes 59

GLYCOGEN STORAGE DISEASES

• Glycogen storage diseases result from a hereditary deficiency of one of the enzymes involved in the synthesis or sequential degradation of glycogen. • Clinical features depend on type of enzyme deficiency and organ involvement. • Glycogen storage diseases can be divided into: 1. Hepatic form (Liver glycogenoses) – Liver is the key player in glycogen metabolism. – It contains enzymes that synthesize glycogen for storage and ultimately break it down into free glucose, which is then released into blood. – So, Aninherited deficiency of hepatic enzymes that are involved in glycogen metabolism leads to i. Storage of glycogen in liver - Hepatomegaly. ii. Reduction in blood glucose - Hypoglycemia. – Examples of liver glycogenoses are - i. Type I glycogenosis - Glucose - 6 - phosphatase deficiency (Von Gierke disease). ii. Type III glycogenosis - Debranching enzyme def iii. Type IV glycogenesis - Branching enzyme def iv. Type VI glycogenosis - Liver phosphorylase def 2. Myopathic form (Muscular glycogenoses) – In skeletal muscle, as apposed to the liver, glycogen is used predominantly as a source of energy. – This is derived by glycolysis, which leads ultimately to the formation of lactate. – So, enzyme deficiency leads to: i. Glycogen deposition in muscle ii. Muscle cramp after exercise iii. Exercise - induced rise in blood lactose levels owing to block in glycolysis iv. There may be myoglobinemia – Examples are: i. Type V glycogenosis - phosphorylase deficiency (McArdle disease). ii. Type II glycogenosis -phosphofructokinase def 60 Pre-NEET Pediatrics

3. Miscellaneous types – Associated with glycogen storage in many organs and death in early life. – Examples: i. Type II glycogenosis - Acid maltase deficiency (Pompe 's disease) Pediatrics in Last Minutes 61

An Approach to IEM with Acidosis

Important Lysosomal Storage Diseases

Disease Deficient enzyme Accumulating metabolites Sphingolipidoses -galactosidase GM1 ganglioside • GM1, gangliosidosis Hexosaminidase -  subunit GM2 ganglioside • GM2, gangliosidosis Hexosaminidase -  subunit GM2 ganglioside i. Tay-sachs disease ii. Sandoff disease Glycogenosis type II  - 1,4, glucosidase Glycogen • Pompe disease Mycopolysaccharidose • MPS I (Hurler)  - 1, Iduranidase Dermatan sulfate • MPS II (Hunter) Induronate sulfatase Heparan sulfate • Scheie syndrome Sulfoiduranate sulfatase Contd... 62 Pre-NEET Pediatrics

Contd... Disease Deficient enzyme Accumulating metabolites

Sulfatidoses • Metachromatic Arylsulfatase A Sulfatide leukodystrophy • Krabbe disease • Gaucher disease Galactosylceramidase Galactocerebroside • Fabry's disease Glucocerebrosidase Glucocerebroside • Niemann- - Galactosidase A Ceramide trihoxoside Pick disease Sphingomyelinase Sphingomyelin Others • Wolman disease Acid lipase Cholesterol ester, triglyceride LEUCODYSTROPHY • Leucodystrophy refers to progressive degeneration of the white matter of the brain due to imperfect growth or development of the myelin sheath, the fatty covering that acts as an insulator around nerve fibre. • Myelin which lends its color to the white matter of the brain, is a complex substance made up of at least ten different chemicals. • The leucodystrophy are a group of disorders that are caused by genetic defects in how myelin produces or metabolizes these chemicals. • Each of the leucodystrophies in the result of a defect in the gene that controls one (and only one) of the chemicals. Specific leucodystrophies include: – Metachromatic – Adrenoleuco- – Canavans disease leucodystrophy dystrophy – Krabbe 's disease – Pelizaeus- – Alexander disease Merzhacher disease • Symptoms vary according to the specific type of leucodystrophy and may be difficult to recognize in the early stages of the disease. Canavan's Disease • Autosomal recessive disorder • Caused due to deficiency of the enzyme N-Aspertoacylase. • This leads to accumulation of N-Acetyl aspartic acid in brain and urine. Pediatrics in Last Minutes 63

• It is characterized by the clinical traid of - , Head lag, Macrocephaly Adrenoleucodystrophy • X-linked recessive disorder • Caused due to deficiency of Acyl-CoA synthetase. • Onset is about 5-10 years • Main symptoms are ataxia, spasticity, motor deficits, cortical blindness. • Marocephaly is not a key feature Metachromatic leucodystrophy • Autosomal recessive disorder • Caused due to Arylsulfatase A deficiency • Onset is in the 2nd year. • Symptoms are inco-ordination, especially gait disturbance, then general regression, optic atrophy, combined upper and lower motor signs. • Marocephaly usually late. Krabbe leucodystrophy • Autosomal recessive • Caused due to deficiency of Beta-galactosidase deficiency • Onset is in the first 6 months of life • Optic atrophy, spasticity • Head often small

Wolman's Disease • Wolman disease is very rare, with only 50 reports of the disease published in the worldwide medical literature. It is an autosomal recessivedisease and affects both males and females. • The disease affects the breakdown and use of fats and cholesterol in the body and belongs to a family of disorders called lipid storage disorders. • Wolman disease is caused by mutations in the LIPA gene. The LIPA gene encodes for an enzyme called lysosomal acid lipase which is found in the lysosomes. It breaks down fats such as cholesteryl esters and triglycerides so that they can be used by the body. 64 Pre-NEET Pediatrics

• A shortage of lysosomal acid lipase leads to accumulation of triglycerides, cholesteryl esters and other fats within the cells of the affected individuals. This accumulation as well as malnutrition caused by the body's inability to use lipids properly result in of wolmans disease.

Clinical Features • In affected individuals harmful amounts of lipids accumulate in the spleen, liver, bone marrow, small intestine, adrenal glands and lymphnodes. • In addition to fat deposits, calcium deposits in the adrenal gland are also seen. • Infants with wolman disease are healthy and active at birth but soon develops signs and symptoms of the disorder. These may include – Enlarged liver and spleen – Poor weight gain – Low muscle tone – Jaundice – Vomiting – Dianthoea – Developmental delays – Anemia (low amount of iron in blood) poor absorption of nutrients from food. • Children affected by this condition develop severe malnutrition and generally do not survive past early childhood.

Diagnosis X-Ray Findings of Wolman Disease Characteristic pattern of calcification (outlining the outline of the cortex of both the glands) in enlarged but normally shaped adrenal glands.

CT Scan and MRI • It shows enlargement of the liver and spleen, calcification of both adrenal glands and lymphadenopathy (enlarged lymph nodes). • The definitive diagnosis of wolmans disease is made by Pediatrics in Last Minutes 65

demonstration of deficient lysosomal acid lipase activity in leucocytes. • Amniocentesis can be performed for prenatal diagnosis of wolmans disease.

Treatment • Currently there is no cure for wolmans disease so treatment focuses on management of symptoms. If the adrenal gland is not making enough hormones and steroids, replacement steroids and hormones can be given. Pre NEET Pediatric INCLUDING RECENT ALL INDIA AND AIIMS TOPICS

Q U E S T I O N S

CLINICAL QUESTIONS AND QUESTION BASED ON EXPECTED NEET PATTERN

Q 1. A neonate delivered at 37 wks of gestation with a birth weight of 1.9 kg develops intolerance to feed/ lethargy/abdominal distension on 2nd day. Sepsis screens is negative. Physical examination is unremarkable. PCV is observed to be 72%. Which of the following represents with best management option: a. Hydration with IV fluids b. Partial exchange transfusion c. Presumptive treatment for sepsis d. Medical management for intestinal obstruction Q 2. A premature baby born at 33 wks of gestation with a birth weight of 1500 gm has stable vitals. Which of the following you will choose as initial feeding method of choice: a. Intravenous fluids b. Intravenous fluids and oral feeding c. Orogastric tube/ alternate oral route d. Total parenteral nutrition Q 3. A 2 yrs old male child presents to paediatric emergency department with history of fever x 2 wks. He has received multiple antibiotics. Blood tests for Typhoid, Malaria, Dengue are all negative. He however had sterile pyuria in one of reports (urine C/s - sterile). BCG scar is indurated and H/o conjunctivitis, chelitis and erythematous rash over body for initial 5 days of illness is obtained the most likely treatment of choice is: a. Cyclosporin b. Prednosolone c. Methotrexate d. IvIg Questions 67

Q 4. A child presented with intermittent episodes of left sided flank pain. USG KUB reveals a large hydronephrosis with dilated renal pelvis and cortical thinning with a normal ureter. Kidney differential function was observed to be 19%, which of the following is the best management: a. Nephrectomy b. Pyeloplasty c. External drainage d. Endopylostomy Q 5. Which of the following statements about neuroblastoma is false: a. Most common extracranial solid tumor in children b. >50% present with metastasis at the time of diagnosis c. Lung metastasis is common d. Often encase aorta and its branches at time of diagnosis Q 6. A very low birth weight preterm baby is on ventilator for respiratory distress. Baby presents with clinical features of necrotizing enterocolitis with perforation. What is the appropriate management? a. Conservative management b. Immediate laparotomy c. Extra corporeal membrane oxygenation with surgery after stabilization d. Peritoneal drainage Q 7. A child with 3 days history of upper respiratory tract infection presents with moderate respiratory distress and stridor at rest. What is the most appropriate management? a. A single daily dose of I/V ceftriaxone b. Mechanical ventilation c. Hospitalization and Nebulization with racemic epinephrine diluted with water. d. Reassure parents and advise on for fever. Q 8. A child is brought with drowsiness, decreased deep tendon reflexes and seizures. On examination the child has a blue line on gums. There is a history of constipation. Which will be most appropriate drug that should be used in this child? a. EDTA b. DMSA c. BAL d. Pencillamine 68 Pre-NEET Pediatrics

Q 9. A 7 yr old girl is bought with complaints of generalized swelling of body. Urinary examination reveals grade 3 proteinuria and the presence of hyaline and fatty casts. She has no history of hematuria, which of the following statements about her condition is true: a. No IgG deposits or C3 deposits on renal biopsy b. L3 level will be low c. IgA nephropathy is the likely diagnosis d. Alport's syndrome is the likely diagnosis Q 10. A 8 yr old male presents to paediatric casualty with non blanching rashes on skin, swelling of knee joint x 2 days, Malena associated with abdominal pain x 1 day. Platelet count, prothrombin time are normal. Urine R/M shows 2+ protein, hematuria. What should be your line of management? a. Symptomatic treatment with rest and analgesia b. Short course of steroids c. Steroids with immunosuppressive d. Skin biopsy and orthopaedic referral Q 11. A 2 yrs old boy presents with short stature and bowing of legs. X-ray knee shows evidence of rickets. He has been given Vit D twice 2 months back. Blood investigation shows S.calcium 9 mg/dL, S. Phosphate 2 mg/dL, S. Alkaline phosphatase 1030 µg/dL, S.PTH and S. Bicarbonate levels are normal. What should be the next line of management? a. Give calcitriol in very large doses b. Oral phosphate and Vit D supplements c. Oral calcium and phosphate supplements d. Restrict phosphate, give calcium and vit D supplements Q 12. A six months old girl is having recurrent UTI. Ultrasound abdomen shows bilateral hydronephrosis. MCU (Micturating cysts urethrogram) shows bilateral grade IV vesicoureteral reflux. The treatment of choice is: a. Endoscopic injection of polyteflon at uretic orifice b. Uretric reimplantation c. Bilateral ureterostomy d. Prophylactic antibiotics Q 13. A 10 months old boy, weighing 3 kg has polyuria, polydipsia and delayed motor milestones. Investigations show blood level of creatinine 0.5 mg/dL, potassium 3 meq.L, sodium 125 meq/L, chloride Questions 69

88 meq/L calcium 8.8 mg/dL, PH 7.46 and bicarbonate 26 meq/L Ultrasonography shows medullary nephrocalcinosis. The most likely diagnosis is: a. Renal tubular acidosis b. Diabetes syndrome c. d. Q 14. A 3 yr old child with acute onset of fever of 1040F developed febrile seizures and was treated. He had similar episode at 2, 2 ½ yr of age. Which of the following statements is false? a. The risk of epilepsy is 4% in this case b. Long term phenytoin is required c. Intermittent clobazan and paracetamol should be used d. Consider EEG and imaging Q 15. A 14 yr old boy has history of frequent myoclonic jerks on awakening, making hair combing and tooth brushing difficult for past 6 months. Now, he has presented to casualty with generalised tonic clonic seizure. Neurological examination is normal, which of the following statements is true? a. Drug of choice is carbamazepine b. Carbamazepine can be discontinued after complete remission c. Response to anticonvulsants is poor d. EEG in this condition shows a 4-6/ sec irregular spike and wave pattern, which is enhanced by photic stimulation. Q 16. A 2 yr old child is bought by parents with history of seizures and development delay. He has multiple hypopigmented macules over the back. What is the most probable diagnosis? a. Neurofibromatosis type I b. Tuberous sclerosis c. Struge Weber's syndrome d. Vao Nippel Lindau disease Q 17. A 5yr old child is admitted with , vomiting and difficulty in walking. Physical findings include truncal ataxia, papilledema and left lateral rectus palsy. Finger to nose test was normal. The most likely diagnosis is: a. Dandy Walker syndrome b. Syringobulbia c. Arnold Chiari malformation d. Medulloblastoma. 70 Pre-NEET Pediatrics

Q 18. A 5 months old formula fed infant has been brought with complaints of watery diarrhoea of 2 days duration and irritability of one day. Physical examination reveals a markedly irritable child with a rather doughy skin and rapid pulse, borderline CFT. Which of the following statements is true? a. Give I/V bolus of normal saline b. Give I/V bolus of ringer lactate c. Calculate water deficit and replace over 24 hrs at a rate of 0.5 meq/L/hr. d. Treat as any dehydration and give moderate dehydration correction. Q 19. Study the following inheritance pattern Which of the following statements is true?

a. Autosomal dominant b. Mito chondrial c. Autosomal recessive d. X-linked dominant Q 20. A newborn is born to a HIV positive woman an ART. He was given Nevirapine at birth. Which of the following statements regarding future management is false? a. Start Cotrimoxazole prophylaxis at 1 month till HIV infection can be ruledout. b. Advise exclusive top feeding or exclusive breast feeding c. Perform DNA PCR for HIV at 6-8 wks of age. d. Do not give OPV and BCG at birth. Questions 71

Q 21. A. Assertion: An infant presents with hypotonia and areflexia. During his intrauterine period there was polyhydramnios and decreased fetal movements. He is diagnosed as floppy infant. B. Reasoning: Electromyography will show a decremental response on repetitive stimulation and muscle biopsy will be normal. a. Both statement A and B are true, and B is the correct explanation of A b. Statement A is true while statement B is false. c. Statement A is false while statement B is true. d. Both statements A and B are true but B is not the correct explanation of A. Q 22. Previously well newborn discharged as "healthy" from the nursery at D1 of life presented casualty at D7 of life with poor feeding, poor pulses, tacycardia, an ashen appearance. He was immediately intubated and put on inotropic support. Sepsis serum was negative. Urgent echo done was suggestive of small left ventricle with atretic mitral valve. Prostaglandin E1 infusion was immediately started. He fails to improve with same. What should be next line f management? a. Increase inotropic support and continue same management b. Emergency atrial septostomy c. Corrective surgery d. Upgrade antibiotics Q 23. A 1 ½ yr old child presents to paediatric casualty with irritability, restlessness, severe respiratory distress and cyanosis. Mother gives history of vigorous crying ½ hr back. On auscultation, a short systolic murmur and soft 2nd heart sound are heard in left 3rd and 4th parasternal spaces. What should be the immediately intervention? a. Assurance of mother that nothing should be done and he will become alright spontaneously. b. An urgent echo and chest x-ray is planned. c. Knee-chest position, oxygen and Inj. Morphine administration d.  agonists like Isoxuprine administration Q 24. A 1 yr old infant presented to paediatric casualty with excessive crying, irritability and billous vomiting. Mother gives history of passage of blood per rectum once mixed with stools. He is hemodynamically stable but not allowing examination. X-ray abdomen shows ascites with distended bowel loops.What should be next line of management: a. Manage as a case of acute invasive gastroenteritis with I.V. antibiotics and I.V. fluids 72 Pre-NEET Pediatrics

b. Send PT, aPTT and give Inj Vit K c. Do urgent USG and perform hydrostatic reduction. d. Do urgent USG and prepare for urgent laparotomy Q 25. A 8 day old breast fed male baby presents with vomiting, poor feeding and loose stools. On examination the heart rate is 190/min, blood pressure 50/30 mmHg, respiratory rate 72 breaths/ minute and capillary refill time of 4 sec. Investigations show Hb 15 gm/dL, SNa+ - 120 meq/L K+ - 6.8 meq/L, Cl- 81 meq/L, bicarbonate 15 meq/L, creatinine 0.6 mg/dL. The most likely diagnosis is: a. 21 hydroxylase deficiency b. 3- hydroxy steroid deficiency c. 11 hydroxylase deficiency d. Aldosterone synthase deficiency Q 26. A. Assertion: A 8 day old child presents yellow sclera, whitish stool and turmeric colour urine with reducing substance positive on 3rd day of septicaemia on broad spectrum antibiotics. R. Reasoning: This is a case of severe sepsis and antibiotics should be immediately upgraded a. A and R both correct and R is explanation of A b. A and R both correct but R is not explanation of A c. A correct R wrong d. A wrong R correct Q 27. A: Assertion: A 1 month baby presents with history of jaundice, turmeric colored urine and pale stools since birth. Examination reveals liver span of 10 cm TORCH IgM is positive for Rubella. R: Reasoning: A liver biopsy should be done to differentiate neonatal hepatitis from biliary atresia a. A and R both correct and R is explanation of A b. A and R both correct but R is not explanation of A c. A correct R wrong d. A wrong R correct Q 28. 30 yr old lady delivered a healthy baby at 37 wk of gestation, she was positive for HBSAg but negative for HBCAg. Which of the following is the most appropriate treatment for the baby: a. Both active and passive immunization soon after birth b. Passive immunization soon after birth and active immunization at 1 yr of age. c. Only passive immunization soon after birth d. Only active immunization soon after birth Questions 73

Q 29. A 7 yr old girl presents to casualty with history of fever x 4 days, epistaxis and vomiting. She is diagnosed to be Dengue Heorrhagic fever (DHF), which of the following does not constitute the diagnosis of DHF? a. Thrombocytopenia b. Hypoalbuminemia c. Pleural effusion d. Decreased hematocrit Q 30. A child died soon after birth. On examination there was pallor, hepato splenomegally and edema all over body. Most probable diagnosis is: a.  - Thalassemia b.  - Thalassemia c. Herditary spherocytosis d. ABO in computability/ sickle cell anemia

AIIMS MAY 2012

Q 1. Which of the following will favour the diagnosis RDS in newborn? (AIIMS Nov 2010) a. Reciept of antenatal steroids b. Air bronchogram in chest x-ray c. Manifests after 6 hours d. Occurs after term gestation

Q 2. All the following can occur in a neonate for heat production except (AIIMS Nov 2006) a. Shivering b. Universal flexion like a fetus c. Breakdown of brown fat with adrenaline secretion 74 Pre-NEET Pediatrics

d. Cutaneous Vasoconstriction Q 3. Due to advances in Cancer treatment the prognosis of which of the following has become better? a. Glioblastoma multiforme b. Esophageal carcinoma c. ALL in children d. Cholangio carcinoma Q 4. A newborn baby presented with profuse bleeding from the umbilical stump after birth. Rest of the examination and PT, APTT are within normal limits. Most probable diagnosis is: a. Factor X deficiency b. Glanzmann thrombasthenia c. Van Willebrand disease d. Bernard Soullier disease

ALL INDIA PG MEDICAL ENTRANCE EXAMINATION 2012

Q 1. A 1 year old girl presents with fever since 24 hrs and on and off coughing. She is passing foul-smelling bulky stools and had 4 attacks of bronchitis in past. The girl is suspected of having: a. Cystic fibrosis b. Maple synp urine disease c. Phenylketonuria d. Impaired glucoronidization Q 2. A 2 yrs old boy keeps on looking at his own hands and does not respond on calling. His mother development is otherwise normal, probable diagnosis would be: a. Autism b. Depression c. ADHD d. Schizophrenia Q 3. Regarding parenteral nutrition of newborns, 20% intralipid has are the following advantage over 10% intralipid solution except a. Reduced phospholipids b. Increased EFA (essential fatty acids) Questions 75

c. Increased Calorie density/ mL of fluid d. Better/improved trigly ceride clearance Q 4. Most frequently identified fetal tumor? a. Neuroblastoma b. Wilms tumor c. Fibro sarcoma d. Sacrococcygeal teratoma Q 5. A case of meningomyelocoele was posted for surgery. Till patient is waiting for surgery the covering of sac will be protected by a gauze piece soaked in a. Normal saline b. Tincture iodine c. Methylene blue d. Mercuro chrome Q 6. Least chances of infective endocarditis is seen with a. Small VSD b. Small ASD c. Mild As d. Mild AR Q 7. True about jaundice in neonates is: a. Can be seen after ventouse delivery b. Physiological jaundice is seen with 48 hours of birth c. Increased conjugated bilirubin leads to kernicterus d. Breast milk jaundice is maximum in 7 days from birth Q 8. A term infant born to a diabetic mother was lethargic few hours after birth and his blood glucose was 3 mg%. What should be done next? a. Give 10% Dextrose orally b. 10% Dextrose I.V. c. Give expressed breast milk d. Do exchange transfusion Q 9. A mentally challenged child with dysphagia and Ophistotonic spasms is also having Choreoathetoid movements and self-mutilating behaviour with positive family history. Which of the following investigations is suggested? a. S. Uric acid 76 Pre-NEET Pediatrics

b. S. Alkaline phosphatase c. S. LDH d. Lead levels in blood Q 10. A child who can use 4-5 words including meaningful nouns and represent his ideas mostly by non-verbal communication then what is his probable age? a. 15 months b. 18 months c. 24 months d. 12 months Q 11. All of the following can occur in a neonate for heat production except: a. Shivering b. Universal flexion like a fetus c. Breakdown of brown fat with adrenaline secretion d. Cutaneous vasoconstriction Q 12. A down's child mentally retarded, a/e a. Deleted 21 b. Trisomy 21 c. Robert Sonian d. Mosaic

AIIMS NOVEMBER 2011

Q 1. PGE causes worsening in infant with: a. PS without VSD b. Hypoplastic left heart syndrome c. Obstructive TAPVC d. Obstruction in aorta Q 2. Premature baby of 34 wks was delivered. Baby had bullous lesion on the body x-ray shows periostitis. What is the next investigation. a. VDRL for mother and baby b. EIISA for HIV c. PCR for T.B. d. Hepatitis surface antigen for mother Questions 77

Q 3. A newborn presents with congestive heart failure, on examination has bulging anterior fantanelle with a bruit on auscultation. Transtantanellar USG shows a hypoechoic midline mass with dilated lateral ventricles. Most likely diagnosis is: a. Medullo blastoma b. Encephalocele c. Vein of galen malformation d. Arachnoid cyst Q 4. What is the diagnosis in a patient who has situs inversus and sinusitis: a. Kartagener's syndrome b. Good pasture syndrome c. Cystic fibrosis d. William Campbell syndrome Q 5. Short child with low T4 and raised TSH and swelling of pituitary, what is the diagnosis? a. Pituitary tumour b. Primary hypothyroidism c. TSH secreting pituitary adenoma d. TSH resistance Q 6. Episodic anemia with jaundice since birth except: a. G 6pd b. Sickle cell anemia c. PNH d. Hereditary spherocytosis Pre NEET Pediatric INCLUDING RECENT ALL INDIA AND AIIMS TOPICS

A N S W E R S

CLINICAL QUESTIONS AND QUESTIONS BASED ON EXPECTED NEET PATTEN

1. Ans. is b. Partial exchange transfusion (Ref: Nelson 18th, Pg. 1773, Meharban Singh 7th, Pg. 305) This is a case of neonatal polycythemia which is symptomatic in the form of lethargy and GIT manifestations The management of choice for such a case is partial exchange transfusion. The diagnosis of polycythemia is made when venous or central hematocrit is greater than 65%

Etiology 1. Delayed clamping of cord 2. Twin-twin transfusion syndrome. Causes polycythenia in the recipient twin 3. Infants of diabetic mother 4. Intra uterine growth retarded babies due to chronic . 5. Rare causes: Cyanotic congenital heart disease, Cretinism, Trisomy 13,18 and 21, Beckwtih-Wiedemann syndromes, dehydration.

Clinical Features 1. Cardiopulmonary–Tacypnea, Tacycardia, cyanosis and cardiomegally. 2. CNS–Lethargy, hypotonia, poor feeding, seizures, irritability. 3. Transient renal failure due to reduced perfusion of kidneys. 4. Hyperbiliribenemia 5. Thrombotic complications including renal vein thrombosis 6. Hypoglycemia, hypocalcemia, hypomagnesemia Answers 79

Treatment

The volume for exchange is determined by the following calculation: Volume = (Observed HCt – desired HCt)/Observed HCt X Blood volume HCt = hematocrit Desired HCt = 50% (usually) Blood volume = 80 ml/kg for term babies 100 ml/kg for preterm babies. 2. Ans. is c. Orogastric tube/ alternate oral route (Ref: Meharban Singh 7th, Pg. 191) The mode of initial feeding depends upon the gestational maturity and hemodynamic stability of the baby. – Infants with severe birth asphyxia respiratory distress, apneic attack, sezures, sepsis, NEC, shock babies should be maintained on intravenous infusion till their clinical condition stabilizes. 80 Pre-NEET Pediatrics

Mode of Initial Feeding Based on Gestational Maturity

Gestational age Maturation of feeding skills Initial feeding method (Birth weight)

<30 wks(<1200 gm) No proper sucking Intravenous fluids efforts No propulsive gut motility

30–32 wks Sucking bursts develop Oro-gastric (1200–1500) No coordination between Or Naso - gastric suck/swallow and feeds breathing

32–34 wks Slightly mature sucking Spoon, paladay (1500–1800 gm) Coordination between Or cup feeding breathing and swallowing begins

>34 wks (>1800 gm) Mature sucking Breast feeding More coordination between breathing and swallowing

Trophic feeds are Minimal (sub nutritional) feeds (5–10 ml/kg/d) used in preterm babies with a gestation of <32 wks or birth weight <1500 gm to harness its trophic benefits on the immature gastro intestinal tract of the baby. 3. Ans is d. IvIg (Ref: Nelson 19th, Pg. 863-865) This is most likely a case of Kawasaki disease (KD) formerly known as mucocutaneous lymph node syndrome and infantile polyarteritis nodosa. The characteristic BCG scar induration, conjunctivitis, skin rash and fever not responding to antibiotics point to diagnosis. Classic clinical criteria for Kawasaki disease are: • Fever persisting at least 5 days and presence of at least 4 principal features: a. Changes in extremities i. Acute: Erythema of palms, soles, edema of hands, feet. ii. Subacute: Periungual peeling fingers, toes in weeks 2 and 3 b. Polymorphous exanthema c. Bilateral bulbar conjunctival injection without exudate Answers 81

d. Changes in lips and oral cavity: erythem, lip cracking, strawberry tongue e. Cervical lymphadenopathy (>1.5 cm diameter), usually inilateral.

Etiology The cause of KD remains unknown, but certain epidemiologic and clinical features support an infectious origin KD is a vascultis that predominantly affects the medium sized arteries especially the coronary arteries. In the absence of treatment, KD can be divided into 3 clinical phases 1. The acute febrile phage: lasts 1-2 wks 2. The subacute phase: associated with desquamation, thrombocytosisand development of coronary aneurysms and generally lasts about 2 wk. 3. The convalescent phase: begins when all clinical signs of illness have disappeared and continues until ESR returns to normal (about 6 – 8 wk after onset of illness)

Treatment of Kawasaki Disease Acute stage Interavenous immunoglobulin 2 gm/kg X 10-12 hr and Aspirin: 80-100 mg/kg/d until patient is afebrile for at least 48 hr.

Convalescent Stage Aspirin 3-5 mg/kg until 6-8 wk after illness onset.

Long Term Therapy for Patients with Coronary Abnormalities • Aspirin 3-5 mg/kg once daily ± clopidrogel • Warfain/ LMW – heparin added for those patients at high risk of thrombosis

Acute Coronary Thrombosis Prompt fibrinolytic therapy with tPA or other thrombolytic agent. 82 Pre-NEET Pediatrics

4. Ans is b. Pyeloplasty (Ref: Nelson 18th, Pg. 2237, Paediatric Nephrology 5th, Pg. 446-448) The child in question is presenting with characteristic feature of PUJ obstruction (dilated renal pelvis with normal ureters). PUJ (Pelvi Uretric Junction) Obstruction It is caused mostly due to intrinsic muscular abnormalities of the ureter, ureter polyps, ureter folds, crossing vessels and rarely secondary to vesicoureteral reflux. The condition is often asymptomatic and detected on antenatal ultrasonography. Incidental detection of an abdominal mass, hamaturia and UTI are usual features in the older child. Ultrasonography and DTPA renogram establish the diagnosis. Epidemiology Left side is involved more commonly than right (Bilateral = 10%) Male more commonly involved than females (M:F = 2:1) Management Answers 83

A differential function of <40% is considered significant and often an indication for pyeloplasty. Differential function of <10% is often considered a cut off for Nephrectomy. 5. Ans is c. Lung metastasis is common (Ref: Nelson 19th, Pg. 1753-1757) Neuroblastoma is the most common extracranial solid tumor in children and the most commonly diagnosed malignancy in infants. It is derived from primordial neural crest cells and approximately half arise in the adrenal glands and remainder in the paraspinal sympathetic ganglia. Pathogenesis: Etiology most cases remains unknown. Familial neuroblastoma accounts for 1-2% of all cases and associated with a younger age at diagnosis. Genetic characteristics that are of prognostic importance include: i. Poor prognosis: Amplication of MYCN, loss of heterozyqosity of 1p, 11q and 14q gain of 17q. ii. Good prognosis: Hyperdiploidy if child is <1yr at diagnosis

Clinical Manifestations 1. Localized disease: asymptomatic mass or mass related symptoms, including compression, bowel obstruction, SVC syndrome. 2. Metastatic disease: Fever, Bone pain, cytopenias, orbital proptosis, subcutaneous nodules, periorbital ecchymosis. 3. Most common sites of metastasis are lymph nodes, (long), skill, bone marrow, liver and skin. Lung and brain metastasis are rare.

Diagnosis • Usually discovered as a mass on plain radiography, CT or MRI • Tumor markers including catecholamine metabolites homovanillic acid (HVA), VMA (vanillylmandelic acid) are elevated in urine in 95% of cases. • Biopsy from tumor tissue.

Treatment 1. Low risk – Localized tumor: Treatment surgery survival rate 98%. 2. Intermediate risk 84 Pre-NEET Pediatrics

– Localized tumor with regional lymphnode extension, metastasis to bone marrow and in infants. – Treatment: Moderate intensity chemotherapy, debulking surgery – Survival rate: 90-95% 3. High risk – Metastasis to bone marrow and bone. Dose intensive chemotherapy, surgery and external beam radiotherapy to primary tumor and resistant metastatic sites. Survival rate: 40-50% 4. Tumor stage 4S Metastasis to liver and skin Treatment: Supportive care because many regress spontaneously survival rate (>90%). • Current induction chemotherapy for high risk neuroblastoma includes cyclophosphamide, topotecan, doxorubicin, vincristine, cisplatin and etoposide. 6. Ans is d. Peritoneal drainage (Ref: Meharban Singh 7th, Pg. 405-407, Nelson 18th, Pg. 756) The definitive treatment of choice for perforation in cases of NEC is surgery (immediate laparotomy with resection of the involved segment and reanastomosis). However, in premature infants with very low birth weight (<1000gm), who are extremely unstable for surgery (preterm baby on ventilator), peritoneal drainage under local anaesthesia has been described as a safe immediate management option.

Necrotizing Enterocolitis Vanous predisposing factors are: • Prematurity (especially <34 wks) • Perinatal hypoxia • Active resuscitation • RDS/ apnea • Assisted ventilation • Acidosis/ Hypoxia/ Shock • Umbilical artery catheterization • Use of H2 blockers Answers 85

• Early/large volume nasogastric feedings – The protective role of human milk for prevention of NEC is well accepted.

Modified Bells Staging for NEC

Stage Signs Radiology Treatment IA (Suspected Lethargy apnea, Normal or NPO, NEC) bradycardia, intestinal Antibiotics elevated dilation, pre-gavage mild illeus residuals, emesis, guaic positive stool

IB (Suspected Same as above Same as Same as NEC) + bright red above above blood per rectum IIA (Definite Same as above Same as Same as NEC) + absent bowel above + above sounds pneumatosis intestinalis

IIB (Definite Same as above Same as IIA Same + NEC) + mild + portal vein Sodabicar- thrombocy- gas bonate for topenia, acidosis acidosis + abdominal cellulites

IIIA (Advanced Same as IIB Same as IIB Same as NEC) + DIC, neutro- + definite above + penia + signs ascitis assisted of peritonitis ventilation paracentesis

IIIB (Advanced Same as IIIA Same as Same as IIIA NEC) IIIA + pneumo + Surgical peritoneum intervention (Laparo- tomy) 86 Pre-NEET Pediatrics

• In ELBW (<1000 gm) or unstable babies, peritoneal drainage is established under local anaesthesia and laparotomy is delayed for stage IIIB. 7. Ans is c. Hospitalization and Nebulization with racemic epinephrine diluted with water. (Ref: Nelson 18th, Pg. 1765, O.P. Ghai 7th, Pg. 351) This is a case of infectious croup (Acute laryngotracheobronchitis) with moderate severity and requires hospitalization and nebulisation with racemic epinephrine. Laryngo Tracheobronchitis, usually caused by parainfluenza type 1. Bacterial etiology is unusual.

Clinical Features Usually there is mild cold for a few days before the child develops a brassy cough and inspiratory stridor. The condition may worsen to severe respiratory distress and cyanosis, Croup is a clinical diagnosis and does not require a radiograph of the neck. However, characteristic "Steeple sign" (Subglottic narrowing) may be seen on PA view of neck.

Severity of Acute LTB

Mild Moderate Severe General Happy Irritable Agitated or altered appearance sensorium

Stridor Stridor on coughing Stridor at rest, Stridor at rest and or no stridor gets worse on worsens on agitation at rest agitation

Respiratory No distress Tacypnea, chest Marked tacypnea with distress retractions chest retractions

Oxygen >92% in room air >92% in room <92% in room air, saturation air cyanosis

Management Mild cases can be managed on ambulatory basis with symptomatic treatment for fever Antibiotics have no role. Moderately severe patient may need hospitalization and treatment with racemic epinephrine diluted with water. Answers 87

A single intramuscular dose of dexamethasone (0.3 - 0.6 mg/kg) reduces overall severity during first 24 hrs. More recently inhalation of budesonide (1mg BD) x 2 days has shown good results. Severe croup may need hospitalization with oxygen inhalation, steroids. Worsening distress may need short term ventilation.

Differential Diagnosis 1. Epiglottitis: H. Influenza type B is the most frequent causative agent. a. Clinical features: High fever with difficulty in swallowing. He often sits up leaning forwards and saliva dribbling from his chin. b. Diagnosis: Direct laryngoscopy where epiglottis appears angry red. c. Management: Hospitalization, Humidified oxygen, fluids, antibiotics (ampicillin/chloramphenicol/ceftriaxone) ± Ventilation. 2. Spasmodic croup: Occurs in children between 1 and 3 yrs. Usually no prodromal symptoms, sudden onset brassy cough and noisy breathing, usually in early hours of morning. Pathogenesis is unknown. – Treatment: Racemic epinephrine and steroids are helpful. 8. Ans is a. EDTA (Ref: Nelson 19th P. 2450) The child in question is presenting with signs and symptoms of plumbism as suggested by blue line on gums (Bertonian line), gastrointestinal manifestations like constipation and symptoms of lead encephalopathy (seizures, drowsiness). As the patient in question in presenting with encephalopathy, he is likely to have blood levels >70 µg/dL and hence, EDTA is the single most important agent for treatment.

Lead Poisoning Lead is a heavy metal that is purely a toxicant in humans. Lead poisoning may occur in utero, because it readily crosses the placenta from maternal blood. The spectrum of toxicity is similar to that experienced by children after birth. Major sources of exposure include lead based paints, batteries; cable sheathing, cosmetic, plastic, toys, mineral supplements. 88 Pre-NEET Pediatrics

Metabolism • After absorption, lead is disseminated throughout the body. Mostly it accumulates in bone and resides for years. • About 97% in blood is bound on or in Red blood cells. • The heme pathway is susceptible to lead inhibitory effects. • Neurotransmitter release is adversely affected by lead.

Clinical Symptoms 1. GIT symptoms: anorexia, abdominal pain, vomiting, constipation 2. CNS symptoms: related to cerebral edema and increased . – Headache, Lethargy, Seizures, Papilledema, Coma. 3. Renal tubular dysfunction at high levels (>100 µg/dL) – Reversible fanconi syndrome. 4. Hemolytic anemia

Diagnosis and Management Blood lead levels (BLL) remains the gold standard for evaluation. Lead can be measured in urine and hair but has problems of contamination and interpretability.

Drug Treatment A child with a venous BLL 45 µg/dL or higher should be treated. Children with BLL of 44-70 µg/dL may be treated with a single drug, preferably DMSA. Those with BLL of 70 µg/dL or greater require 2 drug treatment. 1. EDTA + DMSA / BAL for those without encephalopathy 2. EDTA and BAL for those with encephalopathy 9. Ans. is a. No IgG deposits or C3 deposits on renal biopsy (Ref: Nelson 19th, Pg. 1804 Paediatric Nephrology 5th, Pg. 195) This girl is having generalized edema, grade 3 proteinuria, Fatty + hyaline casts. This is a case of minimal change Nephrotic syndrome. IgA nephropathy and Alport's syndrome present with acute nephritic syndrome rather than nephrotic syndrome. Nephritic syndrome is characterized by massive proteinuria (3+ - 4+ protein), Hypoalbuminemia (<2.5 gm/dL) Hypercholestrolemia, edema. Answers 89

Approximately 90% children with Nephrotic syndrome are idiopathic that is nephrotic syndrome. without evidence of a specific systemic cause. It includes: Minimal change disease (about 85% of total cases) • Mesangial proliferation • Focal segmental glomerulo sclerosis • Membranous nephropathy • Membranoproliferative glomerulonephritis

Differential Diagnosis

• Protein losing enteropathy • Hepatic failure • Heart failure • Protein malnutrition • Chronic glomenlonephritis

Treatment Children with onset of uncomplicated nephritic syndrome between 1-8 yr of age are likely to have steroid responsive MCNS (Minimal change NS)and steroids therapy may be initiated without a diagnostic renal biopsy. Children with features that make MCNS less likely (Gross hematuria, hypertension, renal insufficiency, hypo-complementenia, age <1 yr or >8yr) should be considered for renal biopsy before treatment. About 90% children with MCNS respond to initial steroid therapy. The initial episode is treated with prednisolone, 2 mg/kg/d for 6 wks. Thereafter, the dose of prednisolone is reduced to 1.5 mg/kg given on alternate days as a single morning dose for 6 more weeks. Further prolongation of alternate day prednisolone therapy for 6 months may be beneficial in reducing the risk of subsequent relapses. 10. Ans. is b. Short course of steroids (Ref: Paediatric Nephrology 5th, Pg. 155-157) This is a classical case presentation of Henoch - Schonlein purpura with skin, joint, GIT and renal manifestation. Use of oral steroids for 2- 3 wks is recommended. Skin biopsy is not mandatory for diagnosis. Henoch: Schonlein purpura 90 Pre-NEET Pediatrics

(Anaphylactoid purpura) It is the most common systemic small vessel vasculitis in children. It occurs frequently between 3 and 10 yrs.

Diagnosis Presence of purpura (commonly palpable) or petechiae with lower limb predominance and one of the four following criteria: 1. Abdominal pain 2. Arthritis/ arthralgia 3. Renal: involvement (proteinuria or hematuria or presence of RBC casts) 4. Histopathology showing Leukocytoclastic vasculitis with predominant IgA deposits in skin or proliferative glomerulonephritis with predominant IgA deposits in mesangium on kidney biopsy.

Treatment 1. Joint symptoms only: Rest, analgesia 2. GIT manifestations, orchites, marked subcutaneous edema ' steroids (prednisolone x 2-3 wks) 3. Severe Nephritis or nephritic syndrome, pulmonary hemorrhage 'steroids and cytotoxic drugs. 11. Ans. is b. Oral phosphate and Vit D supplements (Ref: Paediatric Nepthrology 5th, Pg. 324-335)

Approach to Refractory Rickets This case is a classical presentation of hypophosphatemic rickets (normal S. Calcium, low S. Phosphate, Normal S.PTH) and treatment of choice is phosphate and Vit D supplements. This is clearly a refractory rickets that has not responded to 2 doses of Vit D.

Remember • S. Phosphate high in chronic renal failure • S. Parathyroid hormone normal in hypophosphatemic rickets. Answers 91

Hyphosphosphatemic Rickets • Most common form of refractory rickets, inherited as a. X-linked dominant: Most common Mutations in the PHEX gene (chromosome Xp22.1) This gene is a regulator of FGF - 23 production by osteocytes. b. Autosomal dominant c. Autosomal recessive d. Autosomal recessive with hypercalciuria

Remember In hypophosphatemic rickets, symptoms of hypocalcemia (tetany, muscle weakness) are absent. Dental abnormalities are common in hypophosphatemic x linked rickets. 92 Pre-NEET Pediatrics

Treatment 1. Phosphate supplements (Joulie solution: Sodium phosphate, phosphoric acid) 2. Vitamin D supplementation (except hypercalciuric variety)

Vit D – Dependent Rickets (VDDR) • Autosomal recessive, present between 3 - 6 months of age. 1. VDDR type I (Pseudo Vit D deficiency) – Due to deficiency of enzyme 1  hydroxylase – Clinical and biochemical features resemble nutritional rickets but are not corrected by Vit D supplementation. – Treatment: Calcium, phosphate supplements, calcitriol 2. VDDR type II – Due to end organ resistance to 1,25 (OH)2 D3 – High prevalence of alopecia, ectodermal defects. – Refractory to treatment - Require large doses of calcitriol, calcium and phosphate.

Renal Tubular Acidosis • Fanconi syndrome, distal RTA present with refractory rickets. • Treatment: Bicarbonate and phosphate supplementation

Chronic Renal Failure Treatment: Restricting phosphate, calcium supplements, calcitriol. 12. Ans is d. Prophylactic antibiotics (Ref: Nelson 18th, Pg. 2228-2233)

Vesicoureteral Reflux Retrograde flow of urine from the bladder to the ureter and renal pelvis is called vesicuureteral reflux. Reflux is usually congenital and affects approximately 1% of children. Reflux predisposes to renal infection, reflux nephropathy (renal insufficiency).

Classification Based on appearance of urinary tract on a contrast voiding cyst- ourethrogram (VCUG) Answers 93

1. Grade I : reflux into a non dilated ureter 2. Grade II : reflux into the upper collecting system without dilatation 3. Grade III : reflux into dilated ureter 4. Grade IV : reflux into a grossly dilated ureter. 5. Grade V : massive reflux with ureteral tortuosity, loss of papillary impression. Clinical manifestation: Reflux usually is discovered during evaluation for a UTI. Among children with reflux, 80% are female.

Treatment With bladder growth and maturation there is a tendency for reflux to resolve over time especially lower grades. Grade 5 reflux rarely resolves. Surgical correction of VUR also advised for 1. Multiple recurrences of UTI while on prophylaxis 2. Non compliance to medical management 3. Parental preference for surgery 4. Appearance of new renal scars during medical therapy.

American Urological Association Recommendations for VUR.

Grade Age (yr) Scarring Initial Treatment

I - II Any +/- Antibiotic Prophylaxis III - IV 0 - 5 +/- Antibiotic Prophylaxis III - IV 6 - 10 +/- Unilateral Antibiotic prophylaxis Bilateral Surgery V <1 +/- Antibiotic Prophylaxis V 1 - 5 (-) Unilateral Antibiotics prophylaxis Bilateral Surgery V >1 yr (+) Surgery 13. Ans is c. Bartter syndrome (Ref: Paediatric Nephrology 5th, Pg. 318) 94 Pre-NEET Pediatrics

The boy in question has • Hypokalemia – S.K+ 3 meq/L • Hyponatremia – S.Na+ 125 meq/L • Alkalosis – PH – 7.46 • Hypochloremia – S.CI-88 meq/L Medullary Nephrocalcinosis – USG KUB This is a case of Bartter syndrome. Renal tubular acidosis can be easily ruled out as there is no acidosis but alkalosis. In diabetes Insipidus, there is deficiency of vasopressin, [(central) or tubular unresponsiveness to ADH (nephrogenic)]. Hence, there is loss of free water that leads to hypernatremia dehydration with corresponding increase in serum chloride. But in question there is hyponatremia and hypochloremia. In pseudohypo aldosteronism, S.aldosterone is normal or high but due to tubular unresponsiveness, clinical features of aldosterone deficiency ie. Hyperkalemia, hyponatremia, acidosis will be seen which is not the case in this situation. BARTTER SYNDROME It is an autosomal recessive disorder, characterized by hypokalemia, metabolic alkalosis, hyperreninemia and hyperaldosteronism, normal blood pressure and urinary wasting of K+, Na+ and Cl-. Clinical features include failure to thrive, polyuria and polydipsia with recurrent episodes of dehydration, muscle weakness and cramps in older children. Molecular defect is in the thick ascending limb of Henle. Tubular losses of K+, Na+, Cl- and water lead to volume contraction that in turn causes secondary hyperaldosteronism. Bartter syndrome should be differentiated from non renal causes of chloride loss such as vomiting, cystic fibrosis. In these conditions the urinary chloride concentration is in variably below 10meq/L. Therapy: Potassium supplementation and Indomethacin (cyclo oxy genase inhibitor decrease elevated prostaglandins that are responsible for polyuria). GITELMAN SYNDROME Hypokalemia, metabolic alkalosis and hypomagnesemia are chief abnormalities. The combination of Hypocalciuria with renal magnesium wasting distinguishes Gitelman from Bartter syndrome Answers 95

Clinical Features Generally milder symptoms. Appear in older children. There are episodes of muscular weakness, cramps, fatigue, vomiting, tetany, polyuria and growth retardation are mild. Molecular defect is in the apical thiazide sensitive, sodium chloride co-transporter (NCCT) in distal tubule. Features of Gitelman syndrome mimic Chronic thiazide administration. Treatment is with supplementation of potassium and magnesium chloride. 14. Ans is b. Long term phenytoin is required (Ref: Nelson 19th, Pg. 2017-18) This is a case of recurrent febrile seizure which can be treated with intermittent clobazam and paracetamol. The risk of epilepsy is 4%. Considering this to be a low intermediate risk, EEG and may be considered. But certainly long term phenytoin therapy is not required unless EEG/ Neuroimaging turn out to be abnormal.

Febrile Seizures Febrile Seizures are seizures that occur between the age of 6 months - 5 yrs with a temperature of 380C or higher, that are not the result of CNS infection or any metabolic imbalance and that occur in the absence of a history of prior afebrile seizures. A simple febrile seizure - generalized, usually tonic clonic associated with fever, lasting for a maximum of 15 min and not recurrent within a 24 hr period. A complex febrile seizure is more prolonged >15 min, is focal, and/ or recurs within 24 hours.

Risk Factors for Recurrence of Febrile Seizures

Major Minor Age < 1yr Family history of febrile seizures Duration of fever <24 hr Family history of Epilepsy Fever 38-390C Complex febrile seizure Male gender Lower S. Sodium 96 Pre-NEET Pediatrics

Risk Factors for Occurrence of Subsequent Epilepsy

Risk factor Risk for Epilepsy • Simple febrile seizure 1% • Neuro developmental abnormalities 33% • Focal complex partial seizure 29% • Family history of epilepsy 18% • Fever< 1hr before febrile seizure 11% • Recurrent febrile seizure 4%

Investigations is recommended in children <1yr of age after their first febrile seizure or if clinical signs and symptoms suggest meningitis

EEG/ Neuro Imaging Should be done in selected cases with a high/intermediate risk of epilepsy.

Treatment • Rectal Diazepam/ Midazolam/ given at time of Seizure. • Febrile - I/V phenobarbitone/ phenytoin/ valproate. • Intermittent oral Diazepam/ Clobazam/ Clonazepam can be given along with anti pyretics during febrile illness to reduce the risk of seizure. • Chronic antiepileptic therapy - may be considered for children with a high risk for later Epilepsy. 15. Ans is d. EEG in this condition shows a 4-6/ sec irregular spike and wave pattern, which is enhanced by photic stimulation. (Ref: Nelson 18th, Pg. 2463) This is a classical case presentation of juvenile (JANZ syndrome). It begins usually between ages of 12-16 yrs. Patients note frequent myoclonic jerks on awakening. A few years later, early morning GTCS develop in association with . EEG shows a 4-6/sec irregular spike and wave pattern, which is enhanced by photic stimulation. Drug of choice is Valproate and not carbemazepine. This is required lifelong. Discontinuation of drug causes a high rate of recurrence of seizures. Other types of Myoclonic are: 1. Benign myoclonus of infancy: EEG is normal, No are required. Answers 97

2. Typical myoclonic epilepsy of early childhood. a. Mean age of onset  2yrs b. At least 1/3 of children have a positive Family history of epilepsy c. Premorbid development is normal d. EEG shows fast spike wave complex  2.5 Hz and a normal back ground rhythm. 3. Complex Myoclonic Epilepsies: a. A history of hypoxic ischaemic encephalopathy in the perinatal period and abnormal neurological examination is a common finding. b. Focal or GTCS beginning in 1st yr of life antedate the onset of myoclonic epilepsy. c. It has a poor prognosis • Lennox Gastaut syndrome: Characterized by intractable seizures of various types, a slow spike wave EEG and mental retardation. • Landau Kleffner Syndrome: Characterized by receptive aphasia, seizures and behavioural problems. Valproic acid is the drug of choice. 16. Ans is b. Tuberous sclerosis (Ref: Nelson 18th, Pg. 2485, O.P. Ghai 7th, Pg. 564) There are 5 Major Neurocutaneous Syndromes i. Neurofibromatosis ii. Tuberous sclerosis iii. Struge Weber syndrome iv. Von Hippel Landau disease v. Ataxia telengiectasia

Neurofibromatosis Inheritance is autosomal dominant There are 2 types 1. Type I (Von Recklinghausen disease/ Peripheral NF1) 2. Type II (Central NF) – NF1: Deletion of NF gene on chromosome 17 – NF2: Gene is probably located on Chr. 22 – NF1: 2 or more of the following are present: 1. 6 or more Cafe-au-lait spots, each over 5 mm in diameter before puberty or over 15 mm diameter in older persons. 2. 2 or more neurofibromas or one plexiform neuroma 3. Axillary/ Inguinal freckling 98 Pre-NEET Pediatrics

4. Optic glioma 5. 2 or more Lisch nodules, dysplasia of sphenoid bone 6. A first degree relative with NF1 NF 2 - Presence of Bilateral auditory neuroma, unilateral auditory neuroma along with a first degree relative with meningioma, schwanomas or juvenile posterior subcapsular lenticular opacity.

Tuberous Sclerosis • Autosomal dominant inheritance • Cardinal features: Skin lesion, Convulsions and mental retardation • Skin lesions: Hypopigmented ash leaf shaped macules, adenoma sebacum (angiofibromas) shagreen patches, subungal fibroma In infancy, myoclonic jerks often occur (West syndrome) - Drug of choice Vigabatrine

Sturge - Weber - Syndrome Characterized by facial nevus flammeus (usually in the distribution of first branch of trigeminal nerve), contralateral focal seizures, calcification of cortex (tram track calcification), glaucoma on the same side.

Von Hippel Landau Disease There are retinal and cerebellar hemangioblastomas besides spinal cord angiomas and cystic tumors of pancreas, Kidneys and epididymis.

Ataxia - Telengiectasia • An autosomal recessive inherited disease (mapped to Chr. 11q) • Usually manifests with progressive cerebellar ataxia, Oculocutaneous telengiectasia, , pulmonary and sinus infections, immune deficiency and lymphorecticular malignancies. •  feto protein is almost always elevated. Neuro-imaging reveals cerebellar atrophy.

Treatment is Symptomatic

17. Ans is d. Medullobastoma (Ref: Nelson 18th, Pg. 2452, 2427) Options a,b,c are congenital conditions whose presentations vary as follows: Answers 99

1. Chiari Malformation • Type I: – Typically produces symptoms during adolescence /adult life. Patient complains of recurrent headache, urinary frequency and progressive lower limb spasticity. – The deformity consists of displacement of the cerebellar tonsils into the cervical canal. It is usually not associated with hydrocephalus. • Type II: – Characterized by progressive hydrocephalus with a myelomeningocele. – Usually presents in childhood with gait abnormality, spasticity and incoordination. The deformity consists of elongation of 4th ventricle with displacement of inferior vermis, pons and medulla into cervical canal.

2. Dandy Walker Malformation • Consists of a cystic expansion of the 4th ventricle in the posterior fossa and midline cerebellar hypoplasia • Approximately 90% patients have hydrocephalus. Most children have evidence of long tract signs, delayed motor and cognitive milestones, cerebellar ataxia.

3. Syringobulbia • Syringomyelia is a cystic cavity within the spinal cord that may/ may not communicate with the CSF pathways. • Syringobulbia exists when the cystic cavity extends into the medulla. • Communicating syringomyelia is frequently associated with Chiari type I malformation.

Clinical Feature It may rarely produces symptoms during childhood due to its slow evolution. Asymmetric loss of pain and temperature sensation with preservation of light touch can occur (due to disruption of lateral spinothalamic tracts) Later, upper motor signs develop as cavity impinges on the corticospinal tract. 100 Pre-NEET Pediatrics

4. Medulloblastoma • CNS tumors are the second most common malignancy in childhood after leukemias. – <1 yr: Supratentorial tumors predominate – 1-10 yr: Infratentorial tumors predominate – >10 yrs: Supratentorial tumors again predominate with diffuse astrocytomas most common. Medulloblastoma is a cerebellar tumor which occurs predominantly in males and at a median age of 5-7 yr of age.

Clinical Features 1. Increased ICP (ie. Headache, Nausea, Vomiting, Hypertension) 2. Cerebellar dysfunction (ataxia, poor balance, dysmetria) – Medulloblastoma is the childhood brain tumor that most commonly metastasizes extraneuronally. Medulloblastoma is sensitive to both chemotherapy and radiation theray with surgery as the starting point of treatment.

Other CNS Tumors a. Astrocytoma account for majority (40%) of CNS tumors. – Juvenile pilocytic astrocytoma (JPA) is the most common astrocytoma and with a good prognosis. b. Craniopharyngioma: occur in the suprasellar region – Clinical feature: Panhypopituitarism, growth failure, visual loss. Surgery is the primary treatment modality as they are minimally invasive. There is no role for chemotherapy in craniopharyngioma. 18. Ans is a. Give I/V bolus of normal saline (Ref: Nelson 19th, Pg. 213-215) This is a case of hypernatremic dehydration with shock and clinical signs of hypernatremia - Doughy skin irritability. Cause in this case may be improperly made ORS (more concentrated). Ina child with hypernatremic dehydration, as in any child dehydration, the first priority is restoration of intravascular volume with isotonic fucid. Normal saline is preferred to Ringers solution because the lower sodium concentration in the latter can cause the S.sodium to decrease too rapidly. Answers 101

Other Causes of Hypernatremia 1. Iatrogenic: Excess I/V hypertronic saline, I/V sodium bicarbonate 2. Hyperaldosteronism 3. Central and Nephrogenic Diabetes insipidus 4. GIT losses: diarrhea, Emesis 5. Renal losses: CRF, polyuric phase of ATN, Osmotic diuretics, Diabetes mellitus.

Clinical Features • Doughy abdomen • CNS feature: irritability, lethargy, seizures (due to brain hemorrhage). • Fever • Thrombotic complications (secondary to dehydration and hypercoagulability with hypernatremia)

Treatment 1. Restoration of Intravascular volume: Normal saline bolus. 2. Calculation of deficit and replacement over 24-48 hrs Goal- decrease S. Sodium by <12 meq/L every 24 hr, a rate of 0.5 meq/L/hr 3. Peritoneal dialysis: Acute severe hypernatremia 19. Ans is d. X-linked dominant (Ref: Nelson 19th, Pg. 388) It is an X linked dominant pattern of inheritance The features which suggest the X-linked dominant pattern are: • A single abnormal X Chromosome is sufficient to express the disease. • All of the female offsprings of a diseased male XY will receive the abnormal X chromosome and express the disease, whereas none of the male offspring to the diseased father will have the disease (as sons don't receive X chromosome from father). A diseased mother can transmit the abnormal X chromosome to both daughter and sons equally. 102 Pre-NEET Pediatrics

X-lined recessive inheritance

• Males are more commonly and severely affected than females. • Female carriers are generally unaffected. • Females carriers have a 25% risk of having an affected son, 25% risk for a carrier daughter and 50% chance of having an unaffected child. • Affected males will have only carrier daughters. • Affected males have no chance of having an affected son because they will pass their Y chromosome to their sons. Male to male transmission excludes X linkage, but is seen with autosomal dominant and Y-linked inheritance.

20. Ans is d. Do not give OPV and BCG at birth. (Ref: O.P. Ghai 7th, Pg. 207, Nelson 19th, Pg. 1158)

HIV in Newborn HIV-1 and HiV-2 are members of the Retroviridae family. HIV-2 is a rare cause of infection in children. It is most prevalent in Western and southern Africa. Transmission of HIV1 occurs via sexual contact, parenteral exposure to blood, or vertical transmission from mother to child. Vertical transmission can occur intrauterine (before birth), intrapartum during birth), or through breast feeding (after delivery) Answers 103

30-40% of infected newborns are infected in utero. The highest percentage of HIV infected children acquire the virus intrapartum. The risk of transmission through breast feeding in chronically infected women is approx 9-16% Elective Cesarean delivery decrease transmission by 87% if used in conjunction with Ziduvudine therapy in mother and infant. Transfusion of infected blood/blood products has accounted for 3- 6% of all pediatric AIDS cases. In the pediatric population, sexual transmission is infrequent.

Diagnosis All infants born to HIV infected mother test antibody positive at birth because of passive transfer of maternal HIV antibody across the placenta. Therefore HIV EIISA (based on antibody) is done at 18 months of age (By this time, almost all infants loose maternal antibody). Specific viral diagnostic assays, such as HIV DNA or RNA PCR, HIV Culture or HIV P24 antigen are essential for diagnosis of infant HIV infection. HIV DNA PCR is method for diagnosis. 2 positive HIV DNA PCR (1st performed at 6-8 wks of age) confirm HIV infection. HIV P24 antigen assay is the less sensitive, HIV culture is technically complex, expensive and time consuming (2-4 wks compared to 2-3 days with PCR) and with HIV RNA assays, data results are limited.

Prevention of Mother to Child transmission (MTCT)

Recommended Regime for pregnant (Not eligible for ART) Antepartum: Zidovudine starting at 28 wks of pregnancy Intrapartum: A combination of single dose Nevirapine, Zidovudine and Lamivudine Postpartum: A combination of Zidovudine and Lamivudine for 7 days. Recommended treatment for pregnant women in labor and who have not received ART is: Intrapartum: A combination of Nevirapine, Zidovudiine, Lamivudine. Postpartum: Combination of Zidovudine, Lamivudine x 7 wks. 104 Pre-NEET Pediatrics

Omission of Nevirapine for the mother may be considered for women who receive at least 4 wks of Zidovudine before delivery. Recommended regimen for Infants born to HIV positive mother • Nevirapine and Zidovudiine for 1 wk. • When delivery occurs within 2 hrs of women taking Nevirapine, the infant should receive Nevirapine immediately and Zidovudine for four wks. If mother receives less than 4 wks of antenatal ART, then 4 wks of Zidovudine is recommended.

Cotrimoxazole Prophylaxis 1. All HIV exposed infants starting at 4-6 wks of age and continued till HIV infection can be excluded. 2. All HIV infected infants <1yr should receive prophylaxis regardless of symptoms or CD4 percentage. 3. After 1 yr of age, prophylaxis is recommended for symptomatic children or children with CD4 <25%. Immunization: Symptomatic HIV infected children should not be given OPV and BCG. Breast feeding: According to WHO 2001, when replacement feeding is available, avoidance of breast feeding by HIV infected mothers is recommended. Otherwise, exclusive breast feeding is recommended keeping in mind that mixed feeding during this period carries a 70% greater risk of transmission.

21. Ans is b. Statement A is true while statement B is false. (Ref: O.P. Ghai 7th, P. 573) Statement 'A' is a classical example of (SMA) while statement 'B' is EMG finding of myasthenia gravis. 1. Infant has areflexia (absent reflexes) but Deep tendon reflexes are preserved in myasthenia gravis. 2. Myasthenia (congenital) does not present in the intrauterine life (so does not cause polyhydramnios) Hence, statement A is true but B is false. Answers 105

Floppy Infant He/she is an infant with marked hypotonia of the muscles and variable associated weakness. Common causes are: 1. CNS (Central ) – , neonatal encephalopathy, cerebral palsy (atonic type), Down syndrome, Inborn errors of metabolism (amino aciduria, mucopolysaccharidosis, cerebral lipidosis) 2. Spinal cord: Anterior horn cell disease: SMA, polio 3. Peripheral Nerves: Familial dysautonomia, hereditary motor sensory neuropathy 4. Myoneural junction: Neonatal myasthenia gravis, infantile botulism 5. Muscles: Muscualr dystrophies, congenital myopathies, polymyositis, glycogen storage disease 6. Miscellaneous : PEM, hypothyroidism, rickets, Prader willi syndrome, Ehler Danbos syndrome.

Clinical Features Delayed motor milestones frequent, respiratory infections. Antenatal H/O polyhydramnios, decreased fetal movements.

Clinical Examination Frog like postures, diminished resistance to passive movements, excessive joint mobility. 106 Pre-NEET Pediatrics atrophy Denervation pattern Normal Characteristic potentials, fasciculation's Abnormal CNS ± denervation potentials Decremental response on repetitive stimulation Short duration polyphasic potentials increased Decreased/ absent Normal Decreased distal or =or Absent Denervation Neurogenic Proximalweakness DTR EMG Muscle Biopsy Arms Legs weakness Vs Variable ++ +++ > Site ofinvolvement Central Extent of - + + = or > Normal/ Normal Normal Anteriorhorn cell Peripheral ±nerve - +++Neuro-muscular junction +++ ++ +++Muscle +++ > ++ < ++ = EMG - Electromyography NCS - Nerve conduction studies Differentiating Features of Floppy Infant According to the Site Involvement Answers 107

Spinal Muscular Atrophy SMA is an autosomal recessive disorder characterized by muscle weakness due to degeneration of motor in the spinal cord (anterior horn cells) and brainstem nuclei. It can be inherited as autosomal dominant, X-recessive or sporadic.

Types 1. SMA type I: Werdnig Hoffmann disease. A classical patient presents within first 6 months and is never able to sit. More than 90% die by 10 yr of age. 2. Type II: Manifest by first yr of age, unaided sitting is possible but walking is not achieved. More than 90% survive beyond 10 yrs. 3. Type III: Kugelberg-Welander disease, can achieve walking without aids. 4. Type IV: Presents after 30 yrs Severity is variable Lifespan is unaffected The heart is not involved in SMA. Intelligence is normal. 22. Ans is b. Emergency atrial septostomy (Ref: Avery 8th, Pg 841, Cloherty 5th, Pg. 428) This is a classical case presentation of hypoplastic left heart syndrome (ductal dependent lesion) which fails to respond to medical therapy. In these neonates, in order to benefit from a PGE1 infusion, there must be a patent foramen ovale to allow effective systemic blood flow (pulmonary venous return) to cross the atrial septum and enter the systemic vascular bed through the ductus. In these neonates, emergent balloon dilation of atrial septum may be necessary. After a period of stabilization, corrective surgery (Norwood procedure followed by a fontan operation later in childhood) is planned.

Ductus Arteriosus Dependent Lesions Ductus arteriosus is a ductal connection between aorta and pulmonary artery which plays an important role in the fetus shunting pulmonary blood flow to aorta. It is maintained patent by low oxygen and PGE1 during antenatal period. After birth, rising concentrations of oxygen and fall in PGE1 induce closure of ductus which is normally complete by 2-3 wks of age. 108 Pre-NEET Pediatrics

Closure of the Ductus Arteriosus is Particularly Deleterious in Patients with Ductal Dependent 1. Systemic blood flow (SBF) 2. Pulmonary blood flow (PBF) 3. Parellel circulation

1. Congenital lesions with duct dependent systemic blood flow. i. Critical aortic stenosis ii. Coarctation of aorta iii. Interruption of aortic arch iv. Hypoplastic left heart syndrome (HLHS) Of the four, critical aortic stenosis manifests shortly after birth with signs of CHF, gallop rhythm, murmur and poor perfusion. In other three left sided lesions, most common presentation is shock (poor perfusion) in first 1-2 wks of life.

2. Lesion with Duct Dependent Pulmonary Blood Flow a. Tricuspid atresia b. Ebstein anomaly c. Pulmonary atresia with intact ventricular septum ("Hypoplastic right heart syndrome") d. Severe pulmonary stenosis

3. Parellel Circulation/ Transposition of Great Arteries. Treatment Administration of PgE1 will open the ductus arteriosus and depending upon the lesion, increase pulmonary blood flow, systemic blood flow or intercirculatory mixing. Failure to respond or clinical deterioration after institution of PGE1 means 1. Initial diagnosis is incorrect Answers 109

2. Ductus is unresponsive to PGE1 (eg. Older infant) 3. Total anomalous pulmonary venous return with obstruction. 4. Lesion with obstruction of blood flow out of left atrium a. Hypoplastic left heart syndrome b. Variants of mitral atresia with a restrictive foramen ovale c. Transposition of great arteries with intact ventricular septum and restrictive foramen ovale. These 3 conditions require an emergent atrial septostomy to save the neonate (as explained before). 23. Ans is c. Knee-chest position, oxygen and Inj. Morphine administration (Ref: Nelson 19th, Pg. 1573-1575) This is a classical case presentation of a cyanotic spell due to most probable Tetrology of fallot and most appropriate intervention would be knee chest position, Inj. Morphine and Oxygen administration. In the absence of definite history of similar spells in past and echo study, this condition needs to be differentiated from breath holding spells which also peak around similar age (1 ½ - 2 yrs of age). The presence of respiratory distress and systolic murmur with cyanosis preceded by point toward crying a cyanotic spell. A breath holding spell is of 2 types. They are: i. Cyanotic a breath holding spell: a. Usually provoked by up sitting/ scolding an infant. b. Brief shrill cry followed by apnea and cyanosis that may be associated with generalised clonic jerks, opisthotonus c. Management is parental reassurance and not reinforce the child's behaviour after child's recovery from the spell. ii. Pallid spells - Much less common a. Typically initiated by a painful experience eg. Falling, striking head or sudden startle b. Child stops breathing, looses consciousness, becomes pale and may have a tonic seizure. 110 Pre-NEET Pediatrics

Treatment: Reassurance Atropine (by blocking the vagus nerve) may be tried in refractory cases.

Tetrology of Fallot (Must know) Pri mary defect is an anterior deviation of the infundibular septum (septum that separates aortic and pulmonary outflows) consequence of this deviation are 4 componnets: i. Obstruction to right ventricular outflow (pulmonary stenosis) ii. Malalignment type of Ventricular septal defect (VSD) iii. Dextroposition of aorta so that is overrides the ventricular septum iv. Right ventricular hypertrophy • Obstruction to pulmonary arterial blood flow is usually both infundibular (subpulmonic area) and the pulmonary value (Rare)

Clinical Manifestations 1. Acyanotic/pink tetralogy of fallot: when obstruction to right ventricular outflow is mild-moderate and a balanced shunt across the VSD, the patient is not cyanotic. 2. Ductal dependent pulmonary blood flow in infants. 3. Older children: presentation with cyanosis, clubbing and dyspnea on exertion.

Signs 1. Prominent left anterior hemi thorax in older children (due to long standing right ventricular hypertrophy) 2. A systolic thrill/ systolic murmur in left sternal 3rd - 4th parasternal space. It is caused by turbulence through right ventricular outflow tract. It loudness directly proportional to severity of pulmonary stenosis but it can become less prominent with severe obstruction and hypercyanotic spell. 3. Single 2nd heart sound or soft pulmonary component. Chest X-ray "Coeur in Sabot" (Boot shaped heart) due to cardiac apex elevation (due to right ventricle hypertrophy) pulmonary oligemia.

Treatment of Cyanotic Spell 1. Placement of child in knee chest position (It increases systemic vascular resistance and decrease venous return. This decreases right to left shunt, and improves symptoms). Answers 111

2. Oxygen administration 3. Morphine subcutaneous at dose not exceeding 0.2 mg/kg 4. Intravenous soda bicarbonate if metabolic acidosis present 5. Intravenous phenylephrine - increase systemic vascular resistance, decrease right ' left shunt and improves symptoms.

Surgical Management 1. Palliative surgery - Blalock - Taussig shunt (systemic to pulmonary artery shunt) performed to augment pulmonary artery flow. – Indication: Infants with less severe cyanosis without cyanotic spells and with good growth. Modified Blalock Taussig Shunt between subclavian artery and pulmonary artery. Waterson shunt between ascending aorta to main pulmonary artery. Pott's Shunt: descending aorta to pulmonary artery. 2. Corrective surgery a. Electively between 4-6 months of age in case of less severe cyanosis without spells b. Immediately in infants with severe cyanosis (marked right ventricular outflow obstruction). 24. Ans is d. Do urgent USG and prepare for urgent laparotomy (Ref: Nelson 19th, Pg. 1288-1289) This is a classical presentation of Intussusception and in view of X-ray features of peritoneal irritation (ascites with distended bowel loops), hydrostatic reduction should not be attempted. This is not a case of simple gastroenteritis because in enteritis, the pain is less severe, there is diarrhea which is not the case in this clinical problem. Similarly Vit K deficiency is less likely as bleeding per rectum is painless and there are no associated symptoms of excessive crying and vomiting in Vit K deficiency.

Intussusception It occurs when a portion of the alimentary tract is telescoped into an adjacent segment. The upper portion of bowel, the intussusceptions, invaginates into the lower, the intussuscipiens. 112 Pre-NEET Pediatrics

Q. It is the most common cause of intestinal obstruction between 3 month and 6 yr of age Q. the most common abdominal emergency in children <2 yr. Approximately 90% of cases are idiopathic. Swollen peyer patches, in response to GIT infection or introduction of new food proteins may lead to mucosal prolapse causing an intussusceptions. In 2-8% patients, recognizable lead points are found eg. Meckel diverticulum, polyp, neurofibroma, hemangioma, lymphoma Q. Intussusceptions are most often Ileocolic

Clinical Features Suddent onset in a previous well child of severe paroxysmal colicky pain or episodes of excessive crying. Child may initially be comfortable between paroxysms but progressively becomes weak and lethargic. Vomiting occurs in most cases stools of normal appearance with sometimes passage of blood. Eventually a shock like states with fever develop.

Classic Triad Pain, a palpable sausage shaped abdominal mass, and bloody or currant jelly stool is seen in <15% of patients.

Abdomen Palpation Usually (about 70% cases) reveals a slightly tender sausage shaped mass, presence of bloody mucus on rectal examination supports the diagnosis of Intussusception.

Diagnosis Ultrasound 3. Sensitivity 98-100% 4. Specificity 88% 5. Tubular mass in longitudinal views 6. A doughnut or target appearance in transverse images.

Management Immediate reduction of acute intussusceptions. In patients with prolonged intussusception and signs of shock, peritoneal irritation, intestinal Answers 113 perforation or pneumatosis intestinalis, hydrostatic reduction should not be attempted. In such cases, urgent laparotomy should be planned.

25. Ans is a. 21 hydroxylase deficiency To understand the pathophysiology and effects of different congenital adrenal hyperplasia, one should know the synthetic pathway of steroid hormones.

Horizontal Enzymes 3  3- hydroxystroid dehydrogenase 2  21 - hydroxylase 1  11 - hydroxylase

Vertical Enzymes All starting with "17" 4 - 17  hydroxylase 5 - 17, 20 lyase 6 - 17  hydroxy steroid dehydrogenase * 7 - Cholestrol desmolase * 8 - Aldosterone synthase 114 Pre-NEET Pediatrics

CAH (Congenital Adrenal Hyperplasia) CAH is a family of autosomal recessive disorders of cortisol biosynthesis. Cortisol deficiency increases ACTH (corticotrophin) which leads to Adreno cortical hyperplasia.

1. 21 - Hydroxylase Deficiency More than 90% of CAH are caused by 21-hydroxylase deficiency. Enzyme '2' in diagram is required for synthesis of Aldosterone and cortisol. In its absence, progesterone and metabolites are diverted to production of androgens. a. Deficiency of mineralo corticoid ' salt losing - , hyponatremia, hyperkalemia, acidosis b. Excess androgens - Virilization of female. In males, there will be precocioius puberty due to androgen excess.

2. 11 - Hydroxylase Deficiency Enzyme '1' in diagram is required for cortisol synthesis In its absence a. Excess DOC (Deoxy corticosterone) ' mineralo corticoid ' Hypertension b. Excess androgens ' Virilization of female, precocious puberty in males

3. 3  Hydroxyl Steroid Dehydrogenase Deficiency Enzyme '3' in diagram Required for Aldosterone, cortisol and androstendione synthesis In its absence a. cortisol/ Aldosterone not synthesized that lead to salt wasting crisis b.  DHEA  Virilization of females (weak androgen)

4. 17 - Hydroxylase Deficiency Enzyme '4' in diagram Required for cortisol, Sex hormone synthesis in its absence, all pregnenolone is converted to mineralocorticoid. So there will be: a. Excess of mineralocorticoid ' Salt retention, hypertension b. No androgens ' Feminization of external male genitalis, females development will be normal. Answers 115

Summary • In 21 OH, 3  OH dehydrogenase deficiency •  Salt wasting crisis + female virilization • In 11 hydroxylase, deficiency and 17 hydroxylase deficiency •  Hypertension present • 11  hydroxylase  female virilization (female pseudo hermaphroditism) • 17 hydroxylase  male external genitalia feminization (male pseudo hema phroditism) 26. Ans is c. A correct B wrong (Ref: Nelson 18th, Pg. 609) Feature of conjugated hyperbilirubenemia on the 3rd day accompanied by sepsis and positive reducing substance in urine suggest diagnosis of galactosemia. The term Galactosemia generally designates the deficiency of galactose - I phosphate uridyl transferase deficiency. Without the transferase enzyme, the infant is unable to metabolize galactose - I phosphate, the accumulation of which results in injury to kidney, liver and brain.

Clinical Manifestations In newborn or young infants, jaundice, hepatomegally, vomiting, hypoglycaemia, cataracts, sepsis (esp. E coli) are main clinical features. Symptoms improve when milk is temporarily withdrawn and replaced with lactose free nutrition.

Diagnosis Positive reducing substances in urine (Galactosuria) Direct enzyme assay using erythrocytes

Treatment Elimination of galactose from the diet reverses the growth failure, Cataracts, renal and hepatic dysfunction.

Galactokinase Deficiency The deficient enzyme is galactokinase, which catalyzes the phosphorylation of galactose. 116 Pre-NEET Pediatrics

The principal metabolites accumulated are galactose and galactitol. Cataracts are usually the sole manifestation.

Uridine Diphosphate Galactose 4 Epimerase Deficiency The principal metabolitis accumulated are galactose 1 phosphate, UDP - galactose. There are two distinct forms of epimerase deficiency 1. Mild form: affected persons are asymptomatic enzyme deficiency is limited to Leukocytes, erythrocytes. No treatment is required. 2. Severe form: clinical feature resemble transferase deficiency, with additional symptoms of hypotonia, nerve deafness. Patients can't synthesize galactose from glucose, hence they are placed on a galactose restricted diet. 27. Ans is b. A and B both correct and B is not explanation of A (Ref: Nelson 18th, Pg. 1672, O.P. Ghai 7th, Pg. 294) This is a case of neonatal cholestasis possibly due to congenital Rubella. But as proportion of EHBA (extra hepatic biliary atresia) patients show TORCH antibodies, liver biopsy is considered as the most accurate (90- 95%) method to differentiate EHBA from neonatal hepatitis. Neonatal cholestasis is defined as prolonged elevation of serum. Conjugated bilirubin beyond teh 1st 14 days of life. Cholestatic jaundice is always pathological and needs evaluation. Broadly there are 4 pathophysiological patterns of neonatal cholestasis: i. Obstructive - EHBA ii. Neonatal hepatitis iii. Paucity of bile ducts iv. Special categories (metabolic causes, total parental nutrition, alagille syndrome, Byler's disease)

Clinical Features Infants show conjugated Hyperbilirubenemia, yellow urine, clay colored stools. Infants with EHBA are often full term and look apparently healthy except for jaundice. Those with neonatal hepatitis are small for date and show increased association with infections and genetic abnormalities. Answers 117

Differentiation Between EHBA and Neonatal Hepatitis Children with EHBA should be identified before 2 months to prevent irreversible damage and for surgical intervention. The combination of a non-excreting HIDA scan along with GGTP levels more than 50 IU/L are highly suggestive of EHBA. Liver biopsy is considered as the most accurate (90 - 95%) diagnostic test to differentiate between the two. In biliary atresia, ductular proliferation and fibrosis are seen, whereas in neonatal hepatitis, there is alternation in lobular architecture, focal hepatocellular necrosis, bile ductules show little alteration. Giant cell transformation is found in either conditionand has no diagnostic specificity. Currently, peroperative Cholangiography remains the gold standard to differentiate between EHBA and neonatal hepatitis. Kasai's portoenterostomy is done for EHBA to re-establish the biliary flow into gut. Outcome is good if surgery is done before 2 moths of age. 28. Ans is a. Both active and passive immunization soon after birth (Ref: O.P. Ghai 7th, Pg. 193) These infants should receive both active and passive immunization at birth intramuscularly at different sites. Subsequent doses of active immunization is at 1-2 months and 6 months of age.

More About Hepatitis

Hepatitis Virus Incubations period HAV  15-45 days (mean 30 days) HBV  30-180 days HCV  15-160 days (mean 50 days) HDV  30-180 days HEV  14-60 days (mean 40 days)

Hepatitis B

Perinatal Transmission of HBV Most important risk factor that determines perinatal transmission ie. HBeAg. HBSAg. Carrier mother who are HBeAg positive almost invariably (>90%) transmit Hep B to their offspring, whereas carrier mothers with anti HBe rarely (10-15%) infect their offspring. 118 Pre-NEET Pediatrics

Most common time of perinatal transmission is at the time of delivery. Most of the infected neonates are asymptomatic carrier with increased chances of chronic hepatitis and hepato cellular carcinoma.

Common Seropatterns of Hep B. Infection

HBSAg Anti HBS Anti HBe HBe Ag Anti HBe Interpretation + - IgM + - Acute infection - - IgM +/- +/- Acute infection, anti BHC window + - IgG + - Chronic infection, high infectivity + - IgG - + Chronic infection, low infectivity - + IgG - +/- Recovery - + - - - Immunization 29. Ans is d. Decreased hematocrit (Ref: Nelson 19th, Pg. 1147, O.P. Ghai 7th, Pg. 196) Dengue Fever Dengue fever is caused by 4 antigenic types of dengue virus (member of family flaviviridae) spread through the bite of A.aegypti in Asia. It is characterized by biphasic fever, myalgia, arthralgia and rash. Answers 119

Clinical Features Typically, after an incubation period of 4-6 days the patients may develop abrupt onset high grade fever, facial fushing and headache. There may be vomiting, pain abdomen associated with tender hepatomegally. Nearly all patient have some hemorrhagic phenomenon. After subsidence of fever, some children may show varying degrees of peripheral circulatory falure • The presence of thrombocytopenia with concurrent hemo- concentration and evidence of plasma leak differentiate DHF from dengue fever. • Hemo concentration - hematocrit elevated at least 20% above baseline • Thrombocytopenia - <1Lakh cells/ mm3 • Plasma leakage - Presence of pleural effusion on chest x-ray or hypoalbuminemia • DHF with hypotension is called dengue shock syndrome.

Diagnostic Tests for Dengue Fever 1. Within first 5 days of onset of fever - Viral culture, Dengue NS1 antigen 2. After defervescence or in convalescent phase – Serology test – IgM - ELISA – IgG - Hemagglutination inhibition test

Treatment In hospital all children without hypotension should be given Ringer's lactate infusion at rate of 7 ml/kg over 1hr. This is tapered to 3 ml/kg/hr for 24-48 hrs if hematocrit decreases, and vital parameters improve. But if hemocrit is rising and vitals dont' show improvement, fluids infusion rate is increasd. (till 15 ml/kg/hr) In children with hypotension, Ringer lactate boluses are given. If hematocrit, and BP fall despite boluses, Blood is transfused. Majority of patients recover in 24-48 hrs without sequelae. 30. Ans is b. -Thalassemia (Ref: Nelson 18th, Pg. 2037) Out of the given 4 types of haemolytic anemia only -Thalassemia is able to give rise to such a severe presentation at birth. In all other conditions, the newborn is either normal or has mild jaundice. 120 Pre-NEET Pediatrics

Thalassemia Thalassemia are genetic disorders in globin chain production (Autosomal recessive). Adult haemoglobin is a tetramer, composed of 2  globin chains and 2  globin chains. •  Thalassemia is caused by deficient synthesis of  chain with normal  chain synthesis. •  Thalassemia is cause by deficient  chain with normal  chain synthesis • ° Thalassemia - complete absence of  chain • + Thalassemia - Partial reduction  chain • Most common type of genetic abnormality in  Thalessemia is point mutation ie. Nonsense and in  Thalessemia is deletion of  globin genes. • In  Thalassemia, excess of  globin. Chains form Bart's haemoglobin (4) infetal life and  chains for HbH (4) after birth. Barts haemoglobin in unable to release oxygen to tissues and hence, causes severe hypoxia and extravascular hemolysis and causes Non- immune Hydrops in the fetus.

 Thalassemia There are 3 forms of  Thalassemia 1. Thalassemia major (Cooley's anemia) • Homozygous for  Thalassemia genes. • Severe transfusion dependent – Manifesting at 6-9 months as haemoglobin synthesis switches from HbF to HbA. 2. Thalassemia Minor ( Thalassemia trait) • Individual has only one copy of  Thalassemia genes (Heterozygous) • Mildest form - patient are usually asymptomatic 3. Thalassemia intermedia • Condition intermediate between major and minor • Individuals may rarely need occasional transfusion

Hematological Findings in Thalassemia Peripheral Blood Smear • Anisocytosis (Variation in size of RBCs) • Poikilocytosis (variation in shape of RBCs) Answers 121

• Microcytic hypochromic RBCs • Target cells (Hemoglobin collects in centre of RBCs) • Basophilic stippling • Fragmented RBCs Others: MCV,  MCH,  MCHC S. Iron,  S. Ferritin,  Percentage Saturation,  HbA2,  HbF

T. Major T. Intermedia T. Minor (trait)

Genetics Homogygous Double heterozygotes Heterozygotes HbF 30-90% 20-100% 0-5% HbA2 <3.5% <3.5% 3.6-8%

AIIMS MAY 2012

1. Ans is b. Air bronchogram in chest x-ray (Ref: Nelson 19th ed. P. 581-590, O.P. Ghai 7th/ P. 143-144) Respiratory distress syndrome (Hyaline membrane disease)

Incidence Its incidence is inversely related to gestational age and birth weight. It occurs in 60-80% of infants <28 wk of gestational age, in 15-30% of those between 32 and 36 wk, and rarely in those >37 wk.

Etiology and Pathophysiology Surfactant deficiency (decreased production and secretion) is the primary cause of RDS. The major constituents of surfactant are dipalmitoyl phosphatidyl choline (lecithin), phosphatidyl glycerol, apoproteins (surfactant proteins SPA, SP-B, SP-C, SD-D), and cholesterol. These phospholipids are synthesized and stored in type II alveolar cells. They are released into alveoli, where they reduce surface tension and help prevent collapse of small air spaces at end expiration. Abnormalities in surfactant protein B and C genes are associated with severe and often lethal familial called neonatal alveolar proteinosis.

Diagnosis Age of onset: Presents usually within minutes of birth. 122 Pre-NEET Pediatrics

Chest X-ray Features of RDS Bilateral diffuse reticulo granular opacities in the pulmonary parenchyma. "Prominent air bronchograms" (aerated bronchioles superimposed on a background of collapsed alveoli) Conditions Associated with air Bronchogram • Pneumonia • ARDS • HMD (Hyaline membrane disease) • Pulmonary consolidation • Alveolar proteinosis • Sarcoidosis • Lymphoma Prevention Administration of antenatal corticosteroids reduces the incidence of RDS, IVH, NEC, early onset sepsis and developmental delay. Drug Regimens • Inj. Betamethasone 12 mg/ 1/m every 24 hours 2 doses (preferred) • Inj. Dexamethasone 6 mg 1/m every 12 hours, 4 doses. • Maximum effect is seen between 24 hours - 7days Treatment CPAP indication: If oxygen saturation cannot be kept >85% at inspired oxygen concentrations of 40-70% or greater.

Mechanical Ventilation Indication 1. Persistent apnea 2. Respiratory failure a. Arterial blood PH <7.2 b. Arterial blood PCO2  60mm Hg c. Oxygen saturation <85% at oxygen concentrations of 40-70% and CPAP 5-10 mm H2O. Surfactant Indications i. Moderate to severe RDS-Rescue treatment ii. All neonates less than 29 weeks irrespective of presence or absence of RDS - Prophylactic treatment. Answers 123

2. Ans is a. Shivering (Ref: O.P. Ghai 7th, Pg. 115-118, Meharban Singh 7th, Pg. 200-206)

Definitions • : Axillary temperature of baby < 36.50C – Cold stress - 36.0 36.40C – Moderate hypothermia - 32-35.90C – Severe hypothermia - <320C • Hyperthermia: Axillary temperature of baby >37.50C. • Thermoneutral environment: The narrow range of environmental temperature at which baby can maintain normal baby temperature with minimal oxygen consumption. This is also called as the zone of thermal comfort.

Response to Cold in Newborn Muscular activity (Shivering) is not a significant source of heat production. 124 Pre-NEET Pediatrics

Metabolic Thermogenesis Non-Shivering thermogenesis, as result of metabolism of brown fat, is the most important source of heat production in newborn. The fetal brown fat is laid down mostly during the third trimester of pregnancy and is located at the nape of the neck, interscapular region, axillae, groin, around the Kidneys and adrenals. This fat is metabolically very active in view of a large number of mitochondria and increased vascularity. Lipolysis of this fat release fatty acids which are locally consumed for generation of heat.

Reasons why Newborns have increased Susceptibility to Hypothermia • Large surface area of babies especially head as compared to their weight. • Limited heat generating mechanisms. In LBW babies: Decreased subcutaneous and brown fat, more permeable skin, even larger surface area than term babies and early exhaustion of metabolic stores like glucose. Prevention of hypothermia by institution of Warm Chain (10 steps) 1. Warm delivery room 2. Warm resuscitation 3. Immediate drying 4. Skin to skin contact called kangaroo mother care. 5. Breast feeding 6. Bathing postponed 7. Appropriate clothing 8. Mother and baby together 9. Professional alertness 10. Warm transportation.

Signs and Symptoms of Hypothermia • Acrocyanosis, Cool extremities, delayed capillary refill time (due to peripheral vaso constriction). • Poor weight gain (chronic hypothermia) • Cardio vascular manifestations: Bradycardia, hypotension, raised pulmonary pressure with resultant hypoxemia, tacypnea. • CNS depression - Lethargy, poor reflexes, apnea, decreased oral acceptance. Answers 125

• Abdominal distension, Vomiting, feeding intolerance. • Acidosis, Hypoglycemia, Azotemia, Oiguria, generalized bleeding.

Management Cold Stress/ Moderate Hypothermia • Skin to skin contact • Regular temperature monitoring • Supportive measures - Feeding, Monitor vitals.

Severe Hypothermia • Incubator/ Radiant warmer • Regular temperature monitoring Supportive measures: Oxygen, antibiotics (if sepsis suspected), saline bolus (if hypotension present), Prewarmed IV fluids, Inj. Vit K, Monitor Vitals. 3. Ans is c. All in children (Ref: O.P. Ghai 7th, Pg. 580-584, Nelson 19th, Pg. 1732-1737) The Leukemias are the most common malignant neoplasms in childhood, accounting for about 31% of all malignancies that occur in children <15 yrs of age. ALL (Acute Lymphoblastic Leukemia) accounts for about 77% of cases of childhood Leukemia. The Leukemias are a group of malignant diseases in which genetic abnormalities in a hematopoietic cell give rise to an unregulated clonal proliferation of cells.

Factors Predisposing to Childhood Leukemia Genetic Conditions • Down syndrome • Fanconi anemia • Bloom syndrome • Diamond - Blackfan anemia • Schwachman - diamond syndrome • Kostmann syndrome • Neurofibromatosis type I • Ataxia - telongiectasia • Severe combined immune deficiency 126 Pre-NEET Pediatrics

• Paroxysmal Noctural hemoglobinuria • Li- Fraumeni syndrome.

Environmental Factors • Ionizing radiation • Alkylating agents • Nitrosourea • Benzene exposure • Epipodophyllotoxin

Diagnosis Peripheral blood picture that indicates bone marrow failure -anemia, thrombocytopenia, elevated WBC count with/without leukemic cells (atypical lymphocytes). ALL is diagnosed by a bone marrow evaluation that demonstrates >25% of bone marrow cells as lymphoblasts.

Classification Based on

1. Morphology FAB criteria - L1, L2, L3 categories 2. Immunopheno type Precursor B cell, B cell or T cells Leukemia

Prognostic Factors

Good Prognosis Bad Prognosis

1. Children between 1-9 yrs Children <1yr, >9 yrs 2. Leukocyte count <50,000/mm3 Leukocyte count >50,000/mm3 3. Girls-Sex Boys - higher relapse rate 4. B-cell Leukemia T-cell Leukemia 5. Absence of mediastinal widening, Mediastinal mass or CNS disease CNS disease at diagnosis 6. Hyperdiploidy Hypodiploidy 7. Trisomy 4 and 10 Philadelphia positive t(9;22), t(4;11), t(8;14) in B cell ALL 8.  1000/mm3 blasts in peripheral  1000/mm3 blasts in PS following 7 blood following 7 days of days of prednisolone treatment and prednisolone treatment and intrathecal dose of methotrexate. intrathecal dose of methotrexate Answers 127

Management Treatment of ALL is divided into 4 stages i. Induction therapy (to attain remission) ii. CNS prophylaxis iii. Intensification (consolidation) iv. Maintenance therapy Average duration of treatment in ALL ranges between 2-2 ½ yrs. 1. Induction therapy: Lasts 4-6 weeks. Current induction regime includes vincristine, prednisolone, L-asparaginase and an anthracycline. 2. CNS prophylaxis: Essential to eradicate Leukemic cells which have passed the blood brain barrier. It comprises cranial irradiation with Intrathecal methotrexate. Other alternative regimen includes use of methotrexate, hydrocortisone, cytarabine. 3. Intensification therapy: It comprises intensified treatment after remission induction to tackle problem of drug resistance. Commonly used agents include methotrexate, L-asparaginase, Cyclophosphamide, Cytarabine and Epipodophyllotoxin. 4. Maintenance therapy: This is required to prevent relapse. It is continued for an additional 2-2.5 years after induction remission. The main agents used include 6-mercaptopurine daily and methotrexate once a week given orally

Relapse Early bone marrow relapse before completing maintenance therapy has the worst prognosis while late relapses after maintenance therapy have a better prognosis. Relapse in extramedullary sites especially testes is more favourable. The treatment of relapse must be more aggressive than the first line therapy. * Glioblastoma multiforme (grade IV) is an aggressive form of astrocytoma with poor prognosis. Pilocytic astrocytoma is with good prognosis. * Esophagealcarcinoma and Cholangio carcinoma usually remain asymptomatic for a very long time. By the time they are detected these malignancies would have already spread. So the only treatment that can be offered is palliation. 128 Pre-NEET Pediatrics

4. Ans is a. Factor X deficiency (Ref: O.P. Ghai 7th, Pg. 318-319, Nelson 19th, Pg. 1695-1708) To understand these disorders, we should know the normal pathway of coagulation.

Abnormal PT and aPTT Abnormal aPTT Abnormal PT Normal PT Normal aPTT Oral anticoagulants, Liver Hemophilia A and B, Anti-coagulant dysfunction Von willebrand disease therapy (eg: Vit K deficiency, DIC Warfarin) F Vll deficiency • Thrombin time: Prolonged thrombin time occurs with reduced fibrinogen levels (hypofibrogenemia or afibrogenemia). It measures the final step in clotting cascade, in which fibrinogen is converted to fibrin. • Bleeding time (BT): Assesses function of platelets and their interaction with the vascular wall. Prolonged BT may suggest decreased platelets, qualitative platelet defect, VWD or defect in vascular wall. Answers 129

• Deficiency of the contact factors (factor XII, Prekallikrein and HMW Kininogen) causes prolonged PTT but no bleeding symptoms. • Factor V deficiency: Autosomal recessive (parahemophilia) – Mucocutaneous bleeding, hematomas, severe menorrhagia are most common symptoms – Lab evaluation shows prolonged PTT and PT. • Factor X deficiency: It is rare autosomal disorder of variable severity. – A reduced factor X level is associated with prolongation of both PT and PTT. – In some cases of factor X deficiency, PT and aPTT can be normal also. Patients with factor X deficiency can present with umbilical stump bleeding at birth similar to factor XIII deficiency.

Factor XIII Deficiency PT and aPTT are normal. Test used in qualitative evaluation of factor XIII is evaluation of clot solubility in 5M urea.

Clinical Presentation • Umbilical stump bleeding • Poor wound healing • Infertility, abortion among affected females.

Von Willibrand Disease Patients with deficiency of Von Willibrand factor rarely present in neonatal period, because plasma concentration of the factor is high in neonates. Usually presents with mucosal bleeding, menorrhagia. VWF helps in platelet aggregation and also serves as the carrier protein for factor VIII. VWF is stored in platelet & granules and endothelial cell Weibel- palade bodies.

Lab Findings • Prolonged bleeding time, prolonged aPTT. • Normal platelet count (except 2B disease or platelet type disease) 130 Pre-NEET Pediatrics

Classification I. Type I - VWF is quantitatively reduced II. Type II - VWF is qualitatively abnormal III. Type III - VWF is absent

Treatment Desmopressin used in type I disease releases VWF from endothelial cells.

Bernard-Soulier Syndrome • Caused by absence or severe deficiency of VWF receptor (GPIb complex) on platelet membrane. • This is characterized by thrombocytopenia, giant platelets, prolonged BT, absent ristocetin induced platelet aggregation but normal aggregation to other agonists.

Glanzmann Thrombasthenia • Caused by deficiency of platelet fibrinoger receptor IIb/IIIa. • Characterized by prolonged BT, normal platelet count, normal sized platelets, abnormal aggregation with all agonists except restocetin.

ALL INDIA PG MEDICAL ENTRANCE EXAMINATION 2012

1. Ans is a. Cystic fibrosis (Ref: 19th, Pg. 1481-1497, O.P. Ghai 7th, Pg. 274-75) Cystic fibrosis is an inherited multisystem disorder with autosomal recessive transmission. The cystic fibrosis gene is on Chromosome 7 (7q13) and there are more than 1400 mutations at CF locus, of which Delta F508 (which devotes a single deletion at 508 position of protein) is the commonest. The mutation affects the gene's protein product, cystic fibrosis transmembrane regulator (CFTR) which functions as a and is expressed largely in epithelial cells of airways, the gastrointestinal tract, the sweat glands and the genito urinary system. Dysfunction of CFTR, the primary defect, leads to a wide and variable array of clinical manifestations and complications. For eg. Exocrine pancreatic insufficiency, severe chronic lung disease in children, pan sinusitis, nasal polyposis, Cholelithiasis and insulin dependent hyperglycemia. Answers 131

Pathogenesis CF epithelial cells especially the respiratory tract are unable to secrete chloride ions which leads to excessive sodium and water reabsorption. This leads to viscous desiccated secretions that cause airway obstruction. In sweat glands, the main function to absorb rather than secrete chloride not done, consequently sodium levels are elevated in sweat.

Diagnosis

• Presence of typical clinical features ( Respiratory, GIT or Genito urinary) or • History of CF in sibling or • Positive newborn screening test plus • Laboratory evidence for CFTR dysfunction: • 2 elevated sweat chloride concentrations obtained on separate days or • Identification of 2 CF mutations or • An abnormal nasal potential difference measurement. • More than 60 meq/L of chloride in sweat is diagnostic of CF

Management 1. Respiratory: Airway clearance techniques, antibiotics and anti- inflammatory agents. 2. Nutritional: Increasing caloric intake, supplement fat soluble vitamins (A, D, E in twice the recommended doses), replace pancreatic enzymes.

2. Ans is a. Autism (Ref: Nelson 19th, Pg. 101) 132 Pre-NEET Pediatrics

The pervasive developmental disorders (PDD) are disturbance of brain development with genetic basis PDD includes

Autism Asperger's Rett's syndrome Childhood PDD - NOS syndrome disintegrative (not otherwise disorder specified) • Delayed Similar to Almost, always Clinically Features of 1 of and autism affects girls significant the other disorde except in skills regression autism redcom- language between 6-18 in skills spectrum muni- skills months of age. bowel disorders, cation relatively and bladder but insufficient • Atypical intact. control, for a specific social Usually play motor diagnosis intera- not skills) before ction cognitively 10 yrs • Restri- delayed. of age cted range of interests • Onset before 3 yr of age

Autistic Disorder Diagnosis

DSM IV criteria for Autistic disorder (A) A total of six (or more) items from (1), (2), and (3) with at least 2 from (1) and one each from (2) and (3) (1) Qualitative impairment in social interaction, as manifested by least 2 of the following: a. Marked impairment in the use of non-verbal behaviours like eye – eye gaze, facial expression, body postures, and gestures to regulate social interaction. b. Failure to develop peer relationships approximate to developmental level. Answers 133 c. A lack of spontaneous seeking to share enjoyment, interests, or achievements with other people d. Lack of social or emotional reciprocity.

(2) Qualitative impairments in communication as manifested by at least one of the following: a. Delay in, or total lack of, the development of spoken language b. In individuals with adequate speech, marked impairment in ability to initiate or sustain a conversation with others c. Stereo typed and repetitive use of language or idiosyncratic language d. Lack of varied, spontaneous make believe play or social initative lay appropriate to developmental level. (3) Restricted repetitive and Stereo typed patterns of behaviour, and activities, as manifested by lest one of the following: a. Encompassing preoccupation with one or more stereo typed and restricted patterns of interest that is abnormal either in intensity or focus b. Apparently inflexible adherence to specific non functional routines or rituals c. Stereo typed and repetitive motor manners d. Persistent preoccupation with parts of objects (B). Delays or abnormal functioning in at least one of following areas, with onset prior to 3 yrs. 1. Social interaction 2. Language as used in social communication or 3. Symbolic or imaginative play. (C) The disturbance is not better accounted for by Rett’s disorder or childhood disintegrative disorder.

3. Ans is b. Increased EFA (essential fatty acids) (Ref. Avory 8th, Pg. 1061, Cloherty 5th, Pg. 115) The Caloric value of 20% lipid emulsion is 2 kcal/mL. The use of 20% emulsion is preferred over 10% because the higher ratio of phospholipids to triglyceride in the 10% emulsion interferes with triglyceride clearance. Twenty percent emulsion also provide a more concentrated source of calories. For these reason, 20% lipid emulsions are preferred. 134 Pre-NEET Pediatrics

Parenteral Nutrition (PN) Indications 1. Infants with b.wt  1500 gm 2. Infants with b.wt of 1501 - 1800 gm for whom significant enteral nutrition is not expected for >3 days 3. Infants with b.wt >1800 gm for whom significant enteral intake is not expected for >5 days. Goal of PN is to provide aminoacids to prevent negative energy and nitrogen balance. Goal therafter includes promotion of appropriate weight gain and growth, while awating the attainment of adequate enteral intake. i. Carbohydrate - Dextrose is the carbohydrate source in I/V solutions. – Calorie value of dextrose is 3.4 kcal/gm. Dextrose concentration via peripheral line is limited to  12.5% and upto 25% dextrose for central venous infusions. – Infusion rates above 11-14 mg/kg/min may exceed infants oxidative capacity and are generally not recommended ii. Protein: Crystalline amino acid solutions provide the nitrogen source in PN. – The calorie value of amino acids is 4 kcal/gm. Infusion rates are generally initiated at 1gm/kg/dl to a target of 3.5 gm/kg/dl for neonates >1500 gm at birth. iii. Lipid: Soybean oil or a combination of soybean and safflower oil provide for I/V fat emulsions. – Infusion rates are generally initiated at 0.5 - 1gm/kg/dl within 24 - 48 hours of life to a target of 3.0 gm/kg/day. – Use of 20% emulsions is preferred (explained above). * Electrolytes, Vitamins and Minerals are added as per requirement.

Complications of PN 1. Cholestasis: as a result of hepatic dysfunction (Reduced bile flow and bile salt formation). 2. Metabolic bone disease 3. Azotemia, hyperammonemia, hyperchloremic metabolic acidosis seen especially with amino acid intakes >4 gm/kg/dl. 4. Sepsis: associated with decreased Lipoprotein lipase activity. During sepsis, lipid infusion is limited to 2 gm/kg/dl if triglyceride> 150 mg/ dL Answers 135

5. Hyperlipidemia/hypertriglyceridemia 6. Indirect hyperbilirubinemis - due to displacement of bilirubin from albumin binding sites by free fatty acids. 4. Ans is d. Sacrococcygeal teratoma (Ref: Avery 8th, Pg. 1454) Sacrococcygeal Teratoma Sacrococcygeal teratoma is the most common solid tumor in newborn, although it is rarely malignant. Females are affected 2-4 times more frequently than males. In most cases, the tumor manifests as a mass protruding between the coccyx and rectum. Most benign teratomas produce no functional difficulties. Bowel/bladder dysfunction, painful defection and vascular/ lymphatic obstruction suggest that the lesion is malignant. Treatment It is primarily surgical if age adjusted AFP and HCG levels are normal. Chemotherapy (using Cisplatin and/ Bleomycin) + Surgery are used for malignant sacrococcygeal tumors. Other Infant Tumors • The most common intrarenal neoplasm manifesting at birth is congenital mesoblastic nephroms followed by Wilm's tumor. • Wilms tumor or nephroblastoma, is the most common intra abdominal tumor of childhood but is relatively rare in the neonatal period.

Congenital Leukemia Rarely occurs during the 1st month of life. Most of neonatal cases arise from the myeloid lineage. Trisomy 21 is particularly associated with AML in newborns. The chemotherapy regimens used in infants with myeloid leukemia are identical to those used in older children.

Transient Myelo-proliferative Disorder (TMD) • Associated with Down syndrome • Down's syndrome and TMD have a 30% chance of developing leukemia latter in life. 136 Pre-NEET Pediatrics

• TMD is a clonal disorder manifested by Hepatosplenomegally and circulating myeloblasts with/without anemia thrombocytopenia • Cytogenetic analysis often is normal. • In majority of cases, spontaneous resolution occurs.

CNS Tumours CNS malignancies are rare in newborn. Most of brain tumors are supratentorial half of them are gliomas. Infants with primitive neuroectodermal tumors and ependymoma have poor prognosis.

Hepatoblastoma Uncommon in infants and children. The most common malignant neoplasm involving the liver in infancy is "Metastatic neuroblastoma". The most common benign hepatic neoplasm in neonate are mesenchymal hamartomas and hemangiomas. Hereditary conditions with associated tumors 1. Ataxia - telangiectasia - Leukemia, lymphoma 2. Beckwith - Wiedemann syndrome - Wilm's tumor, hepatoblastoma, Rhabdomyosarcoma 3. Bloom syndrome - Leukemia 4. Denys - Drash syndrome - Familial Wilms tumor 5. Fanconi anemia - Leukemia 6. Klinefelter syndrome - Teratoma, leukemia, Breast cancers 7. Li-fraumeni syndrome - Sarcoma, CNS, Breast tumors 8. Neurofibroatosis - Glioma, leukemia, sarcoma 9. Trisomy 18 - Wilms tumor 10. Von hippel landau syndrome - Non Hodgkin lymphoma 11. WAGR syndrome - Wilms tumor 12. X-linked lymphoproliferative disorder - EBV lymphomas 5. Ans is a. Normal saline (Ref: O.P. Ghai 7th, Pg. 417, Nelson 19th, Pg. 1998)

Neural Tube Defects (NTDs) NTDs account for the largest proportion off congenital anomalies of the CNS and result from failure of neural tube to close spontaneously between the 3rd and 4th wk of in utero development. Many factors, including hyperthermia, valproic acid, malnutrition, chemicals, diabetes (maternal) and genetic determinants (mutations in Answers 137 folate responsive or folate, dependent enzyme pathways) can affect normal development of the CNS. The major NTDs include spina bifida occulta, meningocele, myelomeningocele, encephalocele, anencephaly, caudal regression syndrome, dermal sinus, syringomyelia, diastematomyelia, and lipoma involving the conus medullaris. Prenatal screening of maternal serum for AFP in the 16th - 18th wk of gestation identifies foetuses with NIDs in Utero. Spina bifida occulta is a common anomaly consisting of a midline defect of vertebral bodies without protrusion of the spinal or meninges. Most patients are asymptomatic. Meningocele is formed when the meninges herniate through a defect in the posterior vertebral arches or the anterior sacrum. It is often associated with Arnold-Chiari malformation and hydrocephalus is either present at birth or appears later. Myelomeningocele is formed when spinal cord and meninges herniate through a defect in the posterior vertebral arches. It represents the most severe form of dysraphsim.

Prevention • Dose for primary prevention is 0.4 mg folate per day. • A mother who has previously delivered a child with NTD should receive 4 mg per day of folic acid in subsequent . Duration of supplementation is 2 months before and 3 months after conception. • Hydrocephalus in association with a type II Chiari malformation develops in at least 80% of patients with myelomeningocele. Generally, the lower the deformity the less likely is the risk of hydrocephalus.

Treatment The defects which are covered by skin do not need urgent treatment but others should be closed soon after birth because they are likely to get infected. The Sac should be kept covered with a Sterile Saline - moistened gauze sponge to prevent infection and fluid loss. After repair of a myelomeningocele, most infants require a shunting procedure for hydrocephalus. 138 Pre-NEET Pediatrics

6. Ans is b. Small ASD (Ref: Nelson 19th, Pg. 1551-1553)

Atrial Septal Defect (ASD) ASD can occur in any portion of the atrial septum i. Ostium Secundum: defect in region of fossa ovalis ii. Ostium primum: defect is situated in the lower portion of atrial septum iii. Sinus Venosus ASD: defect in upper part of atrial septum in close relation to entry of the superior vena cava. Majority of the cases are sporadic. Autosomal dominant inheritance occurs as part of Holt-Oram syndrome (hypoplastic/absent radii 1st degree heart block, ASD) or in families with secundum ASD heart block. – An isolated valve: incompetent patent foramen ovale (PFO) is usually of no hemodynamic significance and is not considered an ASD. – Device closure of PFO is considered in young adults with history of thromboembolic as it may be a risk for right to left (paradoxical)systemic embolization.

Clinical Manifestations A child with an ASD is most often asymptomatic However, a history of exercise intolerance, easy fatigability and recurrent pneumonias accentuated may be obtained with large left to right shunts. Auscultatory signs include a normal or accentuated 1st Heart sound; wide, fixed splitting of the 2nd sound a pulmonary systolic ejection murmur (due to increased flow of blood across right ventricular outflow tract into pulmonary artery), a short mid diastolic murmur produced by increased volume of blood flow across tricuspid valve. The chest X-ray shows varying degree of enlargement of the right ventricle and atrium, depending on the size of the shunt. The pulmonary artery is enlarged and pulmonary vascularity is increased.

Treatment Surgical closure usually after 1st year and before entry into school. Answers 139

Indications i. All symptomatic patients ii. Asymptomatic patients with pulmonary: Systemic blood food (Qp, Qs ratio) of at least 2:1 or those with right ventricular enlargement.

Complications Infective endocarditis is extremely rare and antibiotic prophylaxis is not recommended. * Partial anomalous pulmonary venous return (one or several pulmonary veins returning anomalously to superior or inferior vena cava, right atrium or the coronary (sinus)may be associated with ASD (mostly of sinus venosus type) * Scimitar syndrome: an anomalous pulmonary vein draining into the inferior vena cava visible on CXR as a crescentic shadow of vascular density along right border of the cardiac silhouette. 7. Ans is a. Can be seen after ventouse delivery (Ref: Meharban Singh 7th, Pg. 254-272, Nelson 19th, Pg. 602-613) Jaundice is observed during the 1st wk of life in approximately 60% of term infants and 80% of preterm infants. Although bilirubin may have a physiologic role as an antioxidant, elevations of indirect, unconjugated bilirubin are potentially neurotoxic. The conjugated form is not neurotoxic, direct hyperbilirubinemia indicates a potentially serious hepatic disorders or a systemic illness.

Physiological Jaundice In term babies, it appears after 36 hrs of age. Maximum intensity is seen on the 4th day, S. Bilirubin does not exceed 15 mg/dL and jaundice disappears by 10 days of life. In preterm babies, the maximum intensity of jaundice is reached on the 5th or 6th day, S. Bilirubin may go upto 15 mg/dL and it may persist upto 14 days. The etiology of physiologic jaundice appears to be over production due to polycythemia, poor hepatic uptake, conjugation and excretion of bilirubin. Pathological jaundice: when jaundice in the newborn does not conform to criteria described for physiological jaundice, it is designated as pathological. or significant bruising attributable to ventouse delivery is a major risk factor for development of pathological 140 Pre-NEET Pediatrics hyperbilirubenemia as it increases the load of bilirubin to be metabolized by the liver.

Jaundice Associated with Breast Feeding 1. Breast feeding jaundice: is due to insufficient Breast feeding that leads to increased enterohepatic circulation. It is of early onset, which occurs in the 1st week of life. Hyperbilirubinemia (>12 mg/ dL) develops in 13% of breast fed infants in 1st wk of life. 2. Breast milk jaundice: develops in an estimates 2% of breast fed term infants after the 7th day of life with maximal concentrations as high as 10-30 mg/dL reached during the 2nd 3rd wk. – if breast feeding is continued, the levels will stay elevated and then fall slowly at 2 wks of age, returning to normal by 4 to 12 wks of age. – if breast feeding is stopped, the bilirubin level will fall rapidly in 48 hrs. – If nursing is then resumed, the bilirubin may rise 2-4 mg/dL but usually will not reach the previous level. Mothers with infants who have breast milk jaundice syndrome have a recurrence rate of 70% in future pregnancies. The etiology of breast milk jaundice may be attributed to the presence of glucuronidase in some breast milk.

Measures to Reduce S Bilirubin I. Phototherapy: Bilirubin absorbs light at 425-475 mm and is converted into: a. Water soluble form of bilirubin by photo-oxidation b. Water soluble E-isomers (25% total S Bilirubin) by photoisomerization c. Most efficient structural photoisomers called lumirubin which are readily excreted in bile, feces and urine. The narrow spectral blue light is most effective for phototherapy.

Bronze Baby Syndrome Infants with Parenchymal liver disease with biliary obstruction may develop bronze discolouration of skin due to excessive accumulation of lumirubin which is polymerized to bilifuscin on exposure to phototherapy. Answers 141

Exchange Blood Transfusion • It is most effective and reliable method to reduce bilirubin levels. • Early indications for exchange blood transfusion in infants with Rh- hemolytic disease of the newborn. a. Cord haemoglobin of <10 g/dL or hematocrit <30% b. Cord bilirubin >5 mg/dL c. Unconjugated S.bilirubin  10 mg/dL within 24hrs or rate of rise of >0.5 mg/dL/hr.

8. Ans is b. 10% Dextrose I.V.

Hypoglycemia in Newborn Definition is controversial, however most workers agree that blood glucose level of less than 40 mg/100mL (irrespective of period of gestation), if associated with symptoms of hypoglycaemia or if confirmed on repeat analysis in asymptomatic babies, is indicative of hypoglycaemia (1.0 mmd/L or glucose is equivalent to 18.02 mg/dL).

Classification of Neonatal Hypoglycemia 1. Fetal/ Neonatal hyperinsulinism – Maternal diabetes mellitus – Erythroblastosis fetalis – Wiedemann - Beckwith syndrome – Transposition of great vessels – Insulin producing tumors (nesidio blastosis, islet cell adenoma) – Maternal therapy with beta- sympathomimetics to colytic agents (Salbutamol, Terbutaline) 2. Decreased glycogen stores and/or increased utilization of glucose – Intrauterine growth retardation – Prematurity – Birth asphyxia – Hypothermia – Polycythemia – Septicemia 142 Pre-NEET Pediatrics

3. Miscellaneous Conditions – Inborn errors of metabolism (glycogen storage disease, galactosemia, fructose intolerance, CAH, cretinism) – Congenital Hypo pituitarism – Maternal therapy with  Blockers (propanolol), Chlorpropamide – Exchange transfusion with heparinised blood/ACD/CPD blood.

Management

• If baby requires >12 mg/kg/min glucose infusion or hypoglycaemia not resolved by day 7, start drugs like steroids, glucagon, diazoxide and investigate for resistant hypoglycaemia. (S. Cortisol, S. Insulin levels, CT scan pancreas, Screen for inborn errors of metabolism) Answers 143

9. Ans is a. S. Uric acid (Ref: Nelson 18th, Pg. 629-631)

Lesch-Nyhan Disease (LND) This is a rare X-linked disorder of purine metabolism that results from HPRT (hypoxanthine guanine PRPP transferase) deficiency. This enzyme is normally present in each cell in the body, but its highest concentration is in the brain, especially in the basal ganglia. Clinical manifestations include hyperuricemia, intellectual disability, dystonic including Choreoathetosis, Spasticity, dysarthric speech and compulsive self-biting, usually beginning with the teeth eruption.

Genetics The HPRT gene has been localized to the long arm of the X- Chromosome. The disorder appears in males, occurrence in females is extremely rare and ascribed to non-random inactivation of the normal X Chromosome. The mechanism whereby HPRT leads to the neurologic and behaviour symptoms is unknown but they are not caused by hyperuricemia or excess hypoxanthine as patients with partial HPRT deficiency do no self injure themselves.

Diagnosis The presence of dystonia along with self-mutilation of mouth and fingers suggests Lesch-Nyhan disease. Serum levels of uric acid >4-5 mg uric acid/dL and a urine uric acid, creatinine ratio of 3:4 or more are highly suggestive of HPRT deficiency. The definitive diagnosis requires an analysis of the HPRT enzyme.

Treatment Medical management of this disorder focuses on the prevention of renal failure by pharmacologic treatment of hyperuricemia with high fluid intake along with alkali and allopurinol. Bone marrow transplantation (BMT) has been carried out in several patients, based on the possibility that the CNS damage is produced by a circulating metabolic toxin. 144 Pre-NEET Pediatrics

10. Ans is a. 15 months (Ref: Nelson 19th, Pg. 31) Receptive language Precedes expressive language. By the time infants speak their first words around 12 months of age, they already respond appropriately to several simple statements, such as "no", "bye-bye", "give- me". By 15 months, the average child points to major body parts and uses 4-6 words spontaneously and correctly. Toddlers also enjoy polysyllabic jargoning but do not seem upset that no one understands. Most communication of wants and ideas continues to be nonverbal. 11. Ans is a. Shivering (Ref: Question AIIMS May 2012) 12. Ans is a. Deleted 21 (Ref: O.P. Ghai 7th, Pg. 613, Nelson 19th, Pg. 401-403)

Down Syndrome This is the most common disorder occurring with a frequency of 1:800 - 1:1000 newborns. Chromosome number 21 is present in triplicate, the origin of extra chromosome 21 is mostly from the mother (97%). The risk in newborn is 1:1550 if maternal age is between 15 and 29 years, 1:800 at 30-34 yrs, 1:270 at 35-39 yrs, 1:100 at 40-44 yrs and 1:50 after 45 yrs.

Cytogenetics In approximately 95% of the cases of Down syndrome there are 3 copies of Chromosome 21. Approximately 1% of Trisomy 21 are mosaics (with some cells having 46 Chromosomes). 4% have translocation that involves Chromosome 21. Majority of translocations are fusions at centromere between Chromosomes 13,14,15,21 and 22 called as Robert Sonian translocation. Translocation (21,21) Carriers have a 100% recurrence risk and other Robert Sonian translocations have a 5-7% recurrence risk when transmitted by females. It is not possible to distinguish the phenotypes of persons with full trisomy 21 and those with a translocation while patients who are mosaics tend to have a milder phenotype. Answers 145

Prenatal Diagnosis Initial Screening - PAPP-A, free hCG in first trimester Second trimester (Quad testing) - S.FP, hCG, Unconjugated estriol and inhibin A. Inhibin A, hCG (Both with 'h'spelling have high levels in Down's syndrome. Rest all are decreased. (i.e. PAPP-A, S.FP, Unconjugated estriol)

Ultrasound Findings • 1st trimester: Nuchal translucency and nasal bone • 2nd trimester: Increased nuchal fold thickness (measured over occiput), short femur and humerus length and duodenal atresia.

AIIMS NOVEMBER 2011

1. Ans. is c. Obstructive TAPVC (Ref: Park- Paediatric cardiology 5th, Pg. 164-165, Nelson 18th, Pg. 1856, 1922-1924) Functional closure of the ductus arteriosus occurs within 10-15 hours after birth by constriction of the medial smooth muscle in the ductus. Anatomic closure is completed by 2-3 wks of age by permanent changes in the endothelium and subintimal layers of the ductus. During fetal life, patency of the ductus appears to be maintained by the combined relaxant effects of low oxygen tension and endogenously produced Prostglandens esp PGE2. However, ductus of a premature infant is less responsive to oxygen, ever though its musculature is developed. PGE2 maintains the patency of ductus arteriosus so it is useful in heart lesions where the patency of ductus is essential to maintain the blood flow in either Aorta or Pulmonary artery (since it is the connection of two). If a cyanotic congenital heart defect or a ductus dependent cardiac defect (e.g: Pulmonary atresia with or without VSD, Trucuspid atresia, Hypoplastic left heart syndrome, interrupted aortic arch. Severe CoA, Transposition of great Vessels) is suspected or confirmed, a PGE, intravenous infusion should be started.

TAPVR (Total Anomalous Pulmonary Venous Return) In TAPVR, the heart has no direct pulmonary venous connection into the left atrium. 146 Pre-NEET Pediatrics

Types of TAPVR: Pulmonary veins may drain into 1. Superior Vena Cava - Supracardiac (50%) 2. Coronary Sinus, Right atrium - Cardiac (25%) 3. Inferior Vena Cava - Infracardiac (20%) 4. Mixed (5%) The manifestations of TAPVR depend on the presence or absence of obstruction of the venous Channels. If pulmonary venous return is obstructed, severe pulmonary congestion and pulmonary hypertension develop. Obstructed TAPVR is a Pediatric cardiac surgical emergency because prostaglandin therapy is usually not effective. Chest X-ray of supracardiac TAPVR shows a "snowman" appearance. This is however not useful in early infancy because of the thymus. 2. Ans is a. VDRL for mother and baby (Ref: Nelson 19th, Pg. 1016-1022) Congenital syphilis results from transplancental transmission of spirochetes. Transmission can occur at any stage of pregnancy. Untreated syphilis during pregnancy has a vertical transmission rate approaching 100% Fetal or Perinatal death occurs in 40% of affected infants. In congenital syphilis, manifestations have been divided into early and late stages. I. Early signs: appear during first 2 yr of life. They result from transplacental spirochetemia and are analogous to secondary stage of acquired syphilis. Symptoms/ Signs include: a. Hepato splenomegally b. Lymphadenopathy c. Coombs negative haemolytic anemia d. Thrombocytopenia e. Osteochondritis, periostitis f. Mucocutaneous rash - Mucous patches g. Persistent rhinitis (snuffles) h. Condylomatous lesions i. X-ray features: Wimberger's lines (metaphyseal demineralization of medial aspect of proximal tibia), osteochondritis (painful, resulting in Parrot's Pseudo paralysis) j. CNS abnormalities, failure to thrive, chorioretinitis, Nephritis and Nephritic syndrome. Answers 147

II. Late Manifestation: appear gradually after 2 yrs of age. These result primarily from chronic granulomatous of bone, teeth and CNS. These are: Olympian brow, Higoumenaki sign (Unilateral or bilateral thickening of sternoclavicular third of the clavicle), Saber Shin's (anterior bowing of tibial midportion), Hutchinson teeth (Peg shaped upper central incisors), Saddle nose (Depression of nasal root), Hutchinson triad (Hutchinson teeth, interstitial keratitis, and 8th nerve deafness), Clutton joints (unilateral or bilateral painless joint swelling).

Diagnosis 1. VDRL (Venereal disease research laboratory) tests are sensitive non- treponemal tests that detect antibodies against phospholipid antigens on the treponema surface that cross react within the mammalian cardiolipin - lecithin - cholesterol antigens. Titres increase with active disease and decline with adequate treatment. – False positive VDRL: Infectious mononucleosis, connective tissue disease, pregnancy. – False negative VDRL: Prozone effect (excess antibody), early primary syphilis, latent syphilis, late congenital syphilis). 2. Treponemal antibody tests: TPHA (T. Pallidum hemagglutination assay), FTA-abs (Fluorescent treponemal antibody absorption) test. These tests become positive soon after initial infection and usually remain positive for life, even with adequate therapy. – They are useful for diagnosis of a 1st episode of syphilis and for distinguishing false positive result of VDRL but cannot accurately identify length of time of infection, response to therapy or reinfection.

Treatment Congenital syphilis: Aqueous crystalline penicillin I/V for 10 days. 3. Ans is c. Vein of galen malformation (Ref: Nelson 18th, Pg. 1988, 2511) Arterio venous malformations result from failure of normal capillary bed development between arteries and veins during embryogenesis. AV malformations produce abnormal shunting of blood, causing vessel expansion, a space occupying effect or rupture and intracerebral bleeding. The vein of galen is an arterio venous malformation that is located under cerebral hemisphere and drains the anterior and central regions of the brain into the sinuses of posterior cerebral fossa. 148 Pre-NEET Pediatrics

Auscultation of skull is positive for a high pitched bruit. USG demonstrate a hypoechoic mass located posterior to the third ventricle with pulsatile flow that helps in differentiating it from other midline cystic lesions. Vein of galen malformation can cause high output congestive heart failure secondary to shunting of large volumes of blood or progressive hydrocephalus secondary to obstruction of the CSF pathways. Vein of galen malformations are difficult to treat (surgery or occlusive therapy) and are associated with a poor prognosis. 4. Ans is a. Kartagener's syndrome (Ref: Nelson 18th, Pg. 1817-1819) Primary ciliary dyskinesis (PCD) comprises those respiratory disorders having malfunction of airway cilia that lead to repeated and chronic lung and sinus infections. About 50% of patients with PCD have Kartagener syndrome: • Situs inversus • Chronic sinusitis and otitis • Airway disease leading to bronchectasis Approximately 25% patients with situs inversus have PCD, the presence of situs inversus does not establish the presence of PCD. In classical PCD, there is absence of both inner and outer dynein arms. Partial dynein arm defects or defects in central doublets are generally not associated with situs inversus.

Genetics Most PCD cases are autosomal recessive though there are reported cases of Autosomal dominant and X-linked modalities. Austosomal dominant and x-linked modalities. 5. Ans is b. Primary hypothyroidism (Ref: Nelson 19th, Pg. 1895-1903) Both choices A and C are similar - Both cause hyperthyroidism with a high T4 level. Choice D, TSH resistance is a very rare condition and many reported cases showed normal T4 level.

Hypothyroidism • Most cases of congenital hypothyroidism are not hereditary and result from thyroid dysgenesis (accouting for 80-85% cases). Most common dysgenesis is an ectopic gland. Thyroid peroxidase defects Answers 149

of organification and coupling are the most common of the T4 synthetic defects. • Pendred syndrome: an autosomal recessive disorder comprising sensineural deafness and goiter, also have impaired iodide organification and a positive perchlorate discharge. It is due to a mutation in the chloride-iodide transport protein common to the thyroid gland and the Cochlea. • The most common cause of acquired hypothyroidism is chronic lymphocytic (Hashi Moto's) Thyroiditis. • Maternal medications causing hypothyroidism are iodides, amiodarone, propylthiouracil, methimazole, Radioiodine. • Kocher-Debre Semelaign syndrome: Generalized muscular pseudohypertrophy esp. Boys in some affected patients with hypothyroidism of longer duration and severity.

Laboratory Findings • Newborn screening is done by heel prick between 2 and 5 days of life (Raised TSH, low T4 level). • Retardation of bone age at birth can be shown in about 60% of congenital hypothyroidism infants. The distal femoral epiphyses, normally present at birth is after absent. • The epiphysis often have multiple foci of ossification (epiphyseal dysgenesis), deformity ("beaking") of the 12th thoracic or 1st or 2nd lumbar vertebra is common. • Treatment: Levothyroxine is the treatment of choice • Neonates - Initial starting dose is 10-15 µg/kg/d • Children - 4 µg/kg/24 hr • Adults - 2 µg/kg/24 hr 6. Ans is b. Sickle cell anemia (Ref: Nelson 19th, Pg. 1662) • G6Pd and hereditary spherocytosis can cause episodic anemia with jaundice since birth. • Our choice here is sickle cell anemia as it never manifests in first 6 month of life. • Beta globin disorders such as sickle cell disease or beta-thalassemia major do not become apparent clinically until several months of age, when the switch from Haemoglobin F to haemoglobin A synthesis reveals the defect. • Moreover sickle cell anemia usually presents with anemia, fever (infections), acute splenic sequestration, dactylitis , stroke etc. 150 Pre-NEET Pediatrics

• While Paroxysmal nocturnal hemoglobinuria is a rare disorder of abnormal marrow stem cells that affects each blood cell lineage. It is an acquired disorder that is characterized by a defect in proteins of the cell membrane (including decay accelerating factor, the C8 binding protein) that renders RBCs and other cells susceptible to damage by plasma complement. The underlying defect involves the glycolipid anchor that maintains these protective proteins on the cell surface. • PIGA gene encodes for phosphatidyl glycan protein which is essential for synthesis of GPI (glycolipid anchor).

Clinical Manifestations 1. Nocturnal and morning hemoglobinuria is a classic finding in adults. However, chronic hemolysis (intravascular) is more common in PNH, despite its name. 2. Thrombocytopenia and leukopenia are after seen. 3. Thrombosis and thromboembolic phenomena are sometimes seen (due to altered glycoproteins on the platelet surface and resultant platelet activation). 4. Hypoplastic or a plastic pancytopenia may follow/precede the onset of PNH 5. PNH rarely progresses to acute myelogenous leukemia

Lab Findings 1. Acidified serum hemolysis (Ham) test or sucrose lysis test. These tests activate the alternative and classic pathways of complement lysis, respectively. 2. Flow cytometry is the diagnostic test of choice for PNH. It uses anti CD59 for RBCs and anti CD55, anti CD59 for granulocytes.

Treatment 1. Splenectomy is not indicated. 2. Glucocorticoids for acute hemolytic episodes 3. Prolonged anticoagulant therapy 4. Bone marrow transplantation 5. Eculizumab, monoclonal antibody against complement proteins C5, stabilized haemoglobin levels and decrease the rate of hemolysis. Previous Year’s Questions of DNB

Q U E S T I O N S PAEDIATRICS 2010

Q 1. Earliest indication of sexual maturation in a girl is: A. Menarche B. Pubarche C. Thelarche D. Maturation of breasts Ans. is C. Thelarche (Ref: O.P. Ghai 7th, Pg. 498) Q 2. Epiphyseal dysgenesis is a pathognomonic feature of: A. Hypoparathyroidism B. Hyperparathyroidism C. Hypothyroidism D. Hyperthyroidism Ans. is C. Hypothyroidism (Ref: Nelson 19th, Pg. 1899) Q 3. Commonest cause of stridor in a new born is: A. Laryngomalacia B. Foreign body C. Meconium aspiration D. Recurrent laryngeal nerve palsy due to birth Ans. is A. Laryngomalacia (Ref: O.P. Ghai 7th, Pg. 340) Q 4. Precocious puberty is seen in: A. Hyperthyroidism B. Addison’s disease 152 Pre-NEET Pediatrics

C. McCune Albright syndrome D. Neuroblastoma Ans. is C. McCune Albright syndrome (Ref: Nelson 19th, Pg. 1892) Q 5. The commonest cause of death in diphtheric child is: A. IIIrd nerve palsy C. Tonsilitis B. Myocarditis D. Septicemia Ans. is B. Myocarditis (Ref: IAP Textbook of Pediatrics 4th, Pg. 364) Q 6. Berger nephropathy is due to mesangiai deposition of: A. Fibrin and C3 B. IgD and C3 C. IgE and C3 D. IgA and C3

Ans. is D. IgA and C3 (Ref: O.P. Ghai 7th, Pg. 446) Q 7. Pawn ball megakaryocytes are characteristic of: A. Myelodysplastic syndrome B. Idiopathic thrombocytopenic purpura C. Thrombotic thrombocytopenic purpura D. Chloramphenicol toxicity Ans. is A. Myelodysplastic syndrome Q 8. Commonest cause of heart failure in infancy is: A. Myocarditis B. Rheumatic fever C. Cardiomyopathy D. Congenital heart disease Ans. is D. Congenital heart disease (Ref: O.P. Ghai 7th, Pg. 372) Q 9. A 3.5 kg baby born to diabetic mother develops seizures at 16 hours.The most likely cause is: A. Hypoglycemia B. Hypoxia/Respiratory distress syndrome Previous Year’s Questions of DNB 153

C. Hypomagnesiumia D. Hypocalcemia Ans. is A. Hypoglycemia (Ref: O.P. Ghai 7th, Pg. 156) Q 10. Which of the following is not used a in term baby as vigorous: A. Color B . Heart rate C. Respiratory effort D. Muscle tone Ans. is A. Color (Ref: O.P. Ghai 7th, Pg. 100) Q 11. Early neonatal sepsis in india is most commonly due to: A. Escherichia coli B . Group-B Stretococci C. Staphylococci D. Pseudomonas Ans. is A. Escherichia coli (Ref: O.P. Ghai 7th, Pg. 136) Q 12. A neonate with recurrent infection and abscess was diagnosed with Kostmann syndrome. Treament is: A. G-CSF B. GM-CSF C. Antithymocyte globulin+cyclosporin D. Antithymocyte globulin+cyclosporin +GM-CSF Ans. is A. G-CSF (Ref: Nelson 19th, Pg. 750) Q 13. Aniridia is associated with: A. Hepatoblastoma B. Medulloblastoma C. Nephroblastoma D. Retinoblastoma Ans. is C. Nephroblastoma (Ref: Nelson 19th, Pg. 1758) 154 Pre-NEET Pediatrics

Q 14. Which of the following is associated with >20% risk of chromosomal anomalies: A. Cleft lip B. Gastroschisis C. Omphalocele D. Spina bifida Ans. is C. Omphalocele (Ref: Avery 8th, Pg. 1114) Q 15. A premature baby weighing 1.5 kg, born with emergnancy C.S. at 32 weeks, now develops respiratory distress with grunting the best management would be: A. C-pap B. Mechanical ventilation C. Moist oxygen through headbox D. Surfactant therapy plus mechanical ventilation Ans. is D. Surfactant therapy plus mechanical ventilation (Ref: Nelson 19th, Pg. 584)

PAEDIATRICS 2009

Q 1. Commonest cause of heart failure in infancy is: A. Myocarditis B. Rheumatic fever C. Cardiomyopathy D. Congenital heart disease Ans is D. Congenital heart disease (Ref: O.P. Ghai 7th, Pg. 372) Q 2. Congenital long QT syndrome is associated with neonatal: A. Sinus bradycardia B. Sinus tachycardia C. Supra ventricular. tachycardia D. Ventricular tachycardia Ans. is A. Sinus bradycardia (Ref: Nelson 19th, Pg. 1617) Previous Year’s Questions of DNB 155

Q 3. Single gene defect causing multiple unrelated problems: A. Pleiotropism B. Pseudodominance C. D. Anticipation Ans. is A. Pleiotropism Q 4. A newborn baby presented with profuse bleeding from umbilical stump after birth. Probable diagnosis is: A. Factor XIII deficiency B. VWF deficiency C. Factor XII deficiency D. Glanzmann thrombosthenia Ans. is A. Factor XIII deficiency (Ref: Nelson 19th, Pg. 1704) Q 5. Common to both acute and chronic malnutrition is: A. Weight for age C. Height for age B. Weight for height D. BMI Ans. is A. Weight for age (Ref: O.P. Ghai 7th, Pg. 62) Q 6. Persistence of Moro’s reflex is abnormal beyond the age of: A. 3rd month B. 4th month C. 5th month D. 6th month Ans. is D. 6th month (Ref: Forfar 6th, Pg. 311) Q 7. Most common cause of renal artery stenosis in children in India is: A. Takayasu Aortoarteritis B. Fibromedial hypertrophy 156 Pre-NEET Pediatrics

C. Fibrointimal hyperplasia D. Polyarteritis Nodosa Ans. is A. Takayasu Aortoarteritis (Ref: Paediatric Nephrology 5th, Pg. 162) Q 8. Drug of choice for Rheumatic fever prophylaxis in penicillin allergic patient: A. Erythromycin B. Clindamycin C. Vancomycin D. Gentamycin Ans. is A. Erythromycin (Ref: Nelson 19th, Pg. 924) Q 9. Common to both acute and chronic malnutrition is: A. Weight for age B. Weight of height C. Height for age D. BMI Ans. is A. Weight for age (Ref: O.P. Ghai 7th, Pg. 62) Q 10. Essential criteria for TOF includes all, except: A. Valvular stenosis B. Infundibular stenosis C. Over riding of aorta D. RVH Ans. is A. Valvular stenosis (Ref: O.P. Ghai 7th, Pg. 408) Q 11. All of the following are features of down’s syndrome, except: A. Increased PAPPA B. Increased free beta HCG levels C. Absent nasal bone D. Abnormal ductus venous flow velocity Ans. is A. Increased PAPPA (Ref: IAP textbook of Pediatrics 4th, Pg. 995) Previous Year’s Questions of DNB 157

Q 12. Most common inherited childhood tumor is: A. Leukemia B. Neuroblastoma C. Retinoblastoma D. Wiulm’s tumor Ans. is C. Retinoblastoma Q 13. Turner syndrome is maximally associated with: A. Horseshoe kidney B. Coarctation of arota C. VSD D. ASD Ans. is B. Coarctation of arota Q 14. Most common sequelae to periventricular leukomalacia is: A. Spastic diplegia B. Spastic quadriplegia C. Hypotonia D. Mental retardation Ans. is A. Spastic diplegia (Ref: O.P. Ghai 7th, Pg. 559) Q 15. Neonate with recurrent infection and abscess diagnosed of kostmann syndrome. Treatment include: A. Anti thymocyte globulin + cyclosporin B. Anti thymocyte globulin + cyclosporin + gm-csf C. G-CSF D. GM-CSF Ans. is C. G-CSF (Ref: Nelson 19th, Pg. 750) Q 16. What constitutes Pentalogy of Fallot: A. TOF + PDA B. TOF + ASD C. TOF+ COA D. TOF + Polysplenia Ans. is B. TOF + ASD 158 Pre-NEET Pediatrics

Q 17. Most sensitive indicator of intravascular volume in infant is: A. Cardiac output B. Heart rate C. Stroke volume D. Preload Ans. is B. Heart rate (Ref: O.P. Ghai 7th, Pg. 697) Q 18. Chang staging is for: A. Retino blastoma B. Rhabdo myosarcoma C. Ewings sarcoma D. Medulloblastoma Ans. is D. Medulloblastoma (Ref: Forfar 6th, Pg. 1115) Q 19. Wilms tumour is associated with all except: A. Aniridia B. Hemihypertrophy C. Hypertension D. Bilateral polycystic Kidney Ans. is D. Bilateral polycystic Kidney (Ref: Nelson 19th, Pg. 1758)

PAEDIATRICS 2007

Q 1. Earliest indication of sexual maturation in a girl is: A. Menarche B. Pubarche C. Thelarche D. Maturation of breasts Ans. is C. Thelarche (Ref: O.P. Ghai 7th, Pg. 498) Q 2. The first permanent teeth to erupt are usually the: A. Lateral incisors B. Central incisors Previous Year’s Questions of DNB 159

C. Second molars D. First molars Ans. is D. First molars (Ref: O.P. Ghai 7th, Pg. 4) Q 3. Blood specimen for neonatal thyroid screening is obtained on: A. Cord blood B. 24 hours after birth C. 48 hours after birth D. 72 hours after birth Ans is C. 48 hours after birth (Ref: O.P. Ghai 7th, Pg. 483) Q 4. Treatment of choice for thalassemia major is: A. Blood transfusion and iron therapy B. Folic acid and desferrioxamine C. Blood transfusion and desferrioxamine D. Iron, blood transfusion and desferrioxamine Ans. is C. Blood transfusion and desferrioxamine (Ref: O.P.Ghai 7th, Pg. 309) Q 5. One of the intestinal enzymes that is generally deficient in children following an attack of severe infectious enteritis is: A. Lactase B. Trypsin C. Lipase D. Amylase Ans. is A. Lactase (Ref: O.P. Ghai 7th, Pg. 266) Q 6. Sure sign of CCF in a infant is: A. Basal crepts B. JVP C. Pedal oedema D. Liver enlargement Ans. is D. Liver enlargement (Ref: O.P. Ghai 7th, Pg. 375) 160 Pre-NEET Pediatrics

Q 7. Epiphyseal dysgenesis is a pathognomonic feature of: A. Hypoparathyroidism B. Hyperparathyroidism C. Hypothyroidism D. Hyperthyroidism Ans. is C. Hypothyroidism (Ref: Nelson 19th, Pg. 1899) Q 9. Commonest cause of stridor in a new born is: A. Laryngomalacia B. Foreign body C. Meconium aspiration D. Recurrent laryngeal nerve palsy due to birth Ans. is A. Laryngomalacia (Ref: O.P. Ghai 7th, Pg. 340) Q 9. Attainment of weight of a preschool normal child is: A. 2-2.5 Kg B. 3-3.5 Kg C. 4-4.5 Kg D. 5-5.5 Kg Ans. is B. 3-3.5 Kg (Ref: IAP textbook of Pediatrics 4th, Pg. 84) Q 10. Precocious puberty is seen in: A. Hyperthyroidism B. Addison’s disease C. McCune Albright syndrome D. Neuroblastoma Ans. is C. McCune Albright syndrome (Ref: Nelson 19th, Pg. 1892) Q 11. The commonest cause of death in diphtheric child is: A. IIIrd nerve palsy B. Myocarditis C. Tonsilitis D. Septicemia Ans. is B. Myocarditis (Ref: IAP textbook of Pediatrics 4th, Pg. 364) Previous Year’s Questions of DNB 161

Q 12. 15 months old child can do all, except: A. Feeds self with spoon B. Says three words C. Builds tower of 2 blocks D. Creeps upstairs Ans. is A. Feeds self with spoon (Ref: Nelson 19th, Pg 32) Q 13. Best method of diagnosis of childhood HIV: A. CD4 cell counts B. P24 antigen C. ELISA D. Anti HIV antibody Ans is B. P24 antigen (Ref: Nelson 19th, Pg. 1167) Q 14. Association of sexual precocity, multiple cystic bone lesions and endocrinopathies are seen in: A. McCune-Albright’s syndrome B. Granulosa cell tumor C. Androblastoma D. Hepatoblastoma Ans. is A. McCune-Albright’s syndrome (Ref: Nelson 19th, Pg. 1892) Q 15. Berger nepbropathy is due to mesangial deposition of: A. Fibrin and C3 B. IgD and C3 C. IgE and C3 D. IgA and C3 Ans. is D. IgA and C3 (Ref: O.P. Ghai 7th, Pg. 446) Q 16. Commonest cause of heart failure in infancy is: A. Myocarditis B. Rheumatic fever C. Cardiomyopathy D. Congenital heart disease Ans. is D. Congenital heart disease (Ref: O.P. Ghai 7th, Pg. 372) 162 Pre-NEET Pediatrics

Q 17. Congenital long QT syndrome is associated with neonatal: A. Sinus bradycardia B. Sinus tachycardia C. Supra ventricular tachycardia D. Ventricular tachycardia Ans. is A. Sinus bradycardia (Ref: Nelson 19th, Pg. 1617) Q 18. Pawn ball megakaryocytes are characteristic of: A. Myelodysplastic syndrome B. Idiopathic thrombocytopenic purpura C. Thrombotic thrombocytopenic purpura D. Chloramphenicol toxicity Ans. is A. Myelodysplastic syndrome

PAEDIATRICS 2006

Q 1. Commonest cause of meningitis in postneonatal period is: A. Mycobacterium tuberculosis B. Staph. aureus C. Str. pneumonae D. Klebsiella Ans is C. Str. Pneumonae (Ref: O.P. Ghai 7th, Pg. 536) Q 2. Epiphyseal dysgenesis is a pathognomonic feature of: A. Hypoparathyroidism B. Hyperparathyroidism C. Hypothyroidism D. Hyperthyroidism Ans. is C. Hypothyroidism (Ref: Nelson 19th, Pg. 1899) Q 3. In Down’s syndrome, following congenital defect is common: A. PDA B. PS Previous Year’s Questions of DNB 163

C. ASD D. VSD Ans. is C. ASD (Ref: IAP textbook of Pediatrics 4th, Pg. 994) Q 4.Craniotabes is found in children with the following conditions except: A. Rickets B. Hydrocephalus C. Syphilis D. Kernicterus Ans. is D. Kernicterus (Ref: Nelson 19th, Pg. 200) Q 5. Commonest haematological malignancy in children is: A. CLL B. AML C. CML D. ALL Ans. is D. ALL (Ref: Nelson 19th, Pg. 1732) Q 6. Commonest cause of stridor in a new born is: A. Laryngomalacia B. Foreign body C. Meconium aspiration D. Recurrent laryngeal nerve palsy due to birth Ans. is A. Laryngomalacia (Ref: O.P. Ghai 7th, Pg. 340

PAEDIATRICS 2005

Q 1. The most important cause of under 5 mortality is: A. Diarrhoea B. Malnutrition C. Respiratory infections D. Trauma Ans. is C. Respiratory infections (Ref: O.P. Ghai 7th, Pg. 356) 164 Pre-NEET Pediatrics

Q 2. Commonest cause of stridor in a new born is: A. Laryngomalacia B. Foreign body C. Meconium aspiration D. Recurrent laryngeal nerve palsy due to birth Ans. is A. Laryngomalacia (Ref: O.P. Ghai 7th, Pg. 340) Q 3. 15 months old child do all, except: A. Feeds self with spoon B. Says three words C. Builds tower of 2 blocks D. Creeps upstairs Ans. is A. Feeds self with spoon (Ref: Nelson 19th, Pg. 32) Q 4. Commonest nephrotic syndrome in child: A. Minimal change B. Chronic glomerulonephritis C. Hemolytic uremic syndrome D. Congenital Ans. is A. Minimal change (Ref: O.P. Ghai 7th, Pg. 451) Q 5. A six years old girl child presents with spotting, no secondary sexual characteristic present, cause can be: A. Menarche B. Foreign body C. Gonococcal infection D. Haemorrhagic disease Ans. is B. Foreign body (Ref: Forfar 6th, Pg. 1152) Q 6. Best method of diagnosis of childhood HIV. A. CD4 cell counts B. P24 antigen C. ELISA D. Anti HIV antibody Ans. is B. P24 antigen (Ref: Nelson 19th, Pg. 1167) Previous Year’s Questions of DNB 165

Q 7. Attainment of weight of a preschool normal child is: A. 2-2.5 Kg B. 3-3.5 Kg C. 4-4.5 Kg D. 5-5.5 Kg Ans. is B. 3-3.5 Kg (Ref: IAP textbook of Pediatrics 4th, Pg.84)

PAEDIATRICS 2004

Q 1. Craniotabes is found in children with the following conditions except: A. Rickets B. Hydrocephalus C. Syphilis D. Kernicterus Ans. is D. Kernicterus (Ref: Nelson 19th, Pg. 200) Q 2. In minimal change disease, correct is: A. Most cases recover spontaneously B. 20% go to CRF C. 20% develop local glomerulosclerosis D. 90% cases best respond to short course of steroid therapy Ans. is D. 90% cases best respond to short course of steroid therapy (Ref: Nelson 19th, Pg. 1803) Q 3. Commonest haematological malignancy in children is: A. CLL B. AML C. CML D. ALL Ans. is D. ALL (Ref: Nelson 19th, Pg. 1732) Q 4. Metabolic acidosis is accompanied with: A. Acetazolamide B. Phenformin 166 Pre-NEET Pediatrics

C. Verapamil D. Triamterene Ans. is A. Acetazolamide (Ref: O.P. Ghai 7th, Pg. 55) Q 5. The commonest cause of death in diphtheric child is: A. IIIrd nerve palsy B. Myocarditis C. Tonsillitis D. Septicemia Ans. is B. Myocarditis (Ref: IAP textbook of Pediatrics 4th, Pg. 364) Q 6. Unconjugated hyperbilirubinemia in newborn is caused by following, except: A. Breast milk jaundice B. Galactosemia C. Sphereocytosis D. Gilbert’s syndrome Ans. is B. Galactosemia (Ref: Nelson 19th, Pg. 1376) Q 7. Commonest cause of meningitis in postneonatal period is: A. Mycobacterium tuberculosis B. Staph. aureus C. Str. pneumoniae D. Klebsiella Ans. is C. Str. Pneumonia (Ref: Nelson 19th, Pg. 2087) Q 8. Sure sign of CCF in a infant is: A. Basal crepts B. JVP C. Pedal oedema D. Liver enlargement Ans. is D. Liver enlargement (Ref: O.P. Ghai 7th, Pg. 375) Previous Year’s Questions of DNB 167

PAEDIATRICS 2003

Q 1. Epiphyseal dysgenesis is a pathognomonic feature of: A. Hypoparathyroidism B. Hyperparathyroidism C. Hypothyroidism D. Hyperthyroidism Ans. is C. Hypothyroidism (Ref: Nelson 19th, Pg. 1899) Q 2. Commonest cause of systemic hypertension in children is: A. Coarctation of aorta B. Acute glomerulonephritis C. Nephrotic syndrome D. Lactic acidosis Ans. is B. Acute glomerulonephritis (Ref: O.P. Ghai 7th, Pg. 434) Q 3. Organism in bronchiolitis is: A. Adeno virus B. Influenza virus C. Rhino virus D. RSV Ans. is D. RSV (Ref: Nelson 19th, Pg.1456) Q 4. Steroids is useful in: A. Post-streptococcal glomerulonephritis B. Membranous glomerulonephritis C. Rapidly progressing glomerulonephritis D. Minimal change type Ans. is D. Minimal change type (Ref: Nelson 19th, Pg. 1803) Q 5. Not seen in Fallot’s tetralogy: A. ASD B. VSD 168 Pre-NEET Pediatrics

C. Pulmonary stenosis D. Left ventricular hypertrophy Ans. is A. ASD (Ref: Nelson 19th, Pg. 1573) Q 6. In Down’s syndrome, following congenital defect is common: A. PDA B. PS C. ASD D. VSD Ans. is C. ASD (Ref: IAP textbook of Pediatrics 4th, Pg. 994) Q 7. Following are causes of Pan systolic murmur, except: A. MR B. MS C. VSD D. TR Ans. is B. MS (Ref: Nelson 19th, Pg. 1556, 1627, 1628) Q 8. Precocious puberty is seen in: A. Hyperthyroidism B. Addison’sdisease C. McCune Albright syndrome D. Neuroblastoma Ans. is C. McCune Albright syndrome (Ref: Nelson 19th, Pg. 1892) Q 9. Craniotabes is found in children with the following conditions, except: A. Rickets B. Hydrocephalus C. Syphilis D. Kernicterus Ans. is D. Kernicterus (Ref: Nelson 19th, Pg. 1072) Previous Year’s Questions of DNB 169

PAEDIATRICS 2002

Q 1. The commonest cause of death in diphtheric child is: A. IIIrd-nerve palsy B. Myocarditis C. Tonsillitis D. Septicemia Ans. is B. Myocarditis (Ref: IAP textbook of Pediatrics 4th, Pg. 364) Q 2. Best method of diagnosis of childhood HIV: A. CD 4 cell counts B. P24 antigen C. ELISA D. Anti HlV antibody Ans. is B. P24 antigen (Ref: Nelson 19th, Pg. 1167) Q 3. Hyaline membrane is seen in all of the following conditions, except: A. Radiation pneumonitis B. Viral pneumonitis C. Uremic pneumonitis D. Staphylococcal bronchopneumonia Ans. is D. Staphylococcal bronchopneumonia

Q 4. Attainment of weight is a preschool normal child is: A. 2-2.5 Kg B. 3-3.5 Kg C. 4-4.5 Kg D. 5-5.5 Kg Ans. is B. 3-3.5 Kg (Ref: IAP textbook of Pediatrics 4th, Pg. 84) Q 5. Subacute sclerosing panencephalit is complication of: A. Pneumonia B. Measles 170 Pre-NEET Pediatrics

C. Diphtheria D. Pertussis Ans. is B. Measles (Ref: Nelson 19th, Pg. 1072) Q 6. Casoni’s test is diagnostic of: A. Echinococcus granulosum B. Toxoplasmosis C. Toxocariasis D. Syphilis Ans. is A. Echinococcus granulosum

PAEDIATRICS 2001

Q 1. A child with diarrhoea has deep and rapid respiration. Diagnosis is: A. Metabolic alkalosis B. Metabolic acidosis C. Respiratory alkalosis D. Respiratory acidosis Ans. is B. Metabolic acidosis (Ref: Nelson 19th, Pg. 231) Q 2. The percentage rise in length of infant in first year of life is: A. 20% B. 30% C. 40% D. 50% Ans. is D. 50% (Ref: IAP textbook of paediatrics 4th, Pg. 84) Q 3.The most important cause of under 5 mortality is: A. Diarrhoea B. Malnutrition C. Respiratory infections D. Trauma Ans. is C. Respiratory infections (Ref: O.P. Ghai 7th, Pg. 350) Previous Year’s Questions of DNB 171

Q 4. A neonate after 12 hrs. of birth passes black colored meconium. True is: A. Intestinal haemorrhage B. Fibrocystic disease of pancreas C. Normal finding D. Hirschsprung’s disease Ans. is C. Normal finding (Ref. O.P. Ghai 7th, Pg. 105) Q 5. All of the following are used to assess IUGR, except: A. Fetal movements B. Head size C. Fundus height D. Liquor volume Ans. is A. Fetal movements (Ref: Avery 8th, Pg. 36) Q 6. Sure sign of CCF in an infant is: A. Basal crepts B. JVP C. Pedal oedema D. Liver enlargement Ans. is D. Liver enlargement (Ref: O.P. Ghai 7th, Pg. 375) Q 7.Commonest cause of stridor in a new born is: A. Laryngomalacia B. Foreign body C. Meconium aspiration D. Recurrent laryngeal nerve palsy due to birth Ans. is A. Laryngomalacia (Ref: Nelson 19th, Pg. 1450) Q 8. 15 months old child do all, except: A. Feeds self with spoon B. Says three words C. Builds tower of 2 blocks D. Creeps upstairs Ans. is A. Feeds self with spoon (Ref: Nelson 19th, Pg. 32) 172 Pre-NEET Pediatrics

Q 9. At what age does a child sees the toy hidden and then again hides it and gives to mother if she asks for: A. 6 months B. 8 months C. 10 months D. 12 months Ans. is D. 12 months (Ref: Forfar 6th, Pg. 119) Q 10. Commonest nephrotic syndrome in child: A. Minimal change B. Chronic glomerulonephritis C. Hemolytic uremic syndrome D. Congenital Ans. is A. Minimal change (Ref: Nelson 19th, Pg. 1804) Q 11. Commonest haematological malignancy in children is: A. CLL B. AML C. CML D. ALL Ans. is D. ALL (Ref: Nelson 19th, Pg. 1732) Q 12. A six years old girl child presents with spotting, no secondary sexual characteristic present. Cause can be: A. Menarche B. Foreign body C. Gonococcal infection D. Haemorrhagic disease Ans. is B. Foreign body (Ref: For far 6th, Pg. 1152) Q 13. Metabolic acidosis is accompanied with: A. Acetazolamide B. Phenformin Previous Year’s Questions of DNB 173

C. Verapamil D. Triamterene Ans. is A. Acetazolamide (Ref: O.P. Ghai 7th, Pg. 55

PAEDIATRICS 2000

Q 1. Not seen in Fallot’s tetralogy: A. ASD B. VSD C. Pulmonary stenosis D. Left ventricular hypertrophy Ans. is A. ASD (Ref: Nelson 19th, Pg. 1573) Q 2. Congestive heart failure in children is best diagnosed by: A. Tachycardia and tender hepatomegaly B.  JVP C.  JVP + pedal edema D. Hypotension Ans. is A. Tachycardia and tender hepatomegaly (Ref: O.P. Ghai 7th, Pg. 375) Q 3. APGAR score of a child born blue with HR 70/ mt. floppy with feeble cry with grimacing on nasal suction is: A. 3 B. 2 C. 4 D. 5 Ans is A. 3 (Ref: Nelson 19th, Pg. 536) Q 4. In Down’s syndrome, following congenital defect is common: A. PDA B. PS 174 Pre-NEET Pediatrics

C. ASD D. VSD Ans. is C. ASD (Ref: IAP textbook of paediatrics 4th, Pg. 994) Q 5. Following is cause of pan systolic murmur: A. MR B. MS C. VSD D. ASD E. TR Ans. is A. MR, C. VSD & E. TR (Ref: Nelson 19th, Pg. 1556, 1627, 1628) Q 6. In hyperparathyroidism, seen is:

++ - Ca PO4

A. 

B. 

C. 

D. 

Ans is A. (Ref: Nelson 19th, Pg. 1921) Q 7. Precocious puberty is seen in: A. Hyperthyroidism B. Addison’s disease C. McCune Albright syndrome D. Neuroblastoma Ans. is C. McCune Albright syndrome (Ref: Nelson 19th, Pg. 1892)